You are on page 1of 52

‫‪ -I‬النهايات‬

‫‪ .1‬بعض نهايات الدوال المرجعية‬

‫∞‪lim 𝑥 = −‬‬ ‫∞‪lim 𝑥 = +‬‬ ‫∞‪lim √𝑥 = +‬‬


‫∞‪𝑥→−‬‬ ‫∞‪𝑥→+‬‬ ‫∞‪𝑥→+‬‬

‫∞‪lim 𝑥 2 = +‬‬ ‫∞‪lim 𝑥 2 = +‬‬ ‫∞‪lim 𝑥 3 = −‬‬ ‫∞‪lim 𝑥 3 = +‬‬


‫∞‪𝑥→−‬‬ ‫∞‪𝑥→+‬‬ ‫∞‪𝑥→−‬‬ ‫∞‪𝑥→+‬‬

‫‪1‬‬ ‫‪1‬‬ ‫‪1‬‬ ‫‪1‬‬


‫‪lim‬‬ ‫‪=0‬‬ ‫‪lim‬‬ ‫‪=0‬‬ ‫‪lim‬‬ ‫∞‪= −‬‬ ‫‪lim‬‬ ‫∞‪= +‬‬
‫𝑥 ‪𝑥 →0‬‬ ‫𝑥 ‪𝑥→0‬‬
‫<‬ ‫>‬
‫𝑥 ∞‪𝑥→−‬‬ ‫𝑥 ∞‪𝑥→+‬‬

‫‪ .2‬نهاية دالة كثير حدود أو دالة ناطقة عند ∞‪ +‬أو ∞‪−‬‬

‫النهاية عند ∞ )‪ (±‬لدالة كثير حدود هي نهاية حد ّها األعلى درجة عند ∞ )‪(±‬‬
‫النهاية عند ∞ )‪ (±‬لدالة ناطقة هي نهاية حاصل قسمة الحد ّين األعلى درجة عند ∞ )‪(±‬‬
‫مثال ‪lim 3𝑥 2 + 2𝑥 + 7 = lim 3𝑥 2 = +∞ :1‬‬
‫∞‪𝑥→−‬‬ ‫∞‪𝑥→−‬‬
‫‪𝑥 2 +4𝑥+4‬‬ ‫‪𝑥2‬‬
‫‪lim‬‬ ‫‪= lim‬‬ ‫مثال ‪= lim 𝑥 = +∞ :2‬‬
‫∞‪𝑥→+‬‬ ‫‪𝑥−2‬‬ ‫𝑥 ∞‪𝑥→+‬‬ ‫∞‪𝑥→+‬‬

‫‪ .3‬حاالت عدم التعيين‬

‫∞‬ ‫‪0‬‬
‫∞‪①∞−‬‬ ‫;‬ ‫∞×‪②0‬‬ ‫;‬ ‫③‬ ‫;‬ ‫④‬
‫∞‬ ‫‪0‬‬

‫‪ .4‬طرق إزالة حاالت عدم التعيين‬


‫أ‪ .‬التحليل واالختزال‬
‫)𝑥(𝑓‬
‫غالبا نستعمل هذه الطريقة عند حساب )𝑥(𝑔 ‪ lim‬حيث ‪𝑓(𝑥0 ) = 𝑔(𝑥0 ) = 0‬‬
‫‪𝑥→𝑥0‬‬
‫في هذه الحالة نقوم بتحليل العبارتين 𝑓 و𝑔 وكتابتهما على الشكل )𝑥(𝑄) ‪ (𝑥 − 𝑥0‬ثم نختزل الكسر ونحسب النهاية‬
‫من جديد‪.‬‬
‫‪4𝑥 2 +3𝑥−7‬‬ ‫)‪(𝑥−1)(4𝑥+7‬‬ ‫‪4𝑥+7‬‬ ‫‪11‬‬
‫‪lim‬‬ ‫‪2 +2𝑥−3 = lim (𝑥−1)(𝑥+3) = lim 𝑥+3‬‬
‫=‬ ‫مثال ‪:1‬‬
‫‪𝑥→1‬‬ ‫𝑥‬ ‫‪𝑥→1‬‬ ‫‪𝑥→1‬‬ ‫‪4‬‬
‫‪𝑥 3 +8‬‬ ‫)‪(𝑥+2)(𝑥 2 −2𝑥+4‬‬ ‫‪𝑥 2 −2𝑥+4‬‬ ‫‪12‬‬
‫‪lim‬‬ ‫‪= lim‬‬ ‫‪= lim‬‬ ‫مثال ‪= −4 = −3 :2‬‬
‫‪𝑥→−2 𝑥 2 −4‬‬ ‫‪𝑥→−2‬‬ ‫)‪(𝑥+2)(𝑥−2‬‬ ‫‪𝑥→−2‬‬ ‫‪𝑥−2‬‬

‫مالحظة ‪ :‬توجد ثالث طرق إليجاد عبارة )𝑥(𝑄 وهي‪:‬‬


‫‪ )1‬المطابقة ‪:‬‬
‫‪2‬‬
‫مثال‪ :‬كتابة العبارة ‪ 4𝑥 + 3𝑥 − 7‬على الشكل )𝑥(𝑄)‪(𝑥 − 1‬‬
‫بما أ ّن )𝑥(𝑄 من الدرجة األولى فهو يُكتب على شكل 𝑏 ‪ ، 𝑎𝑥 +‬منه ‪:‬‬
‫𝑏 ‪(𝑥 − 1)𝑄(𝑥) = (𝑥 − 1)(𝑎𝑥 + 𝑏) = 𝑎𝑥 2 + (𝑏 − 𝑎)𝑥 −‬‬
‫‪𝑎=4‬‬
‫‪𝑎=4‬‬
‫{ ⇒ ‪{𝑏 − 𝑎 = 3‬‬ ‫)‪⇒ 4𝑥 2 + 3𝑥 − 7 = (𝑥 − 1)(4𝑥 + 7‬‬
‫‪𝑏=7‬‬
‫‪−𝑏 = −7‬‬
‫الحظ أنّه ل ّما يكون كثير الحدود من الدرجة الثانية ‪ ،‬يمكن تحليله مباشرة إلى جداء عاملين من الدرجة األولى على النحو التالي‪:‬‬
‫نقسم ‪ 4𝑥 2‬على 𝑥 فنحصل على 𝑥‪ 4‬ونقسم ‪ −7‬على ‪ −1‬فنحصل على ‪ ، +7‬فتكون النتيجة ‪:‬‬
‫)‪4𝑥 2 + 3𝑥 − 7 = (𝑥 − 1)(4𝑥 + 7‬‬

‫‪9‬‬
‫‪ )2‬القسمة اإلقليدية ‪:‬‬
‫‪𝑥3 + 8‬‬ ‫‪𝑥+2‬‬
‫‪−𝑥 3 − 2𝑥 2‬‬ ‫‪2‬‬
‫‪𝑥 − 2𝑥 + 4‬‬
‫‪−2𝑥 2 + 8‬‬
‫𝑥‪2𝑥 2 + 4‬‬
‫‪4𝑥 + 8‬‬
‫‪−4𝑥 − 8‬‬
‫‪0‬‬

‫‪ )3‬خوارزمية هورنر‪ :‬وهي أسهل الطرق على اإلطالق خاصة ل ّما يكون كثير الحدود من الدرجة الثالثة فما فوق‪.‬‬
‫مثال ‪𝑥0 = 1 ، 𝑃(𝑥) = 4𝑥 2 + 3𝑥 − 7 :1‬‬

‫‪𝑎=4‬‬ ‫‪𝑏=3‬‬ ‫‪𝑐 = −7‬‬ ‫معامالت )𝑥(𝑃‬


‫‪𝑥0 = 1‬‬ ‫↓‬ ‫الجذر ‪𝑥0‬‬
‫‪′‬‬
‫‪𝑎 =4‬‬ ‫‪𝑏′ = 7‬‬ ‫𝟎 = ‪𝑐′‬‬ ‫معامالت )𝑥(𝑄‬

‫‪𝑎′ = 𝑎 = 4‬‬
‫)‪{𝑏 ′ = 𝑥0 . 𝑎′ + 𝑏 = 1(4) + 3 = 7 ⇒ 4𝑥 2 + 3𝑥 − 7 = (𝑥 − 1)(4𝑥 + 7‬‬
‫𝟎 = ‪𝑐 ′ = 𝑥0 . 𝑏 ′ + 𝑐 = 1(7) − 7‬‬
‫مثال ‪𝑥0 = −2 ، 𝑃(𝑥) = 𝑥 3 + 8 :2‬‬

‫‪𝑎=1‬‬ ‫‪𝑏=0‬‬ ‫‪𝑐=0‬‬ ‫‪𝑑=8‬‬ ‫معامالت )𝑥(𝑃‬


‫‪𝑥0 = −2‬‬ ‫↓‬ ‫الجذر ‪𝑥0‬‬
‫‪𝑎′ = 1‬‬ ‫‪𝑏 ′ = −2‬‬ ‫‪𝑐′ = 4‬‬ ‫𝟎 = ‪𝑑′‬‬ ‫معامالت )𝑥(𝑄‬

‫‪𝑎′ = 𝑎 = 1‬‬
‫‪𝑏 ′ = 𝑥0 . 𝑎′ + 𝑏 = −2(1) + 0 = −2‬‬
‫‪{ ′‬‬ ‫)‪⇒ 𝑥 3 + 8 = (𝑥 + 2)(𝑥 2 − 2𝑥 + 4‬‬
‫‪𝑐 = 𝑥0 . 𝑏 ′ + 𝑐 = −2(−2) + 0 = 4‬‬
‫𝟎 = ‪𝑑′ = 𝑥0 . 𝑐 ′ + 𝑑 = −2(4) + 8‬‬
‫الحظ أ ّن المعامل األخير يكون دائما معدوما ) ‪ 𝑐 ′‬و ‪(𝑑 ′‬‬

‫ب‪ .‬استعمال المرافق (خاصة بالجذور التربيعية)‬


‫مثال ‪:1‬‬
‫‪√𝑥 − 1 − 1‬‬ ‫)‪(√𝑥 − 1 − 1)(√𝑥 − 1 + 1‬‬
‫‪lim‬‬ ‫‪= lim‬‬
‫‪𝑥→2‬‬ ‫‪𝑥−2‬‬ ‫‪𝑥→2‬‬ ‫)‪(𝑥 − 2)(√𝑥 − 1 + 1‬‬
‫‪𝑥−2‬‬ ‫‪1‬‬ ‫‪1‬‬
‫‪= lim‬‬ ‫‪= lim‬‬ ‫=‬
‫𝑥( ‪𝑥→2‬‬ ‫)‪− 2)(√𝑥 − 1 + 1‬‬ ‫𝑥√( ‪𝑥→2‬‬ ‫)‪− 1 + 1‬‬ ‫‪2‬‬
‫مثال ‪:2‬‬
‫‪√5 − 𝑥 − 2‬‬ ‫)‪(√5 − 𝑥 − 2)(√5 − 𝑥 + 2)(√𝑥 + 8 + 3‬‬
‫‪lim‬‬ ‫‪= lim‬‬
‫𝑥√ ‪𝑥→1‬‬ ‫‪+8−3‬‬ ‫𝑥√( ‪𝑥→1‬‬ ‫)‪+ 8 − 3)(√𝑥 + 8 + 3)(√5 − 𝑥 + 2‬‬
‫)‪(1 − 𝑥)(√𝑥 + 8 + 3‬‬ ‫)‪(√𝑥 + 8 + 3‬‬ ‫‪3‬‬
‫‪= lim‬‬ ‫‪= lim −‬‬ ‫‪= −‬‬
‫𝑥( ‪𝑥→1‬‬ ‫)‪− 1)(√5 − 𝑥 + 2‬‬ ‫‪𝑥→1‬‬ ‫)‪(√5 − 𝑥 + 2‬‬ ‫‪2‬‬

‫مالحظة ‪ :‬ل ّما يؤول 𝑥 إلى ∞ ‪ ،‬إ ّما أ ّن نستعمل المرافق في حالة تساوي معامالت 𝑥 داخل الجذر وخارجه (المثال ‪ )0‬أو نستعمل‬
‫التحليل في حالة عدم تساوي المعامالت (المثال ‪)2‬‬

‫‪01‬‬
‫مثال ‪:1‬‬
‫‪(𝑥 + 1 −‬‬ ‫‪√𝑥 2‬‬ ‫‪+ 𝑥 − 2)(𝑥 + 1 +‬‬ ‫‪√𝑥 2‬‬ ‫)‪+ 𝑥 − 2‬‬
‫‪lim 𝒙 + 1 − √𝒙𝟐 + 𝑥 − 2 = lim‬‬
‫∞‪𝑥→+‬‬ ‫∞‪𝑥→+‬‬ ‫)‪(𝑥 + 1 + √𝑥 2 + 𝑥 − 2‬‬
‫)‪(𝑥 + 1)2 − (𝑥 2 + 𝑥 − 2‬‬ ‫‪𝑥+3‬‬
‫‪= lim‬‬ ‫‪= lim‬‬
‫∞‪𝑥→+‬‬ ‫‪1 2‬‬ ‫∞‪𝑥→+‬‬ ‫‪1 2‬‬
‫) ‪𝑥 + 1 + √𝑥 2 (1 + 𝑥 − 2‬‬ ‫‪𝑥 + 1 + |𝑥|√1 + 𝑥 − 2‬‬
‫𝑥‬ ‫𝑥‬
‫‪3‬‬
‫)𝑥 ‪𝑥 (1 +‬‬
‫‪= lim‬‬ ‫)𝑥 = |𝑥| ⇒ ∞‪(𝑥 → +‬‬
‫∞‪𝑥→+‬‬ ‫√ ‪1‬‬ ‫‪1 2‬‬
‫) ‪𝑥 (1 + 𝑥 + 1 + 𝑥 − 2‬‬
‫𝑥‬
‫‪3‬‬
‫𝑥‪1+‬‬ ‫‪1‬‬
‫‪= lim‬‬ ‫=‬
‫∞‪𝑥→+‬‬ ‫‪1‬‬ ‫‪1 2‬‬ ‫‪2‬‬
‫‪1 + 𝑥 + √1 + 𝑥 − 2‬‬
‫𝑥‬

‫مثال ‪:2‬‬

‫‪1 2‬‬
‫‪lim 𝟐𝒙 + 1 + √𝒙𝟐 + 𝑥 − 2 = lim 2𝑥 + 1 + √𝑥 2 (1 +‬‬ ‫) ‪−‬‬
‫∞‪𝑥→−‬‬ ‫∞‪𝑥→−‬‬ ‫‪𝑥 𝑥2‬‬

‫‪1 2‬‬
‫‪= lim 2𝑥 + 1 + |𝑥|√1 +‬‬ ‫‪−‬‬
‫∞‪𝑥→−‬‬ ‫‪𝑥 𝑥2‬‬

‫‪1 2‬‬
‫‪= lim 2𝑥 + 1 − 𝑥√1 +‬‬ ‫)𝑥‪− (𝑥 → −∞ ⇒ |𝑥| = −‬‬
‫∞‪𝑥→−‬‬ ‫‪𝑥 𝑥2‬‬

‫‪1‬‬ ‫‪1 2‬‬


‫∞‪= lim 𝑥 (2 + − √1 + − 2 ) = −‬‬
‫∞‪𝑥→−‬‬ ‫𝑥‬ ‫𝑥 𝑥‬
‫⏟‬
‫‪→1‬‬

‫ج‪ .‬استعمال العدد المشتق‬


‫) ‪𝑓(𝑥)−𝑓(𝑥0‬‬
‫‪ ، lim‬وهذه النهاية تساوي العدد المشتق ) ‪𝑓′(𝑥0‬‬ ‫الستعمال هذه الطريقة ال بد ّ أن تكون النهاية من الشكل ‪:‬‬
‫‪𝑥→𝑥0‬‬ ‫‪𝑥−𝑥0‬‬
‫مثال ‪:1‬‬
‫‪√𝑥 − 1 − 1‬‬ ‫)‪𝑓(𝑥) − 𝑓(2‬‬
‫‪lim‬‬ ‫‪= lim‬‬ ‫)‪= 𝑓 ′ (2‬‬
‫‪𝑥→2‬‬ ‫‪𝑥−2‬‬ ‫‪𝑥→2‬‬ ‫‪𝑥−2‬‬
‫‪1‬‬ ‫‪1‬‬
‫= )𝑥( ‪𝑓(𝑥) = √𝑥 − 1 ; 𝑓(2) = 1 ; 𝑓 ′‬‬ ‫= )‪; 𝑓 ′ (2‬‬
‫‪2√𝑥 − 1‬‬ ‫‪2‬‬
‫مثال ‪:2‬‬
‫𝑥 ‪sin‬‬ ‫)‪𝑓(𝑥) − 𝑓(0‬‬
‫‪lim‬‬ ‫‪= lim‬‬ ‫)‪= 𝑓 ′ (0‬‬
‫𝑥 ‪𝑥→0‬‬ ‫‪𝑥→0‬‬ ‫‪𝑥−0‬‬
‫‪𝑓(𝑥) = sin 𝑥 ; 𝑓(0) = 0 ; 𝑓 ′ (𝑥) = cos 𝑥 ; 𝑓 ′ (0) = 1‬‬

‫‪ .5‬نهاية دالة مركّبة‬


‫𝒃 = )𝑥(𝑢 ‪lim‬‬
‫𝑎→𝑥{ ; 𝑢 𝑜 𝑣 = 𝑓‬ ‫𝑐 = )𝑥(𝑓 ‪⇒ lim‬‬
‫𝑐 = )𝑥(𝑣 ‪lim‬‬ ‫𝑎→𝑥‬
‫𝒃→𝑥‬

‫‪00‬‬
‫مثال‪:‬‬
‫‪2‬‬
‫‪4𝑥 − 2𝑥 − 1‬‬
‫‪lim‬‬ ‫𝟒=‬
‫‪4𝑥 2 − 2𝑥 − 1‬‬ ‫⏟ ∞‪𝑥→+‬‬
‫‪𝑥 2 − 5𝑥 + 3‬‬
‫‪lim √ 2‬‬ ‫;‬ ‫)𝑥(𝑢‬ ‫‪⇒ lim 𝑓(𝑥) = 2‬‬
‫∞‪𝑥→+‬‬
‫‪⏟ 𝑥 − 5𝑥 + 3‬‬ ‫√ ‪lim‬‬
‫‪⏟𝑥 = 2‬‬
‫∞‪𝑥→+‬‬

‫)𝑥(𝑓‬ ‫𝟒→𝑥‬
‫{‬ ‫)𝑥(𝑣‬
‫‪ .6‬النهايات بالمقارنة‬
‫الحالة األولى‪:‬‬
‫)𝑥(‪𝑔(𝑥) ≤ 𝑓(𝑥) ≤ ℎ‬‬
‫𝑙 = )𝑥(𝑓 ‪{ lim 𝑔(𝑥) = lim ℎ(𝑥) = 𝑙 ⇒ lim‬‬
‫∞‪𝑥→+‬‬
‫∞‪𝑥→+‬‬ ‫∞‪𝑥→+‬‬
‫مثال‪:‬‬
‫𝑥 ‪2𝑥+cos‬‬
‫𝑥‪: lim 1+‬‬ ‫احسب‬
‫∞‪𝑥→+‬‬
‫‪2𝑥 − 1 2𝑥 + cos 𝑥 2𝑥 + 1‬‬
‫⇒ ‪−1 ≤ cos 𝑥 ≤ 1 ⇒ 2𝑥 − 1 ≤ 2𝑥 + cos 𝑥 ≤ 2𝑥 + 1‬‬ ‫≤‬ ‫≤‬
‫𝑥‪1+‬‬ ‫𝑥‪1+‬‬ ‫𝑥‪1+‬‬
‫مالحظة ‪ :‬المتراجحة لم تتغيّر أل ّن ‪ ، (𝑥 → +∞) 1 + 𝑥 > 0‬أما إذا كان ∞‪ 𝑥 → −‬فإ ّن ‪ ، 1 + 𝑥 < 0‬منه تتغيّر‬
‫المتراجحة عند القسمة على )𝑥 ‪.(1 +‬‬
‫‪2𝑥 − 1 2𝑥 + cos 𝑥 2𝑥 + 1‬‬
‫≤‬ ‫≤‬
‫{‬ ‫‪1‬‬ ‫‪+‬‬ ‫𝑥‬ ‫‪1‬‬ ‫‪+‬‬ ‫𝑥‬ ‫‪1 + 𝑥 ⇒ lim 2𝑥 + cos 𝑥 = 2‬‬
‫‪2𝑥 − 1‬‬ ‫‪2𝑥 + 1‬‬ ‫∞‪𝑥→+‬‬ ‫𝑥‪1+‬‬
‫‪lim‬‬ ‫‪= lim‬‬ ‫‪=2‬‬
‫𝑥 ‪𝑥→+∞ 1 +‬‬ ‫𝑥 ‪𝑥→+∞ 1 +‬‬
‫الحالة الثانية‪:‬‬
‫)𝑥(𝑔 ≥ )𝑥(𝑓‬
‫∞‪{ lim 𝑔(𝑥) = +∞ ⇒ lim 𝑓(𝑥) = +‬‬
‫∞‪𝑥→+‬‬
‫∞‪𝑥→+‬‬
‫مثال‪:‬‬
‫𝑥‬
‫‪: lim‬‬ ‫احسب‬
‫𝑥 ‪𝑥→+∞ 2−sin‬‬
‫‪1‬‬ ‫‪1‬‬ ‫𝑥‬ ‫‪1‬‬
‫≤ ⇒ ‪−1 ≤ sin 𝑥 ≤ 1 ⇒ −1 ≤ − sin 𝑥 ≤ 1 ⇒ 1 ≤ 2 − sin 𝑥 ≤ 3‬‬ ‫≤ ⇒‪≤1‬‬ ‫𝑥≤‬
‫𝑥 ‪3 2 − sin‬‬ ‫𝑥 ‪3 2 − sin‬‬
‫𝑥‬ ‫𝑥‬
‫≥‬ ‫𝑥‬
‫‪{ 2 − sin‬‬
‫𝑥‬
‫‪𝑥 3 ⇒ lim‬‬ ‫∞‪= +‬‬
‫𝑥 ‪𝑥→+∞ 2 − sin‬‬
‫∞‪lim = +‬‬
‫‪𝑥→+∞ 3‬‬
‫الحالة الثالثة‪:‬‬
‫)𝑥(𝑔 ≤ )𝑥(𝑓‬
‫∞‪{ lim 𝑔(𝑥) = −∞ ⇒ lim 𝑓(𝑥) = −‬‬
‫∞‪𝑥→+‬‬
‫∞‪𝑥→+‬‬
‫مثال‪:‬‬
‫‪2‬‬
‫احسب 𝑥 ‪: lim −2𝑥 + 𝑥 cos‬‬
‫∞‪𝑥→−‬‬
‫⇒ ‪−1 ≤ cos 𝑥 ≤ 1‬‬ ‫𝑥 ‪𝑥 ≤ 𝑥 cos 𝑥 ≤ −𝑥 ⇒ −2𝑥 2 + 𝑥 ≤ −2𝑥 2 + 𝑥 cos 𝑥 ≤ −2𝑥 2 −‬‬
‫)‪(𝑥<0‬‬
‫𝑥 ‪−2𝑥 2 + 𝑥 cos 𝑥 ≤ −2𝑥 2 −‬‬
‫{‬ ‫∞‪lim −2𝑥 2 − 𝑥 = −∞ ⇒ 𝑥→−‬‬‫∞‪lim −2𝑥 2 + 𝑥 cos 𝑥 = −‬‬
‫∞‪𝑥→−‬‬
‫مالحظة ‪ :‬غالبا ما نستعمل المقارنة لحساب نهايات الدوال المثلثية )𝑥 ‪ (sin 𝑥 ; cos‬ل ّما 𝑥 يؤول إلى ∞)‪ ، (±‬حيث ّ‬
‫أن هذه‬
‫الدوال ال تقبل نهاية عند ∞)‪.(±‬‬
‫ولحصر )𝑥(𝑓 دائما ننطلق من حصر الدالة المثلثية )𝑥 ‪ (sin 𝑥 ; cos‬بين )‪ (−1‬و)‪(+1‬‬
‫‪ .7‬المستقيمات المقاربة‬
‫المستقيم 𝒂 = 𝒙 مستقيم مقارب عمودي )يوازي محور التراتيب( ⇒ ∞ = )𝑥(𝑓 ‪1) lim‬‬
‫𝑎→𝑥‬
‫المستقيم 𝒃 = 𝒚 مستقيم مقارب أفقي )يوازي محور الفواصل( ⇒ 𝑏 = )𝑥(𝑓 ‪2) lim‬‬
‫∞→𝑥‬
‫المستقيم 𝒃 ‪ 𝒚 = 𝒂𝒙 +‬مستقيم مقارب مائل ⇒ ‪3) lim 𝑓(𝑥) − (𝑎𝑥 + 𝑏) = 0‬‬
‫∞→𝑥‬

‫‪02‬‬
‫مالحظة ‪:‬‬
‫‪ )1‬إذا كانت )𝑥(𝜑 ‪ 𝑓(𝑥) = 𝑎𝑥 + 𝑏 +‬و‪ّ ، lim 𝜑(𝑥) = 0‬‬
‫فإن المستقيم 𝑏 ‪ 𝑦 = 𝑎𝑥 +‬مستقيم مقارب مائل للمنحنى ) ‪(Cf‬‬
‫∞→𝑥‬
‫‪ )2‬قد يكون للمنحنى ) ‪ (Cf‬مستقيمين مقاربين أحدهما بجوار )∞‪ (−‬واآلخر بجوار )∞‪(+‬‬
‫𝟑‬
‫مثال ‪𝒇(𝒙) = (𝒙−𝟒)𝟐 :1‬‬
‫‪3‬‬
‫‪lim 𝑓(𝑥) = lim‬‬ ‫‪=0‬‬
‫∞‪|𝑥|→+‬‬ ‫‪|𝑥|→+∞ (𝑥 − 4)2‬‬
‫‪3‬‬ ‫‪3‬‬
‫‪lim‬‬ ‫)𝑥(𝑓‬ ‫=‬ ‫‪lim‬‬ ‫‪2‬‬
‫=‬ ‫∞‪+‬‬ ‫;‬ ‫‪lim‬‬ ‫)𝑥(𝑓‬ ‫=‬ ‫‪lim‬‬ ‫‪2‬‬
‫∞‪= +‬‬
‫)‪𝑥→4 (𝑥 − 4‬‬ ‫)‪𝑥→4 (𝑥 − 4‬‬
‫<‬ ‫<‬ ‫>‬ ‫>‬
‫‪𝑥→4‬‬ ‫‪𝑥→4‬‬
‫يقبل المنحنى )‪ (Cf‬مستقيما مقاربا عموديا معادلته ‪ 𝑥 = 4 :‬وآخر أفقيا معادلته ‪𝑦 = 0 :‬‬

‫)‪(Cf‬‬

‫‪𝑥=4‬‬

‫‪𝑦=0‬‬

‫𝟑‬
‫مثال ‪𝒇(𝒙) = 𝒙 − 𝟐 + 𝒙−𝟏 :2‬‬
‫‪3‬‬
‫‪lim‬‬ ‫‪𝑓(𝑥) = −∞ ; lim‬‬ ‫‪𝑓(𝑥) = +∞ ; lim 𝑓(𝑥) − (𝑥 − 2) = lim‬‬ ‫‪=0‬‬
‫<‬ ‫>‬ ‫∞‪|𝑥|→+‬‬ ‫‪|𝑥|→+∞ 𝑥 − 1‬‬
‫‪𝑥→1‬‬ ‫‪𝑥→1‬‬
‫يقبل المنحنى )‪ (Cf‬مستقيما مقاربا عموديا ‪ (∆): 𝑥 = 1‬وآخر مائال ‪(∆′): 𝑦 = 𝑥 − 2‬‬
‫‪(∆): 𝑥 = 1‬‬

‫‪ -II‬االستمرارية واالشتقاقية‬
‫‪ .1‬االستمرارية‬
‫‪lim‬‬
‫<‬
‫‪𝑓(𝑥) = lim‬‬
‫>‬
‫تكون الدالة 𝑓 مستمرة عند ‪ 𝑥0‬إذا وفقط إذا ‪𝑓(𝑥) = 𝑓(𝑥0 ) :‬‬
‫‪𝑥→𝑥0‬‬ ‫‪𝑥→𝑥0‬‬
‫‪𝑥 2 +3‬‬
‫;‬ ‫‪𝑥>1‬‬
‫{ = )𝑥(𝑓 ‪𝑥0 = 1 ،‬‬ ‫‪𝑥+1‬‬ ‫مثال ‪:‬‬
‫‪+3; 𝑥 ≤ 1‬‬ ‫‪√𝑥 2‬‬
‫‪2‬‬
‫𝑥‬ ‫‪+‬‬ ‫‪3‬‬
‫‪lim‬‬ ‫‪𝑓(𝑥) = lim‬‬ ‫‪√𝑥 2 + 3 = 2 ; lim 𝑓(𝑥) = lim‬‬ ‫‪= 2 ; 𝑓(1) = √1 + 3 = 2‬‬
‫‪𝑥→1 𝑥 + 1‬‬
‫<‬ ‫<‬ ‫>‬ ‫>‬
‫‪𝑥→1‬‬ ‫‪𝑥→1‬‬ ‫‪𝑥→1‬‬

‫‪lim‬‬
‫<‬
‫‪𝑓(𝑥) = lim‬‬
‫>‬
‫الدالة 𝑓 مستمرة عند ‪𝑓(𝑥) = 𝑓(1) ⇒ 𝑥0 = 1‬‬
‫‪𝑥→1‬‬ ‫‪𝑥→1‬‬

‫‪01‬‬
‫‪ .2‬االشتقاقية‬
‫تكون الدالة 𝑓 قابلة لالشتقاق عند ‪ 𝑥0‬إذا وفقط إذا ‪:‬‬
‫) ‪𝑓(𝑥) − 𝑓(𝑥0‬‬ ‫) ‪𝑓(𝑥) − 𝑓(𝑥0‬‬
‫‪lim‬‬ ‫‪= lim‬‬ ‫)∞ ≠ 𝑙( 𝑙 =‬
‫<‬
‫‪𝑥→𝑥0‬‬ ‫‪𝑥 − 𝑥0‬‬ ‫>‬
‫‪𝑥 →𝑥0‬‬ ‫‪𝑥 − 𝑥0‬‬
‫‪ .3‬التفسير الهندسي‬
‫) ‪𝑓(𝑥)−𝑓(𝑥0‬‬ ‫) ‪𝑓(𝑥)−𝑓(𝑥0‬‬
‫‪ : lim‬يقبل المنحنى )‪ (Cf‬عند النقطة ذات الفاصلة ‪ 𝑥0‬مماسا‬ ‫‪= lim‬‬ ‫‪= 𝑓′(𝑥0 ) ‬‬
‫<‬ ‫𝑥‪𝑥−‬‬ ‫‪0‬‬ ‫>‬ ‫𝑥‪𝑥−‬‬ ‫‪0‬‬
‫‪𝑥→𝑥0‬‬ ‫‪𝑥→𝑥0‬‬
‫𝒙( ‪′‬‬
‫معادلته ‪𝒚 = 𝒇 𝟎 )(𝒙 − 𝒙𝟎 ) + 𝒇(𝒙𝟎 ) :‬‬
‫) ‪𝑓(𝑥)−𝑓(𝑥0‬‬ ‫) ‪𝑓(𝑥)−𝑓(𝑥0‬‬
‫‪ : lim‬يقبل المنحنى )‪ (Cf‬عند النقطة ذات الفاصلة ‪ 𝑥0‬نصفي مماسين‬ ‫‪= 𝑙; lim‬‬ ‫‪= 𝑙′‬‬ ‫‪‬‬
‫<‬ ‫𝑥‪𝑥−‬‬ ‫‪0‬‬ ‫>‬ ‫𝑥‪𝑥−‬‬ ‫‪0‬‬
‫‪𝑥→𝑥0‬‬ ‫‪𝑥→𝑥0‬‬
‫ميليهما 𝑙 و ‪ 𝑙′‬على الترتيب‬
‫‪ : 𝑓 ′ (𝑥0 ) = 0‬يقبل المنحنى )‪ (Cf‬عند النقطة ذات الفاصلة ‪ 𝑥0‬مماسا أفقيا معادلته ‪𝒚 = 𝒇(𝒙𝟎 ) :‬‬ ‫‪‬‬
‫) ‪𝑓(𝑥)−𝑓(𝑥0‬‬
‫‪ : lim‬يقبل المنحنى )‪ (Cf‬عند النقطة ذات الفاصلة ‪ 𝑥0‬نصف مماس عمودي (يوازي محور‬ ‫∞=‬ ‫‪‬‬
‫‪𝑥→𝑥0‬‬ ‫‪𝑥−𝑥0‬‬
‫التراتيب)‬
‫‪ : 𝑓 ′′ (𝑥0 ) = 0‬يقبل المنحنى )‪ (Cf‬نقطة انعطاف )) 𝟎𝒙(𝒇 ; 𝟎𝒙(𝝎 وتتغير وضعية )‪ (Cf‬بالنسبة للمماس عند‬ ‫‪‬‬
‫هذه النقطة‪.‬‬

‫مثال ‪(𝑇): 𝑦 = 2𝑥 − 1 ، 𝑓 ′ (𝑥0 ) = 2 ، 𝑥0 = 1 ، 𝑓(𝑥) = 𝑥 2 :1‬‬

‫‪𝑥 2 +3‬‬
‫;‬ ‫‪𝑥>1‬‬
‫{ = )𝑥(𝑓 ‪𝑥0 = 1 ،‬‬ ‫‪𝑥+1‬‬ ‫مثال ‪:2‬‬
‫‪+3; 𝑥 ≤ 1‬‬ ‫‪√𝑥 2‬‬
‫𝟏 )‪𝑓(𝑥) − 𝑓(1‬‬ ‫)‪𝑓(𝑥) − 𝑓(1‬‬
‫‪lim‬‬ ‫‪= ; lim‬‬ ‫𝟎=‬
‫<‬
‫‪𝑥→1‬‬ ‫‪𝑥−1‬‬ ‫‪𝟐 𝑥→1‬‬‫>‬ ‫‪𝑥−1‬‬

‫التفسير البياني للنتيجة ‪ :‬يقبل المنحنى )‪ (Cf‬نصفي مماسين عند النقطة ذات الفاصلة ‪1‬‬

‫‪1‬‬ ‫‪1‬‬
‫مثال ‪، 𝑓 ′ (𝑥) = 𝑥 2 − 𝑥 − 2 ، 𝑓(𝑥) = 3 𝑥 3 − 2 𝑥 2 − 2𝑥 + 1 :3‬‬
‫‪7‬‬ ‫‪13‬‬
‫= 𝑦 ‪(𝑇2 ): 𝑦 = − ، (𝑇−1 ):‬‬ ‫‪، 𝑓 ′ (2) = 0 ، 𝑓 ′ (−1) = 0‬‬
‫‪3‬‬ ‫‪6‬‬

‫‪5‬‬
‫مثال ‪𝐷𝑔 = [−1; +∞[ ، 𝑔(𝑥) = 𝑥 + 2 − 2 √𝑥 + 1 :4‬‬
‫‪5‬‬
‫‪𝑔(𝑥) − 1‬‬ ‫‪𝑥 + 1 − 2 √𝑥 + 1‬‬ ‫‪5‬‬
‫‪lim‬‬ ‫=‬ ‫‪lim‬‬ ‫=‬ ‫‪lim‬‬ ‫‪1‬‬ ‫‪−‬‬ ‫∞‪= −‬‬
‫‪𝑥→−1 𝑥 + 1‬‬ ‫‪𝑥+1‬‬
‫>‬ ‫>‬ ‫>‬
‫‪𝑥 →−1‬‬ ‫‪𝑥→−1‬‬ ‫‪2√𝑥 + 1‬‬

‫تفسير النتيجة بيانيا ‪ :‬الدالة 𝑔 غير قابلة لالشتقاق عند النقطة ذات الفاصلة (‪ )-1‬ويقبل‬
‫المنحنى )‪ (Cg‬نصف مماس عمودي‪.‬‬

‫‪01‬‬
‫‪1‬‬ ‫‪1‬‬ ‫‪1‬‬
‫‪، 𝑓 ′′ (2) = 0 ، 𝑓 ′′ (𝑥) = 2𝑥 − 1‬‬ ‫مثال ‪، 𝑓 ′ (𝑥) = 𝑥 2 − 𝑥 − 2 ، 𝑓(𝑥) = 3 𝑥 3 − 2 𝑥 2 − 2𝑥 + 1 :5‬‬
‫‪1‬‬ ‫‪1‬‬
‫)‪ 𝜔 (2 ; − 12‬نقطة انعطاف‪.‬‬

‫‪ .4‬مبرهنة القيم المتوسطة‬


‫أ‪ .‬لبيان أ ّن المعادلة 𝒄 = )𝒙(𝒈 تقبل حال وحيدا 𝜶 في المجال [𝒃 ;𝒂]‬
‫نبيّن أ ّن الدالة 𝑔 مستمرة ورتيبة على المجال [𝑏 ;𝑎] وأ ّن )𝑏(𝑔 < 𝑐 < )𝑎(𝑔 إذا كانت 𝑔 متزايدة‬
‫أو )𝑎(𝑔 < 𝑐 < )𝑏(𝑔 إذا كانت 𝑔 متناقصة‪.‬‬
‫إذا كانت المعادلة ‪ ، 𝑔(𝑥) = 0‬يكفي أن نبيّن أ ّن الدالة 𝑔 مستمرة ورتيبة على المجال [𝑏 ;𝑎] ّ‬
‫وأن ‪:‬‬
‫‪.𝑔(𝑎) × 𝑔(𝑏) < 0‬‬
‫ب‪ .‬لتعيين حصر للحل 𝜶‬
‫𝑏‪𝑎+‬‬
‫نقوم بتقسيم المجال [𝑏 ;𝑎] من خالل حساب ) ‪ 𝑔 ( 2‬فنأخذ نصف المجال الذي يشمل 𝛼 ونلغي النصف الثاني ‪،‬‬
‫ث ّم نواصل العملية حتى نصل إلى الحصر المطلوب‪.‬‬
‫(انظر الملحق الخاص بكيفية استعمال الحاسبة لحصر الحل 𝛼 في نهاية الكتاب)‬
‫ج‪ .‬لكتابة )𝜶(𝒇 بداللة 𝜶 وتعيين حصر للعدد )𝜶(𝒇‬
‫ننطلق دائما من المعادلة ‪ 𝑔(𝛼) = 0‬لكتابة ‪ 𝛼 3‬أو 𝛼 𝑒 أو 𝛼 ‪ ln‬بداللة 𝛼 ‪ ،‬ث ّم ّ‬
‫نعوض العبارة المتحصل عليها في الدالة 𝑓‪.‬‬
‫‪𝑥 3 +2𝑥 2‬‬
‫مثال‪𝑓(𝑥) = 𝑥 2 −1 ، 𝑔(𝑥) = 𝑥 3 − 3𝑥 − 4 :‬‬
‫‪ .1‬بيان أ ّن المعادلة 𝟎 = )𝒙(𝒈 تقبل حال وحيدا ‪ ، α‬حيث ‪𝛂 ∈ 𝟐; 𝟑‬‬
‫‪ . 𝑔(3) = 14 ، 𝑔(2) = −2‬الدالة 𝑔 مستمرة ورتيبة على المجال ‪ 𝑔(2) × 𝑔(3) < 0 ، 2; 3‬منه المعادلة‬
‫‪ 𝑔(𝑥) = 0‬تقبل حال وحيدا ‪ α‬على المجال‪2; 3‬‬
‫‪ .2‬تعيين حصر للعدد ‪ α‬بالتقريب إلى 𝟏‪𝟏𝟎−‬‬
‫𝟐‪𝒈(𝟐) = −‬‬ ‫𝟐‪𝒈(𝟐) = −‬‬ ‫‪𝑔(2) = −2‬‬
‫{‬‫‪𝒈(𝟐,‬‬ ‫)𝟓‬ ‫≈‬ ‫‪𝟒,‬‬ ‫𝟑𝟏‬ ‫{→‬ ‫‪𝒈(𝟐,‬‬ ‫)𝟓𝟐‬ ‫≈‬ ‫‪𝟎,‬‬ ‫𝟒𝟔‬ ‫{→‬ ‫‪𝒈(𝟐,‬‬ ‫𝟒𝟎 ‪𝟏) ≈ −𝟏,‬‬
‫‪𝑔(3) = 14‬‬ ‫‪𝑔(2,5) ≈ 4,13‬‬ ‫𝟓𝟎 ‪𝒈(𝟐, 𝟐) ≈ 𝟎,‬‬
‫أن ‪2,1 < 𝛼 < 2,2 :‬‬ ‫من النتائج السابقة ‪ ،‬نستنتج ّ‬
‫𝟒‪𝟑𝜶+‬‬
‫= )𝜶(𝒇‬ ‫‪ .3‬بيان أنّ ‪:‬‬
‫𝟐‬
‫‪3‬‬ ‫‪3‬‬
‫‪𝑔(𝛼) = 𝛼 − 3𝛼 − 4 = 0 ⇒ 𝛼 = 3𝛼 + 4‬‬
‫‪3‬‬ ‫‪2‬‬ ‫‪2‬‬
‫‪𝛼 3 + 2𝛼 2 𝛼 (1 + 𝛼 ) (3𝛼 + 4) (1 + 𝛼) 3𝛼 2 + 10𝛼 + 8‬‬
‫‪𝑓(𝛼) = 2‬‬ ‫=‬ ‫=‬ ‫=‬
‫‪𝛼 −1‬‬ ‫‪𝛼3‬‬ ‫‪3𝛼 + 4‬‬ ‫‪2𝛼 + 4‬‬
‫‪−‬‬ ‫‪1‬‬ ‫‪−1‬‬
‫𝛼‬ ‫𝛼‬
‫)‪(𝛼 + 2)(3𝛼 + 4‬‬ ‫‪3𝛼 + 4‬‬
‫= )𝛼(𝑓‬ ‫=‬
‫)‪2(𝛼 + 2‬‬ ‫‪2‬‬
‫‪ .4‬تعيين حصر لـ )𝜶(𝒇‬
‫‪10,3 3𝛼 + 4 10,6‬‬
‫⇒ ‪2,1 < 𝛼 < 2,2 ⇒ 6,3 < 3𝛼 < 6,6 ⇒ 10,3 < 3𝛼 + 4 < 10,6‬‬ ‫<‬ ‫<‬
‫‪2‬‬ ‫‪2‬‬ ‫‪2‬‬
‫‪⇒ 5,15 < 𝑓(𝛼) < 5,3‬‬

‫‪01‬‬
‫‪ -III‬الشفعية والتناظر‬
‫‪ ‬لبيان أ ّن الدالة 𝑓 زوجية نتحقّق أوال أ ّن 𝑓𝐷 متناظر بالنسبة إلى ‪ ، 1‬ث ّم نبيّن أنّه من أجل كل 𝑓𝐷 ∈ 𝑥 فإ ّن ‪𝑓(−𝑥) = :‬‬
‫)𝑥(𝑓‪ .‬في هذه الحالة يقبل المنحنى )‪ (Cf‬محور تناظر‪.‬‬
‫‪ ‬لبيان أ ّن الدالة 𝑓 فردية نتحقّق أوال أ ّن 𝑓𝐷 متناظر بالنسبة إلى ‪ ، 1‬ث ّم نبيّن أنّه من أجل كل 𝑓𝐷 ∈ 𝑥 فإ ّن ‪𝑓(−𝑥) = :‬‬
‫)𝑥(𝑓‪ .−‬في هذه الحالة يقبل المنحنى )‪ (Cf‬مركز تناظر‪.‬‬
‫‪ .1‬لبيان أ ّن المستقيم ذي المعادلة 𝒂 = 𝒙 محور تناظر للمنحنى )‪(Cf‬‬
‫أن ‪.𝑓(2𝑎 − 𝑥) = 𝑓(𝑥) :‬‬ ‫نتحقّق أوال أنّه من أجل كل 𝑓𝐷 ∈ 𝑥 فإ ّن 𝑓𝐷 ∈ )𝑥 ‪ ، (2𝑎 −‬ث ّم نبيّن ّ‬
‫‪ .2‬لبيان أ ّن النقطة )𝒃 ;𝒂(𝝎 مركز تناظر للمنحنى )‪(Cf‬‬
‫ّ‬
‫‪ ،‬ث ّم نبيّن أن ‪.𝑓(2𝑎 − 𝑥) + 𝑓(𝑥) = 2𝑏 :‬‬ ‫𝑎‪(2‬‬ ‫نتحقّق أوال أنّه من أجل كل 𝑓𝐷 ∈ 𝑥 فإ ّن 𝑓𝐷 ∈ )𝑥 ‪−‬‬
‫مثال ‪𝐷𝑔 = ℝ ، 𝑔(𝑥) = 𝑥 2 − 2 :1‬‬
‫𝑔𝐷 متناظر بالنسبة إلى ‪ ، 1‬ومن أجل كل ‪ 𝑥 ∈ ℝ‬فإ ّن ‪:‬‬
‫)𝑥(𝑔 = ‪ ، 𝑔(−𝑥) = (−𝑥)2 − 2 = 𝑥 2 − 2‬منه الدالة 𝑔 زوجية‬

‫𝑥‪3‬‬
‫مثال ‪𝐷𝑓 = ℝ − {−1; 1} ، 𝑓(𝑥) = 𝑥 2 −1 :2‬‬
‫𝑓𝐷 متناظر بالنسبة إلى ‪ ، 1‬ومن أجل كل 𝑓𝐷 ∈ 𝑥 فإ ّن ‪:‬‬
‫)𝑥‪3(−‬‬ ‫𝑥‪3‬‬
‫)𝑥(𝑓‪ ، 𝑓(−𝑥) = (−𝑥)2 −1 = − 𝑥 2 −1 = −‬منه الدالة 𝑓 فردية‬

‫‪2𝑥 2 −8𝑥+7‬‬
‫مثال ‪𝐷𝑓 = ℝ − {1; 3} ، 𝑓(𝑥) = 𝑥 2 −4𝑥+3 :3‬‬
‫بيان أ ّن المستقيم ‪ (∆): 𝑥 = 2‬محور تناظر لـ ) ‪(Cf‬‬
‫𝑓𝐷 ∈ )𝑥 ‪(4 −‬‬
‫{‬ ‫يكون المستقيم ‪ (∆): 𝑥 = 2‬محور تناظر لـ ) ‪ (Cf‬إذا وفقط إذا ‪:‬‬
‫)𝑥(𝑓 = )𝑥 ‪𝑓(4 −‬‬
‫𝑓𝐷 ∈ 𝑥 ‪𝑥 ∈ 𝐷𝑓 ⇒ 𝑥 ∉ {1; 3} ⇒ −𝑥 ∉ {−1; −3} ⇒ 4 − 𝑥 ∉ {3; 1} ⇒ 4 −‬‬
‫‪2(4 − 𝑥)2 − 8(4 − 𝑥) + 7‬‬
‫= )𝑥 ‪𝑓(4 −‬‬
‫‪(4 − 𝑥)2 − 4(4 − 𝑥) + 3‬‬
‫‪2𝑥 2 − 16𝑥 + 32 − 32 + 8𝑥 + 7 2𝑥 2 − 8𝑥 + 7‬‬
‫‪= 2‬‬ ‫‪= 2‬‬ ‫)𝑥(𝑓 =‬
‫‪𝑥 − 8𝑥 + 16 − 16 + 4𝑥 + 3‬‬ ‫‪𝑥 − 4𝑥 + 3‬‬
‫‪𝑥 2 +4𝑥+2‬‬
‫مثال ‪𝐷𝑓 = ℝ − {1} ، 𝑓(𝑥) = 𝑥−1 :4‬‬
‫بيان أ ّن النقطة )‪ 𝜔(1; 6‬مركز تناظر لـ ) ‪(Cf‬‬
‫𝑓𝐷 ∈ )𝑥 ‪(2 −‬‬
‫{‬ ‫تكون )‪ 𝜔(1; 6‬مركز تناظر لـ ) ‪ (Cf‬إذا وفقط إذا ‪:‬‬
‫‪𝑓(2 − 𝑥) + 𝑓(𝑥) = 12‬‬
‫𝑓𝐷 ∈ 𝑥 ‪𝑥 ∈ 𝐷𝑓 ⇒ 𝑥 ≠ 1 ⇒ −𝑥 ≠ −1 ⇒ 2 − 𝑥 ≠ 1 ⇒ 2 −‬‬

‫‪01‬‬
‫‪(2 − 𝑥)2 + 4(2 − 𝑥) + 2 𝑥 2 + 4𝑥 + 2‬‬
‫= )𝑥(𝑓 ‪𝑓(2 − 𝑥) +‬‬ ‫‪+‬‬
‫‪2−𝑥−1‬‬ ‫‪𝑥−1‬‬
‫‪𝑥 2 − 4𝑥 + 4 + 8 − 4𝑥 + 2 𝑥 2 + 4𝑥 + 2‬‬
‫=‬ ‫‪+‬‬
‫𝑥‪1−‬‬ ‫‪𝑥−1‬‬
‫)‪−𝑥 2 + 8𝑥 − 14 + 𝑥 2 + 4𝑥 + 2 12𝑥 − 12 12(𝑥 − 1‬‬
‫=‬ ‫=‬ ‫=‬ ‫‪= 12‬‬
‫‪𝑥−1‬‬ ‫‪𝑥−1‬‬ ‫‪𝑥−1‬‬

‫‪ -IV‬استعمال التمثيل البياني وجدول التغيّرات‬


‫‪ .1‬القراءة البيانية‬
‫مثال‪:‬‬
‫من المنحنى المقابل يمكننا استنتاج ما يلي‪:‬‬

‫‪ .1‬تعيين ‪𝑓 ′′ (2) ; (𝑓 ∘ 𝑓)′ (2) ; 𝑓 ′ (3) ; 𝑓 ′ (2) ; 𝑓(2) :‬‬


‫)نقطة انعطاف( ‪ ); 𝑓"(2) = 0‬مماس أفقي( ‪𝑓(2) = 0 ; 𝑓 ′ (2) = −2 ; 𝑓 ′ (3) = 0‬‬
‫‪(𝑓 ∘ 𝑓)′ (2) = (𝑓 ′ ∘ 𝑓)(2) × 𝑓 ′ (2) = 𝑓 ′ (0) × 𝑓 ′ (2) = 0(−2) = 0‬‬
‫‪ .2‬حل بيانيا المتراجحتين‪𝑓′(𝑥) ≥ 0 ، 𝑓(𝑥) < 0 :‬‬
‫أ) ‪𝑆 = [−3; −2[ ∪ ]2; 4[ : 𝑓(𝑥) < 0‬‬
‫ب) ‪𝑆 = [−3; 0] ∪ [3; 5] : 𝑓′(𝑥) ≥ 0‬‬
‫‪ .3‬كتابة معادلة المماس )𝑇( للمنحنى عند النقطة ذات الفاصلة ‪2‬‬
‫‪𝑦 = 𝑓 ′ (2)(𝑥 − 2) + 𝑓(2) = −2(𝑥 − 2) ⇒ (𝑇): 𝑦 = −2𝑥 + 4‬‬
‫‪ .4‬جدول تغيرات الدالة ‪f‬‬

‫‪ .5‬بيان ّ‬
‫أن المعادلة ‪ 𝑓(𝑥) = 2‬تقبل حال وحيدا في المجال ]‪[3; 5‬‬
‫الدالة ‪ f‬مستمرة ورتيبة على المجال ]‪ [3; 5‬و )‪ ، 𝑓(3) < 2 < 𝑓(5‬منه المعادلة ‪ 𝑓(𝑥) = 2‬تقبل حال وحيدا في‬
‫المجال ]‪ [3; 5‬حسب مبرهنة القيم المتوسطة‪.‬‬
‫المعرفة بـ‪𝑔(𝑥) = 2√𝑓(𝑥) :‬‬
‫ّ‬ ‫‪ .6‬جدول تغيرات الدالة 𝑔‬
‫)𝑥(‪𝑓′‬‬
‫= )𝑥( ‪𝑔(𝑥) = 2√𝑓(𝑥) ⇒ 𝐷𝑔 = [−2; 2] ∪ [4; 5] ; 𝑔′‬‬
‫)𝑥(𝑓√‬
‫)‪′ (0‬‬
‫𝑓‬ ‫‪0‬‬
‫= )‪𝑔(−2) = 2√𝑓(−2) = 0 ; 𝑔′ (0‬‬ ‫=‬ ‫; ‪= 0 ; 𝑔(0) = 2√𝑓(0) = 2√1,5 ≈ 2,45‬‬
‫‪√𝑓 (0) √1,5‬‬
‫‪𝑔(2) = 2√𝑓(2) = 0 ; 𝑔(4) = 2√𝑓(4) = 0 ; 𝑔(5) = 2√𝑓(5) = 2√3 ≈ 3,46‬‬

‫‪01‬‬
‫‪ .2‬استنتاج عبارة دالة من خالل تمثيلها البياني أو جدول تغيّراتها‬
‫𝑐‬
‫‪𝑔(𝑥) = 𝑎𝑥 + 𝑏 +‬‬ ‫مثال ‪:1‬‬
‫‪𝑥+1‬‬
‫𝑐‬
‫‪𝑔′ (𝑥) = 𝑎 −‬‬
‫‪(𝑥 + 1)2‬‬
‫‪𝑔(0) = 3‬‬ ‫‪𝑏+𝑐 =3‬‬
‫𝑐‬
‫‪{ 𝑔′ (1) = 0 ⇒ { 𝑎 − = 0‬‬
‫‪4‬‬
‫‪𝑔′ (0) = −3‬‬ ‫‪𝑎 − 𝑐 = −3‬‬
‫‪𝑏 = −𝑐 + 3‬‬
‫𝑐‪3‬‬ ‫‪𝑏 = −1‬‬ ‫‪4‬‬
‫{⇒‬ ‫‪= 3 ⇒ { 𝑐 = 4 ⇒ 𝑔(𝑥) = 𝑥 − 1 +‬‬
‫‪4‬‬ ‫‪𝑥+1‬‬
‫‪𝑎=1‬‬
‫‪𝑎 =𝑐−3‬‬
‫𝑐‬ ‫𝑐‬
‫مثال ‪𝑓 ′ (𝑥) = 𝑎 − (𝑥+𝑑)2 ، 𝑓(𝑥) = 𝑎𝑥 + 𝑏 + 𝑥+𝑑 :2‬‬

‫𝟏 = 𝒅 ⇒ ‪𝐷𝑓 = ℝ − {−1} ⇒ −1 + 𝑑 = 0‬‬


‫𝑐‬ ‫𝑐‬
‫‪𝑓′(1) = 0‬‬ ‫‪𝑎− =0‬‬ ‫=𝑎‬
‫‪4‬‬ ‫‪4‬‬ ‫𝟏=𝒂‬ ‫‪4‬‬
‫⇒ ‪{ 𝑓(0) = 0‬‬ ‫‪𝑏+𝑐 =0‬‬ ‫‪⇒ 𝑏 = −𝑐 ⇒ {𝒃 = −𝟒 ⇒ 𝑓(𝑥) = 𝑥 − 4 +‬‬
‫𝑐‬ ‫𝑐‬ ‫‪𝑥+1‬‬
‫‪𝑓(1) = −1‬‬ ‫‪𝑎 + 𝑏 + = −1‬‬ ‫‪− = −1‬‬ ‫𝟒=𝒄‬
‫{‬ ‫‪2‬‬ ‫‪{ 4‬‬
‫‪ .3‬المناقشة البيانية‬
‫لمناقشة بيانيا حسب قيم الوسيط الحقيقي 𝑚 عدد وإشارة حلول معادلة ما ال بد ّ من استخراج عبارة الدالة ال ُممثّلة بيانيا من المعادلة‬
‫‪ ،‬وتكون المعادلة النهائية على أحد األشكال التالية‪:‬‬
‫‪ : 𝑚 = 𝑓(𝑥) ‬في هذه الحالة ندرس تقاطع المنحنى ) ‪ (Cf‬مع المستقيم المتحرك الموازي لمحور الفواصل‪.‬‬
‫‪ : 𝑎𝑥 + 𝑚 = 𝑓(𝑥) ‬في هذه الحالة ندرس تقاطع ) ‪ (Cf‬مع المستقيم المتحرك الموازي للمستقيم الذي ميله 𝑎 (غالبا ما‬
‫يكون مستقيما مقاربا مائال أو مماسا)‪.‬‬
‫مع المستقيم المتحرك الموازي‬ ‫‪ 𝑚2 = 𝑓(𝑥) ‬أو )𝑥(𝑓 = |𝑚| ‪ :‬في هذه الحالة ندرس تقاطع المنحنى ) ‪(Cf‬‬
‫لمحور الفواصل مع مراعاة أن تكون بداية الدراسة من محور الفواصل وليس من األسفل كما هو الحال في المناقشتين‬
‫السابقتين‪.‬‬
‫‪ : 𝑚𝑥 = 𝑓(𝑥) ‬في هذه الحالة ندرس تقاطع المنحنى ) ‪ (Cf‬مع المستقيم الذي يدور حول المبدأ‪.‬‬

‫مالحظات ‪:‬‬
‫‪ .0‬تُحد ّد قيم 𝑚 من خالل تقاطع المستقيم المتحرك مع المنحنى ) ‪(Cf‬‬
‫‪ .2‬يكون الحل موجبا إذا كانت نقطة التقاطع على يمين محور التراتيب ‪ ،‬ويكون سالبا إذا كانت نقطة التقاطع على يسار‬
‫محور التراتيب‪.‬‬
‫‪ .1‬إذا كان المستقيم المتحرك مماسا للمنحنى ) ‪ (Cf‬يكون الحل مضاعفا‪.‬‬

‫‪‬‬
‫‪‬‬

‫‪01‬‬
‫)‪ (Cf‬يقبل مستقيما مقاربا عموديا معادلته 𝒂 = 𝒙‬ ‫∞ = )𝒙(𝒇 𝐦𝐢𝐥‬
‫𝒂→𝒙‬

‫)‪ (Cf‬يقبل مستقيما مقاربا أفقيا معادلته 𝒃 = 𝒚‬ ‫𝒃 = )𝒙(𝒇 𝐦𝐢𝐥‬


‫∞‪𝒙→+‬‬

‫)‪ (Cf‬يقبل مستقيما مقاربا مائال معادلته 𝒃 ‪𝒚 = 𝒂𝒙 +‬‬ ‫𝟎 = ])𝒃 ‪𝐥𝐢𝐦 [𝒇(𝒙) − (𝒂𝒙 +‬‬
‫∞‪𝒙→+‬‬

‫)𝒂(𝒇 ‪𝒇(𝒙) −‬‬


‫)‪ (Cf‬يقبل مماسا أفقيا عند النقطة ))𝒂(𝒇 ;𝒂(‬ ‫𝐦𝐢𝐥‬ ‫𝟎=‬
‫𝒂→𝒙‬ ‫𝒂‪𝒙−‬‬
‫)𝒂(𝒇 ‪𝒇(𝒙) −‬‬
‫)‪ (Cf‬يقبل نصف مماس عمودي عند النقطة ))𝒂(𝒇 ;𝒂(‬ ‫𝐦𝐢𝐥‬ ‫∞=‬
‫𝒂→𝒙‬ ‫𝒂‪𝒙−‬‬

‫المنحنيان )‪ (Cf‬و )‪ (Cg‬متقاربان بجوار ∞‬ ‫𝟎 = ])𝒙(𝒈 ‪𝐥𝐢𝐦 [𝒇(𝒙) −‬‬


‫∞→𝒙‬

‫الدالة 𝒇 فردية و )‪ (Cf‬يقبل مركز تناظر (المبدأ)‬ ‫𝟎 = )𝒙(𝒇 ‪𝒇(−𝒙) +‬‬

‫الدالة 𝒇 زوجية و )‪ (Cf‬يقبل محور تناظر (التراتيب)‬ ‫𝟎 = )𝒙(𝒇 ‪𝒇(−𝒙) −‬‬

‫المستقيم ‪ 𝒙 = 𝒂 :‬محور تناظر لـ )‪(Cf‬‬ ‫)𝒙(𝒇 = )𝒙 ‪𝒇(𝟐𝒂 −‬‬

‫النقطة )𝒃 ;𝒂(𝝎 مركز تناظر لـ )‪(Cf‬‬ ‫𝒃𝟐 = )𝒙(𝒇 ‪𝒇(𝟐𝒂 − 𝒙) +‬‬

‫المنحنيان )‪ (Cf‬و )‪ (Cg‬متطابقان‬ ‫)𝒙(𝒇 = )𝒙(𝒈‬

‫)‪ (Cf‬و )‪ (Cg‬متناظران بالنسبة لمحور التراتيب‬ ‫)𝒙‪𝒈(𝒙) = 𝒇(−‬‬

‫)‪ (Cf‬و )‪ (Cg‬متناظران بالنسبة لمحور الفواصل‬ ‫)𝒙(𝒇‪𝒈(𝒙) = −‬‬

‫)‪ (Cf‬و )‪ (Cg‬متناظران بالنسبة للمبدأ‬ ‫)𝒙‪𝒈(𝒙) = −𝒇(−‬‬


‫𝒂‪−‬‬
‫( ⃗‬
‫𝒖‬ ‫)‪ (Cg‬صورة )‪ (Cf‬باالنسحاب الذي شعاعه )‬ ‫𝒃 ‪𝒈(𝒙) = 𝒇(𝒙 + 𝒂) +‬‬
‫𝒃‬
‫الدالة 𝒈 زوجية‬ ‫‪‬‬
‫من أجل 𝟎 > 𝒙 ‪𝒈(𝒙) = 𝒇(𝒙) :‬‬ ‫‪‬‬
‫)‪ (Cf‬و )‪ (Cg‬متطابقان‬ ‫)|𝒙|(𝒇 = )𝒙(𝒈‬
‫من أجل 𝟎 < 𝒙 ‪ :‬نكمل رسم )‪ (Cg‬بالتناظر‬ ‫‪‬‬
‫المحوري (بال نسبة لمحور التراتيب)‬
‫‪ ‬من أجل 𝟎 > )𝒙(𝒇 ‪𝒈(𝒙) = 𝒇(𝒙) :‬‬
‫)‪ (Cf‬و )‪ (Cg‬متطابقان‬
‫‪ ‬من أجل 𝟎 < )𝒙(𝒇 ‪𝒈(𝒙) = −𝒇(𝒙) :‬‬ ‫|)𝒙(𝒇| = )𝒙(𝒈‬
‫)‪ (Cf‬و )‪ (Cg‬متناظران بالنسبة لمحور الفواصل‬
‫(المنحنى )‪ (Cg‬دائما فوق محور الفواصل)‬

‫نح ّل المعادلة 𝟎 = )𝒙(𝒇‬ ‫تعيين تقاطع )‪ (Cf‬مع محور الفواصل‬

‫نحسب )𝟎(𝒇‬ ‫تعيين تقاطع )‪ (Cf‬مع محور التراتيب‬

‫) 𝟎𝒙(𝒇 ‪(𝑻): 𝒚 = 𝒇′ (𝒙𝟎 )(𝒙 − 𝒙𝟎 ) +‬‬ ‫كتابة معادلة المماس عند النقطة ذات الفاصلة 𝟎𝒙‬

‫نعيّن 𝟎𝒙 حيث 𝒂 = ) 𝟎𝒙( ‪ 𝒇′‬ث ّم نكتب معادلة المماس‬ ‫كتابة معادلة المماس الذي معامل توجيهه 𝒂‬

‫نعيّن 𝟎𝒙 حيث ) 𝟎𝒙(𝒇 ‪𝒃 = 𝒇′(𝒙𝟎 )(𝒂 − 𝒙𝟎 ) +‬‬


‫كتابة معادلة المماس الذي يشمل النقطة )𝒃 ;𝒂(‬
‫ث ّم نكتب معادلة المماس عند النقطة ذات الفاصلة 𝟎𝒙‬

‫‪09‬‬
‫‪ .1‬توجد دالة وحيدة 𝑓 قابلة لالشتقاق على ‪ ℝ‬تحقّق 𝑓 = ‪ 𝑓 ′‬و ‪ 𝑓(0) = 1‬نرمز إليها بالرمز 𝑥 𝑒 = )𝑥(𝑓 وتُسمى الدالة‬
‫األسّية‪.‬‬
‫‪ .2‬العدد 𝑒 هو صورة العدد ‪ 1‬بالدالة األسّية حيث 𝑒 = )‪)𝑒 ≈ 2,718( .𝑓(1‬‬
‫سية ‪:‬‬‫‪ .3‬خواص الدالة األ ّ‬
‫𝑥‬
‫‪1‬‬ ‫𝑒‬
‫𝑥𝑛 𝑒 = 𝑛) 𝑥 𝑒( ; 𝑦‪𝑒 0 = 1 ; 𝑒 −𝑥 = 𝑥 ; 𝑒 𝑥 . 𝑒 𝑦 = 𝑒 𝑥+𝑦 ; 𝑦 = 𝑒 𝑥−‬‬
‫𝑒‬ ‫𝑒‬
‫مثال ‪:‬‬
‫‪1‬‬ ‫‪𝑒 4𝑥 + 2𝑒 2𝑥 + 1‬‬
‫‪(𝑒 𝑥 + 𝑒 −𝑥 )2 = 𝑒 2𝑥 + 𝑒 −2𝑥 + 2𝑒 𝑥 . 𝑒 −𝑥 = 𝑒 2𝑥 +‬‬ ‫‪+‬‬ ‫‪2‬‬ ‫=‬
‫𝑥‪𝑒 2‬‬ ‫𝑥‪𝑒 2‬‬
‫سية ‪:‬‬
‫‪ .4‬نهايات الدالة األ ّ‬
‫𝑥‬
‫‪𝑒 −1‬‬
‫‪① lim 𝑒 𝑥 = 0 ; ② lim 𝑒 𝑥 = +∞ ; ③ lim‬‬ ‫‪=1‬‬
‫∞‪𝑥→−‬‬ ‫∞‪𝑥→+‬‬ ‫‪𝑥→0‬‬ ‫𝑥‬
‫𝑥𝑒‬
‫‪④ lim‬‬ ‫‪= +∞ ; ⑤ lim 𝑥𝑒 𝑥 = 0‬‬
‫𝑥 ∞‪𝑥→+‬‬ ‫∞‪𝑥→−‬‬
‫مثال ‪:‬‬
‫𝑥‪2‬‬ ‫𝑥‬ ‫𝑥‬ ‫𝑥‬
‫𝑒 ‪1) lim‬‬ ‫∞‪− 2𝑒 = lim 𝑒 (𝑒 − 2) = +‬‬
‫∞‪𝑥→+‬‬ ‫∞‪𝑥→+‬‬
‫𝑥‪2‬‬ ‫𝑥‬
‫𝑒 ‪2) lim‬‬ ‫‪+ 𝑒 − 4 = −4‬‬
‫∞‪𝑥→−‬‬
‫‪1‬‬ ‫‪1‬‬ ‫‪1‬‬
‫‪3) lim−(1 − 𝑥)𝑒 𝑥 + 2 = 2 ( lim−‬‬ ‫)‪= −∞ ; lim− 𝑒 𝑥 = 0‬‬
‫‪𝑥→0‬‬ ‫𝑥 ‪𝑥→0‬‬ ‫‪𝑥→0‬‬
‫𝑥‪2‬‬ ‫𝑋‬
‫‪𝑒 −1‬‬ ‫‪𝑒 −1‬‬
‫‪4) lim‬‬ ‫‪; 𝑋 = 2𝑥 ; lim‬‬ ‫‪= 1‬‬
‫‪𝑥→0‬‬ ‫𝑥‪2‬‬ ‫‪𝑋→0‬‬ ‫𝑋‬
‫‪𝑒 2𝑥+1‬‬ ‫‪𝑒 𝑥 𝑥+1‬‬
‫‪5) lim‬‬ ‫‪= lim‬‬ ‫𝑒‬ ‫∞‪= +‬‬
‫𝑥 ∞‪𝑥→+‬‬ ‫𝑥 ∞‪𝑥→+‬‬
‫‪6) lim (𝑥 2 + 2𝑥 + 2)𝑒 𝑥 = lim 𝑥 2 𝑒 𝑥 + 2𝑥𝑒 𝑥 + 2𝑒 𝑥 = 0‬‬
‫∞‪𝑥→−‬‬ ‫∞‪𝑥→−‬‬
‫سية ‪:‬‬
‫‪ .5‬اتجاه تغ ّير الدالة األ ّ‬
‫)𝑥(𝑢‬ ‫)𝑥(𝑢‬ ‫‪(𝑒 𝑥 )′‬‬ ‫𝑥‬
‫𝑒[‬ ‫𝑒)𝑥(‪]′ = 𝑢′‬‬ ‫‪،‬‬ ‫الدالة األسّية قابلة لالشتقاق على ‪= 𝑒 : ℝ‬‬
‫الدالة األسّية متزايدة تماما على ‪ ، ℝ‬منه نستنتج ّ‬
‫أن ‪:‬‬
‫‪ ‬المعادلة )𝑥(𝑣 𝑒 = )𝑥(𝑢 𝑒 تكافئ المعادلة )𝑥(𝑣 = )𝑥(𝑢‬
‫‪ ‬المتراجحة )𝑥(𝑣 𝑒 > )𝑥(𝑢 𝑒 تكافئ المتراجحة )𝑥(𝑣 > )𝑥(𝑢‬
‫‪ ‬المتراجحة )𝑥(𝑣 𝑒 < )𝑥(𝑢 𝑒 تكافئ المتراجحة )𝑥(𝑣 < )𝑥(𝑢‬

‫مثال ‪ :1‬حساب المشتقات‬


‫𝑥‬ ‫‪−𝑥+2‬‬ ‫)𝑥( ‪′‬‬ ‫𝑥‬ ‫‪−𝑥+2‬‬
‫𝑒‪1) 𝑓(𝑥) = 𝑒 + 3‬‬ ‫𝑓 ;𝑥‪−‬‬ ‫𝑒‪= 𝑒 − 3‬‬ ‫‪−1‬‬
‫𝑥‪2) 𝑓(𝑥) = 𝑒 −2𝑥 − 𝑒 −3𝑥 + 4 ; 𝑓 ′ (𝑥) = −2𝑒 −2𝑥 + 3𝑒 −3‬‬
‫𝑥𝑒‬ ‫)𝑥( ‪′‬‬
‫𝑥 𝑒 ‪𝑒 𝑥 (𝑒 2𝑥 − 3) − 2𝑒 2𝑥 .‬‬ ‫)‪𝑒 𝑥 (𝑒 2𝑥 + 3‬‬
‫𝑥‪3) 𝑓(𝑥) = 2‬‬ ‫𝑓;‬ ‫=‬ ‫𝑥‪= − 2‬‬
‫‪𝑒 −3‬‬ ‫‪(𝑒 2𝑥 − 3)2‬‬ ‫‪(𝑒 − 3)2‬‬
‫‪1 − 𝑒 −2𝑥 ′‬‬ ‫) 𝑥‪2𝑒 −2𝑥 (1 + 𝑒 −2𝑥 ) + 2𝑒 −2𝑥 (1 − 𝑒 −2‬‬ ‫𝑥‪4𝑒 −2‬‬
‫= )𝑥(𝑓 )‪4‬‬ ‫;‬ ‫𝑓‬ ‫)𝑥(‬ ‫=‬ ‫=‬
‫𝑥‪1 + 𝑒 −2‬‬ ‫‪(1 + 𝑒 −2𝑥 )2‬‬ ‫‪(1 + 𝑒 −2𝑥 )2‬‬

‫‪21‬‬
‫مثال ‪ :2‬حل المعادالت والمتراجحات‬
‫‪1‬‬
‫‪1) 𝑒 −5𝑥 = 𝑒 ⇒ −5𝑥 = 1 ⇒ 𝑥 = −‬‬
‫‪5‬‬
‫‪2) 𝑒 2𝑥+1 − (𝑒 𝑥 )3 = 0 ⇒ 𝑒 2𝑥+1 = 𝑒 3𝑥 ⇒ 2𝑥 + 1 = 3𝑥 ⇒ 𝑥 = 1‬‬
‫‪ 𝑋 = 5‬أو ‪3) 𝑒 2𝑥 − 𝑒 𝑥 − 20 = 0 ; 𝑋 = 𝑒 𝑥 ; 𝑋 2 − 𝑋 − 20 = 0 ⇒ 𝑋 = −4‬‬
‫)الحل ‪ − 4‬مرفوض ّ‬
‫ألن ‪𝑒 𝑥 = 5 ⇒ 𝑥 = ln 5 (𝑒 𝑥 > 0‬‬
‫‪2‬‬
‫‪4) 𝑒 2𝑥 ≤ 𝑒 5𝑥+3 ⇒ 2𝑥 2 ≤ 5𝑥 + 3 ⇒ 2𝑥 2 − 5𝑥 − 3 ≤ 0‬‬
‫‪1‬‬ ‫‪1‬‬
‫]‪∆= 49 ; 𝑥 ′ = − ; 𝑥" = 3 ; 𝑆 = [− ; 3‬‬
‫‪2‬‬ ‫‪2‬‬
‫‪ .6‬المعادالت التفاضلية ‪:‬‬
‫𝑒𝑐 = 𝑦 ‪𝑐 ∈ ℝ ،‬‬ ‫𝑥𝑎‬
‫‪ ‬حلول المعادلة التفاضلية 𝑦𝑎 = 𝑦 هي ‪:‬‬
‫‪′‬‬
‫𝑏‬
‫حلول المعادلة التفاضلية 𝑏 ‪ 𝑦 ′ = 𝑎𝑦 +‬هي ‪𝑐 ∈ ℝ ، 𝑦 = 𝑐𝑒 𝑎𝑥 − :‬‬ ‫‪‬‬
‫𝑎‬
‫مثال ‪:‬‬
‫‪′‬‬
‫‪ 𝑓 = 3𝑓 .1‬و ‪𝑓(0) = 1‬‬
‫𝑥‪𝑓 ′ = 3𝑓 ⇒ 𝑓(𝑥) = 𝑐𝑒 3𝑥 ; 𝑓(0) = 1 ⇒ 𝑐 = 1 ⇒ 𝑓(𝑥) = 𝑒 3‬‬
‫‪ 𝑓 ′ − 2𝑓 = 4 .2‬و ‪𝑓(1) = 0‬‬
‫‪4‬‬
‫‪𝑓 ′ − 2𝑓 = 4 ⇒ 𝑓 ′ = 2𝑓 + 4 ⇒ 𝑓(𝑥) = 𝑐𝑒 2𝑥 − 2 = 𝑐𝑒 2𝑥 − 2‬‬
‫‪2‬‬
‫)‪𝑓(1) = 0 ⇒ 𝑐𝑒 2 − 2 = 0 ⇒ 𝑐 = 2 = 2𝑒 −2 ⇒ 𝑓(𝑥) = 2𝑒 −2 𝑒 2𝑥 − 2 = 2(𝑒 2𝑥−2 − 1‬‬
‫𝑒‬
‫‪ 𝑓 ′ − 3𝑓 + 3 = 0 .3‬و ‪𝑓′(0) = 3‬‬
‫‪𝑓 ′ − 3𝑓 + 3 = 0 ⇒ 𝑓 ′ = 3𝑓 − 3 ⇒ 𝑓(𝑥) = 𝑐𝑒 3𝑥 + 1‬‬
‫‪𝑓 ′(0) = 3 ⇒ 3𝑐𝑒 3(0) = 3 ⇒ 3𝑐 = 3 ⇒ 𝑐 = 1 ⇒ 𝑓(𝑥) = 𝑒 3𝑥 + 1‬‬

‫‪‬‬
‫‪‬‬

‫‪20‬‬
‫‪ .1‬من أجل كل عدد حقيقي 𝑎 موجب تماما ‪ ،‬يوجد عدد حقيقي وحيد 𝑥 حيث ‪ 𝑒 𝑥 = 𝑎 :‬يُس ّمى العدد 𝑥 اللوغاريتم النيبيري‬
‫للعدد 𝑎 حيث ‪𝑥 = ln 𝑎 :‬‬
‫عرفة على المجال [∞‪]0; +‬‬
‫‪ .2‬الدالة اللوغاريتمية 𝑥 ‪ُ 𝑓(𝑥) = ln‬م ّ‬
‫‪ .3‬خواص الدالة اللوغاريتمية ‪:‬‬
‫𝑦‬ ‫𝑥 ‪ln‬‬ ‫𝑥‬
‫𝑒 ② ; 𝑒 = 𝑥 ⇒ 𝑦 = 𝑥 ‪① ln‬‬ ‫)‪= 𝑥 (𝑥 > 0) ; ③ ln 𝑒 = 𝑥 (𝑥 ∈ ℝ‬‬
‫𝑥‬
‫𝑥 ‪④ ln(𝑥𝑦) = ln 𝑥 + ln 𝑦 ; ⑤ ln ( ) = ln 𝑥 − ln 𝑦 ; ⑥ ln 𝑥 𝑛 = 𝑛 ln‬‬
‫𝑦‬
‫‪1‬‬ ‫‪1‬‬
‫𝑥 ‪⑦ ln ( ) = − ln 𝑥 ; ⑧ ln √𝑥 = ln‬‬
‫𝑥‬ ‫‪2‬‬
‫مثال ‪:‬‬
‫‪1‬‬ ‫‪1‬‬ ‫‪1‬‬ ‫‪3‬‬
‫= ‪1) ln√𝑒 − ln ( ) = ln 𝑒 + ln 𝑒 = + 1‬‬
‫𝑒‬ ‫‪2‬‬ ‫‪2‬‬ ‫‪2‬‬
‫‪3‬‬ ‫‪3‬‬
‫‪2) 𝑒 ln 3−ln 5+ln 15 = 𝑒 ln5+ln 15 = 𝑒 ln(5×15) = 𝑒 ln 9 = 9‬‬

‫‪ .4‬نهايات الدالة اللوغاريتمية ‪:‬‬


‫𝑥 ‪ln‬‬
‫‪① lim‬‬ ‫‪ln‬‬ ‫𝑥‬ ‫=‬ ‫∞‪−‬‬ ‫;‬ ‫②‬ ‫‪lim‬‬ ‫‪ln‬‬ ‫𝑥‬ ‫=‬ ‫∞‪+‬‬ ‫;‬ ‫③‬ ‫‪lim‬‬ ‫‪=0‬‬
‫>‬ ‫∞‪𝑥→+‬‬ ‫𝑥 ∞‪𝑥→+‬‬
‫‪𝑥→0‬‬
‫)𝑥 ‪ln(1 +‬‬ ‫𝑥 ‪ln‬‬
‫‪④ lim‬‬ ‫𝑥‬ ‫‪ln‬‬ ‫𝑥‬ ‫=‬ ‫‪0‬‬ ‫;‬ ‫⑤‬ ‫‪lim‬‬ ‫=‬ ‫‪1‬‬ ‫;‬ ‫⑥‬ ‫‪lim‬‬ ‫‪=1‬‬
‫>‬
‫‪𝑥→0‬‬
‫‪𝑥→0‬‬ ‫𝑥‬ ‫‪𝑥→1 𝑥 − 1‬‬

‫مثال ‪:‬‬
‫∞‪1) lim 𝑥 ln 𝑥 − 𝑥 = lim 𝑥(ln 𝑥 − 1) = +‬‬
‫∞‪𝑥→+‬‬ ‫∞‪𝑥→+‬‬
‫∞‪1+ln 𝑥 → −‬‬
‫‪2) lim‬‬ ‫∞‪= −‬‬
‫>‬
‫‪𝑥→0‬‬
‫‪𝑥2‬‬ ‫‪→ 0+‬‬
‫‪3‬‬ ‫‪3‬‬
‫)‪ln(𝑥 2 +3‬‬ ‫]) ‪ln[𝑥(𝑥+‬‬ ‫𝑥 ‪ln‬‬ ‫) ‪ln(𝑥+‬‬
‫𝑥‬ ‫𝑥‬
‫‪3) lim‬‬ ‫‪= lim‬‬ ‫⏟ ‪= lim‬‬ ‫𝑥⏟ ‪+‬‬ ‫‪= 0‬‬
‫∞‪𝑥→+‬‬ ‫𝑥‬ ‫∞‪𝑥→+‬‬ ‫𝑥‬ ‫𝑥 ∞‪𝑥→+‬‬
‫‪→0‬‬ ‫‪→0‬‬
‫‪2‬‬
‫‪4) lim‬‬
‫>‬
‫‪𝑥 ln 𝑥 − 𝑥 = lim‬‬
‫>‬
‫⏟‪2‬‬
‫‪𝑥 ln 𝑥 − 𝑥 = 0‬‬
‫‪𝑥→0‬‬ ‫‪𝑥 →0‬‬ ‫‪→0‬‬
‫‪1‬‬ ‫‪1‬‬ ‫)𝑋 ‪ln(1 +‬‬ ‫‪1‬‬
‫‪5) lim 𝑥 ln (1 + ) = lim‬‬ ‫‪ln(1‬‬ ‫‪+‬‬ ‫)𝑋‬ ‫=‬ ‫‪lim‬‬ ‫=‬ ‫‪1‬‬ ‫;‬ ‫𝑋(‬ ‫=‬ ‫)‬
‫∞‪𝑥→+‬‬ ‫𝑥‬ ‫𝑋 ‪𝑋→0‬‬
‫>‬ ‫>‬
‫‪𝑋→0‬‬ ‫𝑋‬ ‫𝑥‬
‫‪ .5‬اتجاه تغ ّير الدالة اللوغاريتمية ‪:‬‬
‫)𝑥( ‪𝑢′‬‬ ‫‪1‬‬
‫= ‪[ln 𝑢(𝑥)]′‬‬ ‫الدالة اللوغاريتمية قابلة لالشتقاق على [∞‪، (ln 𝑥)′ = 𝑥: ]0; +‬‬
‫)𝑥(𝑢‬
‫الدالة اللوغاريتمية متزايدة تماما على ‪ ، ℝ‬منه نستنتج ّ‬
‫أن ‪:‬‬
‫‪ ‬المعادلة )𝑥(𝑣 ‪ ln 𝑢(𝑥) = ln‬تكافئ المعادلة )𝑥(𝑣 = )𝑥(𝑢‬
‫‪ ‬المتراجحة )𝑥(𝑣 ‪ ln 𝑢(𝑥) > ln‬تكافئ المتراجحة )𝑥(𝑣 > )𝑥(𝑢‬
‫‪ ‬المتراجحة )𝑥(𝑣 ‪ ln 𝑢(𝑥) < ln‬تكافئ المتراجحة )𝑥(𝑣 < )𝑥(𝑢‬
‫مثال ‪ :1‬حساب المشتقات‬
‫‪2𝑥+4‬‬
‫‪1) 𝑓(𝑥) = ln(𝑥 2 + 4𝑥 − 5) ; 𝑓 ′ (𝑥) = 𝑥 2 +4𝑥−5‬‬

‫‪22‬‬
1
𝑥 1
2) 𝑓(𝑥) = 𝑥 ln 𝑥 − ln(ln 𝑥) ; 𝑓 ′ (𝑥)
= ln 𝑥 + − 𝑥 = ln 𝑥 + 1 −
𝑥 ln 𝑥 𝑥 ln 𝑥
4
2𝑥−1 (2𝑥+1)2 4 4𝑥 2 +3
3) 𝑓(𝑥) = 𝑥 + ln (2𝑥+1) ; 𝑓 ′ (𝑥) = 1 + 2𝑥−1 = 1 + (2𝑥+1)(2𝑥−1) = 4𝑥 2 −1
2𝑥+1
1
2 ln 𝑥 √ln 𝑥 2 ln 𝑥
4) 𝑓(𝑥) = √ln 𝑥 + (ln 𝑥)2 ; 𝑓 ′ (𝑥) = 𝑥 + = +
2√ln 𝑥 𝑥 2𝑥 ln 𝑥 𝑥

‫ حل المعادالت والمتراجحات‬:2 ‫مثال‬


1) ln(2𝑥 − 3) = ln(𝑥 − 3) + ln 5
3
2𝑥 − 3 > 0
{ ⇒ {𝑥 > 2 ⇒ 𝑫 = ]𝟑; +∞[
𝑥−3 >0
𝑥>3
ln(2𝑥 − 3) = ln(𝑥 − 3) + ln 5 ⇒ ln(2𝑥 − 3) = ln 5(𝑥 − 3)
⇒ 2𝑥 − 3 = 5𝑥 − 15 ⇒ 3𝑥 = 12 ⇒ 𝑥 = 4 ; 4 ∈ 𝐷 ⇒ 𝑆 = {4}
2) ln(𝑥 2 − 5) − ln(4 − 𝑥) = 2 ln 2
2 𝑥 ∈ ]−∞; −√5[ ∪ ]√5; +∞[ … 𝐷1
{𝑥 − 5 > 0 ⇒ { ⇒ 𝑥 ∈ 𝐷1 ∩ 𝐷2
4−𝑥 >0 𝑥 ∈ ]−∞; 4[ … 𝐷2
⇒ 𝑫 = ]−∞; −√𝟓[ ∪ ]√𝟓; 𝟒[
2
𝑥2 − 5 𝑥2 − 5
ln(𝑥 − 5) − ln(4 − 𝑥) = 2 ln 2 ⇒ ln = ln 4 ⇒ =4
4−𝑥 4−𝑥
⇒ 𝑥 2 − 5 = 16 − 4𝑥 ⇒ 𝑥 2 + 4𝑥 − 21 = 0 ⇒ (𝑥 − 3)(𝑥 + 7) = 0
−7 ∈ 𝐷
⇒ 𝑥 = 3 ‫ = 𝑥 أو‬−7; { ⇒ 𝑆 = {−7; 3}
3∈𝐷
3) (ln 𝑥)2 − ln 𝑥 − 6 = 0
𝑥 > 0 ⇒ 𝑫 = ]𝟎; +∞[ ; ln 𝑥 = 𝑋
(ln 𝑥)2 − ln 𝑥 − 6 = 0 ⇒ 𝑋 2 − 𝑋 − 6 = 0 ⇒ (𝑋 − 3)(𝑋 + 2) = 0
−2
⇒ 𝑋 = 3 ‫ = 𝑋 أو‬−2 ⇒ 𝑥 = 𝑒 3 ‫ 𝑒 = 𝑥 أو‬−2 ; {𝑒 3 ∈ 𝐷 ⇒ 𝑆 = {𝑒 −2 ; 𝑒 3 }
𝑒 ∈𝐷
𝟑
4) ln(2𝑥 + 3) < 4 ; 𝑫 = ]− 𝟐 ; +∞[
𝑒4 − 3
4
3 𝑒4 − 3
ln(2𝑥 + 3) < 4 ⇒ 2𝑥 + 3 < 𝑒 ⇒ 𝑥 < ⇒ 𝑆 = ]− ; [
2 2 2
5)(ln 𝑥)2 − 1 ≤ 0 ; 𝑫 = ]𝟎; +∞[
(ln 𝑥)2 − 1 ≤ 0 ⇒ (ln 𝑥 − 1)(ln 𝑥 + 1) ≤ 0 ⇒ −1 ≤ ln 𝑥 ≤ 1
⇒ 𝑒 −1 ≤ 𝑥 ≤ 𝑒 ⇒ 𝑆 = [𝑒 −1 ; 𝑒]




21
‫الجزء األول ‪:‬‬

‫‪ ABCDEFGH‬مكعب ضلعه ‪a‬‬

‫⃗⃗⃗⃗⃗ ‪،‬‬
‫⃗⃗⃗⃗⃗ ‪𝐴𝐵 .‬‬
‫⃗⃗⃗⃗⃗ ‪𝐶𝐻 ،‬‬
‫⃗⃗⃗⃗⃗ ‪𝐴𝐵 .‬‬
‫⃗⃗⃗⃗⃗ ‪𝐷𝐺 ،‬‬
‫⃗⃗⃗⃗⃗ ‪𝐴𝐵 .‬‬
‫‪ .1‬احسب الجداء السلمي بداللة ‪ a‬لكل من‪𝐵𝐹 :‬‬
‫⃗⃗⃗⃗⃗‬ ‫⃗⃗⃗⃗⃗ ‪𝐷𝐹 ،‬‬
‫⃗⃗⃗⃗⃗ ‪𝐴𝐺 .‬‬ ‫⃗⃗⃗⃗⃗ ‪𝐴𝐺 .‬‬
‫⃗⃗⃗⃗⃗ ‪𝐸𝐺 ،‬‬
‫⃗⃗⃗⃗⃗ ‪𝐴𝐶 .‬‬
‫⃗⃗⃗⃗⃗ ‪𝐴𝐺 ،‬‬
‫⃗⃗⃗⃗⃗ ‪𝐴𝐶 .‬‬
‫𝐹𝐷‬

‫⃗⃗⃗⃗⃗‬
‫⃗⃗⃗⃗⃗ ‪𝐴𝐵 .‬‬ ‫⃗⃗⃗⃗⃗ ⊥ ⃗⃗⃗⃗⃗‬
‫𝐵𝐴(‪𝐵𝐹 = 0‬‬ ‫ّ‬
‫)ألن 𝐹𝐵‬ ‫⃗⃗⃗⃗⃗ ;‬
‫⃗⃗⃗⃗⃗ ‪𝐴𝐵 .‬‬
‫𝐺𝐷‬ ‫=‬
‫⏟‬ ‫⃗⃗⃗⃗⃗‬
‫⃗⃗⃗⃗⃗ ‪𝐴𝐵 .‬‬
‫𝐶𝐷‬ ‫=‬
‫⏟‬ ‫‪𝑎. 𝑎 = 𝑎2‬‬
‫نسقط 𝐺 على )𝐷𝐵𝐴(‬ ‫مرتبطان خطيا وفي نفس االتجاه‬
‫𝐻𝐶 ‪⃗⃗⃗⃗⃗ .‬‬
‫𝐵𝐴‬ ‫⃗⃗⃗⃗⃗‬ ‫=‬
‫⏟‬ ‫𝐷𝐶 ‪⃗⃗⃗⃗⃗ .‬‬
‫𝐵𝐴‬ ‫⃗⃗⃗⃗⃗‬ ‫=‬
‫⏟‬ ‫𝑎‪− 𝑎. 𝑎 = −‬‬ ‫‪2‬‬

‫نسقط 𝐻 على )𝐶𝐵𝐴(‬ ‫مرتبطان خطيا ومتعكسان في االتجاه‬


‫⃗⃗⃗⃗⃗‬
‫⃗⃗⃗⃗⃗ ‪𝐴𝐶 .‬‬
‫𝐹𝐷‬ ‫=‬
‫⏟‬ ‫⃗⃗⃗⃗⃗‬
‫⃗⃗⃗⃗⃗⃗ ‪𝐴𝐶 .‬‬
‫= 𝐵𝐷‬ ‫)قطرا المربع متعامدان( ‪⏟ 0‬‬
‫نسقط 𝐹 على )𝐶𝐷𝐴(‬ ‫متعامدان‬
‫𝐺𝐴 ‪⃗⃗⃗⃗⃗ .‬‬
‫𝐶𝐴‬ ‫⃗⃗⃗⃗⃗‬ ‫=‬
‫⏟‬ ‫𝐶𝐴 ‪⃗⃗⃗⃗⃗ .‬‬
‫𝐶𝐴‬ ‫) ‪⃗⃗⃗⃗⃗ = 𝐴𝐶 2 = 2𝑎2 (𝐴𝐶 2 = 𝐴𝐵 2 + 𝐵𝐶 2‬‬
‫نسقط 𝐺 على )𝐶𝐵𝐴(‬
‫⃗⃗⃗⃗⃗‬
‫⃗⃗⃗⃗⃗ ‪𝐴𝐺 .‬‬
‫𝐺𝐸‬ ‫=‬
‫⏟‬ ‫⃗⃗⃗⃗⃗‬
‫⃗⃗⃗⃗⃗ ‪𝐸𝐺 .‬‬
‫) ‪𝐸𝐺 = 𝐸𝐺 2 = 2𝑎2 (𝐸𝐺 2 = 𝐸𝐹 2 + 𝐹𝐺 2‬‬
‫نسقط 𝐴 على )𝐺𝐹𝐸(‬
‫𝐹𝐷 ‪⃗⃗⃗⃗⃗ .‬‬
‫𝐺𝐴‬ ‫= ⃗⃗⃗⃗⃗‬ ‫𝐺𝐶 ‪⃗⃗⃗⃗⃗ +‬‬
‫𝐶𝐴( ⏟‬ ‫𝐵𝐷( ‪⃗⃗⃗⃗⃗ ).‬‬
‫𝐹𝐵 ‪⃗⃗⃗⃗⃗⃗ +‬‬ ‫𝐵𝐷 ‪⃗⃗⃗⃗⃗ .‬‬
‫⏟ = ) ⃗⃗⃗⃗⃗‬
‫𝐶𝐴‬ ‫𝐹𝐵 ‪⃗⃗⃗⃗⃗ .‬‬
‫⏟ ‪⃗⃗⃗⃗⃗⃗ +‬‬
‫𝐶𝐴‬ ‫𝐵𝐷 ‪⃗⃗⃗⃗⃗ .‬‬
‫⏟ ‪⃗⃗⃗⃗⃗ +‬‬
‫𝐺𝐶‬ ‫𝐹𝐵 ‪⃗⃗⃗⃗⃗ .‬‬
‫⏟ ‪⃗⃗⃗⃗⃗⃗ +‬‬
‫𝐺𝐶‬ ‫‪⃗⃗⃗⃗⃗ = 𝑎2‬‬
‫عالقة شال‬ ‫𝐵𝐷⊥ ⃗⃗⃗⃗⃗‬
‫𝐶𝐴(‪0‬‬ ‫) ⃗⃗⃗⃗⃗⃗‬ ‫𝐹𝐵⊥ ⃗⃗⃗⃗⃗‬
‫𝐶𝐴(‪0‬‬ ‫) ⃗⃗⃗⃗⃗‬ ‫𝐵𝐷⊥ ⃗⃗⃗⃗⃗‬
‫𝐺𝐶(‪0‬‬ ‫) ⃗⃗⃗⃗⃗⃗‬ ‫𝐹𝐵∥ ⃗⃗⃗⃗⃗‬
‫𝐺𝐶( ‪𝑎2‬‬ ‫) ⃗⃗⃗⃗⃗‬

‫المكونة للجداء السلمي واسقاط النقطة‬


‫ّ‬ ‫مالحظة هامة ‪ :‬طريقة اإلسقاط تعتمد على اختيار مستوي يشمل ثالث نقط من األربعة‬
‫الرابعة على هذا المستوي‪ .‬وال يصح اسقاط نقطتين في آن واحد بل نستعمل عالقة شال ‪ ،‬ولو أسقطنا كال من 𝐺 و 𝐹‬
‫⃗⃗⃗⃗⃗ وهذا خطأ‬ ‫⃗⃗⃗⃗⃗ = 𝐹𝐷‬
‫⃗⃗⃗⃗⃗ ‪𝐴𝐺 .‬‬ ‫⃗⃗⃗⃗⃗⃗ ‪𝐴𝐶 .‬‬
‫على المستوي )𝐶𝐵𝐴( في المثال األخير لكانت النتيجة ‪𝐷𝐵 = 0 :‬‬

‫𝐵𝐸 ‪⃗⃗⃗⃗⃗ .‬‬


‫𝐹𝐷 ‪ ،‬ث ّم استنتج ّ‬
‫أن المستقيم )𝐹𝐷( عمودي على المستوي )𝐺𝐸𝐵(‬ ‫⃗⃗⃗⃗⃗ ‪⃗⃗⃗⃗⃗ .‬‬
‫𝐹𝐷 و ‪⃗⃗⃗⃗⃗ = 0‬‬ ‫‪ .2‬بيّن ّ‬
‫أن ‪𝐸𝐺 = 0‬‬

‫⃗⃗⃗⃗⃗ ‪𝐷𝐹 .‬‬


‫⃗⃗⃗⃗⃗‬ ‫𝐺𝐸‬ ‫=‬
‫⏟‬ ‫⃗⃗⃗⃗⃗ ‪𝐻𝐹 .‬‬
‫⃗⃗⃗⃗⃗‬ ‫= 𝐺𝐸‬ ‫⃗⃗⃗⃗⃗ ; ‪⏟ 0‬‬
‫⃗⃗⃗⃗⃗ ‪𝐷𝐹 .‬‬
‫𝐵𝐸‬ ‫=‬
‫⏟‬ ‫⃗⃗⃗⃗⃗‬
‫⃗⃗⃗⃗⃗ ‪𝐴𝐹 .‬‬
‫= 𝐵𝐸‬ ‫‪⏟ 0‬‬
‫نسقط 𝐷 على )𝐺𝐹𝐸(‬ ‫متعامدان‬ ‫نسقط 𝐷 على )𝐵𝐹𝐸(‬ ‫متعامدان‬

‫أن المستقيم‬ ‫⃗⃗⃗⃗⃗ و 𝐵𝐸‬


‫⃗⃗⃗⃗⃗ وهما شعاعان غير مرتبطين خطيا من المستوي )𝐺𝐸𝐵( ‪ ،‬استنتج ّ‬ ‫⃗⃗⃗⃗⃗ عمودي على 𝐺𝐸‬
‫أن 𝐹𝐷‬ ‫بما ّ‬
‫)𝐹𝐷( عمودي على المستوي )𝐺𝐸𝐵(‬

‫‪ .3‬عيّن طبيعة المثلث ‪ DBG‬واحسب مساحته‪(𝑎 = 2𝑐𝑚) .‬‬

‫أن‪، 𝐷𝐵 = 𝐷𝐺 = 𝐵𝐺 :‬‬ ‫أن أضالع المثلث 𝐺𝐵𝐷 هي أوتار للمربعات المتقايسة 𝐷𝐶𝐵𝐴 ‪ ، 𝐶𝐷𝐻𝐺, 𝐵𝐶𝐺𝐹,‬نستنتج ّ‬
‫بما ّ‬
‫𝐼𝐺×𝐵𝐷‬
‫منه المثلث 𝐺𝐵𝐷 متقايس األضالع ومساحته هي ‪ ، 𝑆𝐷𝐵𝐺 = 2 :‬حيث 𝐼 هي المسقط العمودي للنقطة 𝐺 على ]𝐵𝐷[‬
‫(أي مركز المربّع 𝐷𝐶𝐵𝐴)‪.‬‬

‫𝑚𝑐 ‪𝐷𝐵 2 = 𝐷𝐴2 + 𝐴𝐵 2 = 2𝑎2 ⇒ 𝐷𝐵 = √2𝑎 = 2√2‬‬


‫‪2‬‬
‫‪1‬‬ ‫‪1‬‬ ‫‪1‬‬ ‫‪3‬‬ ‫‪3‬‬
‫‪𝐺𝐼 = 𝐺𝐷 − 𝐷𝐼 = 𝐺𝐷 − ( 𝐷𝐵) = 𝐺𝐷2 − 𝐷𝐵 2 = 𝐷𝐵 2 − 𝐷𝐵 2 = 𝐷𝐵 2 = (8) = 6‬‬
‫‪2‬‬ ‫‪2‬‬ ‫‪2‬‬ ‫‪2‬‬
‫‪2‬‬ ‫‪4‬‬ ‫‪4‬‬ ‫‪4‬‬ ‫‪4‬‬
‫‪2√2×√6‬‬
‫= 𝐺𝐵𝐷𝑆 ⇒ ‪𝐺𝐼 = √6‬‬ ‫‪= √12 = 2√3 𝑐𝑚2‬‬
‫‪2‬‬

‫‪21‬‬
‫الجزء الثاني ‪:‬‬

‫𝑘 ‪ (𝑂, 𝑖, 𝑗,‬النقط )‪، C(3; 1; −2) ، B(2; 2; 3) ، A(1; 0; −1‬‬ ‫نعتبر في الفضاء المنسوب إلى معلم متعامد ومتجانس ) ⃗‬
‫)‪D(−4; 2; 1‬‬
‫‪ .1‬بيّن أن النقط 𝐴 ‪ 𝐶, 𝐵,‬تعين مستويا‬
‫⃗⃗⃗⃗⃗ غير مرتبطين خ ّطيا‬ ‫لبيان أن النقط 𝑨 ‪ 𝑪, 𝑩,‬تعيّن مستويا (أو أنّها في استقامية) ‪ ،‬نبيّن أنّ الشعاعين 𝑩𝑨‬
‫⃗⃗⃗⃗⃗⃗ و 𝑪𝑨‬
‫‪1‬‬ ‫‪2‬‬ ‫‪1 2‬‬
‫⃗⃗⃗⃗⃗‬
‫⃗⃗⃗⃗⃗ ; )‪𝐴𝐵 (2‬‬ ‫⃗⃗⃗⃗⃗ غير مرتبطين خطيا ⇒ ≠ ; ) ‪𝐴𝐶 ( 1‬‬ ‫⃗⃗⃗⃗⃗ و 𝐶𝐴‬
‫النقط 𝐴 ‪ 𝐶, 𝐵,‬تعيّن مستويا ⇒ 𝐵𝐴‬
‫‪2 1‬‬
‫‪4‬‬ ‫‪−1‬‬
‫‪ .2‬عيّن شعاعا ناظميا للمستوي )‪(ABC‬‬
‫𝑎‬
‫⃗⃗⃗⃗⃗‬ ‫⃗⃗⃗⃗⃗‬
‫ليكن ) 𝑏( ⃗𝑛 شعاعا ناظميا للمستوي )𝐶𝐵𝐴( ‪ ،‬فهو إذن عمودي على كل من الشعاعين 𝐵𝐴 و 𝐶𝐴 ‪ ،‬منه ‪:‬‬
‫𝑐‬
‫⃗⃗⃗⃗⃗‬ ‫‪𝑎 + 2𝑏 + 4𝑐 = 0 … ‬‬ ‫‪𝑎 + 2𝑏 + 4𝑐 = 0‬‬
‫{ ⇒ ‪{𝑛⃗. 𝐴𝐵 = 0‬‬ ‫⇒‬ ‫{‬ ‫⇒‬ ‫‪− 3𝑎 + 6𝑐 = 0‬‬
‫⃗⃗⃗⃗⃗‬
‫‪𝑛⃗. 𝐴𝐶 = 0‬‬ ‫𝑎‪2‬‬ ‫‪+‬‬ ‫𝑏‬ ‫‪−‬‬ ‫𝑐‬ ‫=‬ ‫‪0‬‬ ‫…‬ ‫‪‬‬ ‫‪−2‬‬ ‫في‬ ‫‪‬‬ ‫المعادلة‬ ‫نضرب‬ ‫𝑎‪−4‬‬ ‫نجمع المعادلتين ‪− 2𝑏 + 2𝑐 = 0‬‬
‫‪2‬‬
‫⇒ 𝑐‪⇒ 𝑎 = 2‬‬ ‫⇒ 𝑐‪4𝑐 + 𝑏 − 𝑐 = 0 ⇒ 𝑏 = −3‬‬ ‫)‪𝑛⃗ (−3‬‬
‫نعوض 𝑎 في المعادلة ‪‬‬
‫ّ‬ ‫من أجل ‪𝑐=1‬‬
‫‪1‬‬
‫‪ .3‬استنتج معادلة ديكارتية للمستوي )‪(ABC‬‬
‫𝒂‬
‫𝒏 شعاعا ناظميا للمستوي )𝑪𝑩𝑨( ‪ ،‬فإنّ هذا األخير يقبل معادلة من الشكل ‪𝒂𝒙 + 𝒃𝒚 + 𝒄𝒛 + 𝒅 = 𝟎 :‬‬ ‫إذا كان )𝒃( ⃗⃗‬
‫𝒄‬
‫طريقة أولى ‪:‬‬
‫⇒ )𝐶𝐵𝐴( ∈ 𝐴 ; ‪(𝐴𝐵𝐶): 2𝑥 − 3𝑦 + 𝑧 + 𝑑 = 0‬‬ ‫‪2 − 1 + 𝑑 = 0 ⇒ 𝑑 = −1‬‬
‫نعوض إحداثيات 𝐴 في معادلة )𝐶𝐵𝐴(‬
‫ّ‬
‫‪(𝐴𝐵𝐶): 2𝑥 − 3𝑦 + 𝑧 − 1 = 0‬‬
‫طريقة ثانية ‪:‬‬
‫⃗⃗⃗⃗⃗⃗ ‪𝑀(𝑥; 𝑦; 𝑧) ∈ (𝐴𝐵𝐶) ⇒ 𝑛⃗.‬‬
‫‪𝐴𝑀 = 0 ⇒ 2(𝑥 − 1) − 3𝑦 + (𝑧 + 1) = 0 ⇒ 2𝑥 − 3𝑦 + 𝑧 − 1 = 0‬‬
‫‪ .4‬أثبت أن المثلث 𝐶𝐵𝐴 قائم‬
‫إلثبات أن المثلث 𝑪𝑩𝑨 قائم ‪ ،‬نستعمل الجداء السلمي أو النظرية العكسية لفيثاغورس‬
‫طريقة أولى ‪:‬‬
‫⃗⃗⃗⃗⃗ ‪⃗⃗⃗⃗⃗ .‬‬
‫𝐵𝐴‬ ‫𝐵𝐴 ⇒ ‪𝐴𝐶 = 1(2) + 2(1) + 4(−1) = 2 + 2 − 4 = 0‬‬ ‫𝐶𝐴 ‪⃗⃗⃗⃗⃗ ‬‬
‫المثلث 𝐶𝐵𝐴 قائم في 𝐴 ⇒ ⃗⃗⃗⃗⃗‬
‫طريقة ثانية ‪:‬‬

‫‪𝐴𝐵 = √1² + 2² + 4² = √21; 𝐴𝐶 = √2² + 1² + (−1)² = √6; 𝐵𝐶 = √1² + (−1)² + (−5)² = √27‬‬

‫المثلث 𝐶𝐵𝐴 قائم في 𝐴 ⇒ ‪𝐵𝐶 2 = 𝐴𝐵 2 + 𝐴𝐶 2‬‬


‫‪ .5‬أحسب مساحة المثلث ‪ABC‬‬
‫القاعدة × االرتفاع‬
‫مساحة المثلث تساوي‬
‫𝟐‬

‫‪𝐴𝐵 × 𝐴𝐶 √1² + 2² + 4² × √2² + 1² + (−1)² √21 × √6‬‬ ‫‪3‬‬


‫= 𝐶𝐵𝐴𝑆‬ ‫=‬ ‫=‬ ‫𝑎 ‪= √14 𝑢.‬‬
‫‪2‬‬ ‫‪2‬‬ ‫‪2‬‬ ‫‪2‬‬
‫‪ .6‬عيّن بعد النقطة ‪ D‬عن المستوي )‪ ، (ABC‬ثم احسب حجم رباعي الوجوه ‪DABC‬‬
‫|𝒅‪|𝒂𝒙′+𝒃𝒚′+𝒄𝒛′+‬‬
‫بعد النقطة )‪ 𝑴′(𝒙′; 𝒚′; 𝒛′‬عن المستوي 𝟎 = 𝒅 ‪ (𝑷): 𝒂𝒙 + 𝒃𝒚 + 𝒄𝒛 +‬هو ‪:‬‬
‫𝟐𝒄‪√𝒂𝟐 +𝒃𝟐 +‬‬

‫‪21‬‬
‫|‪|2(−4) − 3(2) + 1 − 1‬‬ ‫‪14‬‬
‫= ])𝐶𝐵𝐴( ‪𝑑[𝐷,‬‬ ‫=‬ ‫‪= √14‬‬
‫‪√22 + (−3)2 + 12‬‬ ‫‪√14‬‬
‫مساحة القاعدة × االرتفاع‬
‫حجم رباعي الوجوه يساوي‬
‫𝟑‬
‫‪1‬‬ ‫‪1 3‬‬ ‫‪14‬‬
‫= 𝐶𝐵𝐴𝐷𝑉‬ ‫⏟‬
‫× 𝑆‬ ‫⏟‬
‫‪ℎ‬‬ ‫= ‪= . √14. √14‬‬ ‫𝑣 ‪= 7 𝑢.‬‬
‫𝑆‪3‬‬ ‫])𝐶𝐵𝐴(‪𝑑[𝐷,‬‬
‫‪3‬‬ ‫‪2‬‬ ‫‪2‬‬
‫𝐶𝐵𝐴‬

‫‪ .7‬عين 𝜔 مركز سطح الكرة )‪ (S‬الذي معادلته ‪𝑥 + 𝑦 2 + 𝑧 2 + 2𝑦 − 6𝑧 − 15 = 0‬‬


‫‪2‬‬

‫لتعيين مركز ونصف قطر سطح كرة نكتب معادلته على الشكل ‪(𝒙 − 𝒙𝟎 )𝟐 + (𝒚 − 𝒚𝟎 )𝟐 + (𝒛 − 𝒛𝟎 )𝟐 = 𝒓𝟐 :‬‬
‫‪𝑥 2 + 𝑦 2 + 𝑧 2 + 2𝑦 − 6𝑧 − 15 = 0 ⇒ 𝑥 2 + (𝑦 + 1)2 − 12 + (𝑧 − 3)2 − 32 − 15 = 0‬‬
‫‪𝑥 2 + (𝑦 + 1)2 + (𝑧 − 3)2 = 25 ⇒ 𝜔(0; −1; 3) ; 𝑟 = √25 = 5‬‬
‫‪ .8‬أحسب بعد النقطة 𝜔 عن المستوي )‪(ABC‬‬
‫|‪|2(0) − 3(−1) + 3 − 1‬‬ ‫‪5‬‬ ‫‪5√14‬‬
‫= ])𝐶𝐵𝐴( ‪𝑑[𝜔,‬‬ ‫=‬ ‫=‬
‫‪√22 + (−3)2 + 12‬‬ ‫‪√14‬‬ ‫‪14‬‬
‫‪ .9‬اعط تمثيال وسيطيا للمستقيم )∆( الذي يشمل النقطة 𝜔 و يعامد المستوي )‪(ABC‬‬
‫𝒕𝜶 ‪𝒙 = 𝒙𝟎 +‬‬ ‫𝜶‬
‫التمثيل الوسيطي لمستقيم يشمل نقطة ) 𝟎𝒛 ; 𝟎𝒚 ; 𝟎𝒙( 𝟎𝑴 و شعاع توجيهه )𝜷( ⃗‬
‫𝒖 هو ‪{𝒚 = 𝒚𝟎 + 𝜷𝒕 ; 𝒕 ∈ ℝ :‬‬
‫𝒕𝜸 ‪𝒛 = 𝒛𝟎 +‬‬ ‫𝜸‬

‫⃗⃗⃗⃗⃗⃗⃗ ‪ ،‬أي ّ‬
‫إن الشعاع الناظمي للمستوي )𝐶𝐵𝐴( هو شعاع توجيه للمستقيم )∆(‬ ‫أن )𝐶𝐵𝐴( ⊥ )∆( ‪ّ ،‬‬
‫فإن ⃗𝑛 = )∆(𝑢‬ ‫بما ّ‬
‫𝑡‪𝑥 = 2‬‬ ‫𝑡‪𝑥 = 2‬‬
‫⃗⃗⃗⃗⃗⃗⃗‬ ‫⃗⃗⃗⃗⃗⃗⃗‬
‫‪𝑀(𝑥; 𝑦; 𝑧) ∈ (∆) ⇒ 𝜔𝑀 ∥ 𝑛⃗ ⇒ 𝜔𝑀 = 𝑡. 𝑛⃗ ⇒ {𝑦 + 1 = −3𝑡 ⇒ (∆): {𝑦 = −1 − 3𝑡 ; 𝑡 ∈ ℝ‬‬
‫𝑡=‪𝑧−3‬‬ ‫𝑡‪𝑧 =3+‬‬

‫‪ .11‬عيّن طبيعة و خصائص تقاطع المستوي )‪ (ABC‬و سطح الكرة )‪(S‬‬

‫لتعيين طبيعة تقاطع مستوي )𝑷( و سطح الكرة )𝑺( مركزها 𝝎 ونصف قطرها 𝒓 ‪ ،‬نقارن ])𝑷( ‪ 𝒅[𝝎,‬و 𝒓 ‪:‬‬
‫)ال يتقاطعان( ∅ = )𝑺( ∩ )𝑷( ⇒ 𝒓 > ])𝑷( ‪ 𝒅[𝝎,‬‬
‫‪‬‬ ‫)يتماسّان في نقطة وحيدة( }‪𝒅[𝝎, (𝑷)] = 𝒓 ⇒ (𝑷) ∩ (𝑺) = {𝝎′‬‬
‫‪‬‬ ‫)يتقاطعان وفق دائرة( ) ‪𝒅[𝝎, (𝑷)] < 𝒓 ⇒ (𝑷) ∩ (𝑺) = 𝑪(𝝎′ ;𝒓′‬‬
‫حيث ‪ 𝝎′‬هي نقطة تعامد المستقيم الذي يشمل 𝝎 والمستوي )𝑷( ‪ ،‬و ‪ 𝒓′‬يعطى بالعالقة ‪𝒓′ = √𝒓𝟐 − 𝒅𝟐 :‬‬

‫الحالة الثالثة ‪𝒅[𝝎, (𝑷)] < 𝒓 :‬‬ ‫الحالة الثانية ‪𝒅[𝝎, (𝑷)] = 𝒓 :‬‬ ‫الحالة األولى ‪𝒅[𝝎, (𝑷)] > 𝒓 :‬‬

‫) ‪(𝑷) ∩ (𝑺) = 𝑪(𝝎′ ;𝒓′‬‬ ‫}‪(𝑷) ∩ (𝑺) = {𝝎′‬‬ ‫∅ = )𝑺( ∩ )𝑷(‬

‫‪21‬‬
‫أن 𝑟 < ])𝐶𝐵𝐴( ‪ّ ، 𝑑[𝜔,‬‬
‫فإن المستوي )𝐶𝐵𝐴( يقطع سطح الكرة )𝑆( وفق دائرة مركزها ‪ 𝜔′‬ونصف قطرها ‪ 𝑟′‬حيث ‪𝜔′‬‬ ‫بما ّ‬
‫‪5√182‬‬ ‫‪25‬‬
‫= ‪𝑟′‬‬ ‫هي نقطة تقاطع المستوي )𝐶𝐵𝐴( والمستقيم )∆( ‪ ،‬و ‪ ، 𝑟 ′ = √𝑟 2 − 𝑑 2 = √25 − 14‬أي‬
‫‪14‬‬

‫لدراسة الوضع النسبي لمستقيم )∆( ومستوي )𝑃( ‪ ،‬نع ّوض التمثيل الوسيطي للمستقيم )∆( في المعادلة الديكارتية‬
‫للمستوي )𝑃( فنحصل على معادلة من الدرجة األولى ذات المجهول 𝑡 (الوسيط) ‪ ،‬ونميّز ثالث حاالت ‪:‬‬
‫‪ ‬المعادلة ال تقبل حلوال )‪ = 0‬عدد( ‪ :‬المستقيم )∆( يوازي المستوي )𝑃(‬
‫‪ ‬المعادلة تقبل حال وحيدا )عدد = 𝑡( ‪ :‬المستقيم )∆( يقطع المستوي )𝑃( في نقطة وحيدة‬
‫‪ ‬المعادلة تقبل ما ال نهاية من الحلول )‪ : (0 = 0‬المستقيم )∆( محتوى في المستوي )𝑃(‬

‫نعوض التمثيل الوسيطي للمستقيم )∆( في المعادلة الديكارتية للمستوي )𝐶𝐵𝐴( ‪:‬‬ ‫لتعيين إحداثيات ‪ّ 𝜔′‬‬
‫‪5‬‬ ‫‪5 1 37‬‬
‫‪2(2𝑡) − 3(−1 − 3𝑡) + 3 + 𝑡 − 1 = 0 ⇒ 14𝑡 = −5 ⇒ 𝑡 = −‬‬ ‫) ; ; ‪⇒ 𝜔′ (−‬‬
‫‪14‬‬ ‫‪7 14 14‬‬
‫‪ .11‬ليكن )‪ (P‬المستوي الذي معادلته‪ .𝑥 + 𝑦 + 𝑧 − 1 = 0 :‬بيّن ّ‬
‫أن المستويين )‪ (P‬و )‪ (ABC‬متعامدان‬
‫⃗⃗⃗ شعاعا ناظميا للمستوي )𝑃(‪ .‬لدينا ‪:‬‬
‫ليكن )‪𝑛′ (1; 1; 1‬‬
‫⃗⃗⃗ ‪𝑛⃗.‬‬
‫⃗⃗⃗ ⊥ ⃗𝑛 ⇒ ‪𝑛′ = 2(1) − 3(1) + 1(1) = 0‬‬
‫)𝑃( ⊥ )𝐶𝐵𝐴( ⇒ ‪𝑛′‬‬
‫‪ .12‬اعط التمثيل الوسيطي للمستقيم )‪ (∆′‬تقاطع )‪ (P‬و )‪(ABC‬‬
‫)𝐶𝐵𝐴( ∈ 𝑀‬ ‫‪2𝑥 − 3𝑦 + 𝑧 − 1 = 0 … ‬‬
‫{ ⇒ ) ‪𝑀(𝑥; 𝑦; 𝑧) ∈ (∆′‬‬ ‫{⇒‬ ‫𝑦‪⇒ 𝑥 − 4𝑦 = 0 ⇒ 𝑥 = 4‬‬
‫)𝑃( ∈ 𝑀‬ ‫‪𝑥 + 𝑦 + 𝑧 − 1 = 0 …  −‬‬
‫بتعويض 𝑥 في المعادلة ‪ ‬نجد ‪ ، 4𝑦 + 𝑦 + 𝑧 − 1 = 0 :‬منه ‪ ، 𝑧 = −5𝑦 + 1‬وبوضع ‪ 𝑦 = 𝑡′‬نحصل على‬
‫التمثيل الوسيطي للمستقيم )‪.(∆′‬‬
‫‪𝑥 = 4𝑡′‬‬
‫‪(∆′): { 𝑦 = 𝑡′‬‬ ‫‪; 𝑡′ ∈ ℝ‬‬
‫‪𝑧 = 1 − 5𝑡′‬‬
‫‪ .13‬عين بعد النقطة ‪ D‬عن المستوي )‪ ، (P‬ث ّم استنتج المسافة بين ‪ D‬و )‪(∆′‬‬
‫|‪|1(−4) + 1(2) + 1(1) − 1‬‬ ‫‪2‬‬‫‪2√3‬‬
‫= ])𝑃( ‪𝑑[𝐷,‬‬ ‫=‬ ‫=‬
‫‪√12 + 12 + 12‬‬ ‫‪√3‬‬ ‫‪3‬‬
‫أن المستويين )‪ (P‬و )‪ (ABC‬متعامدان ‪ّ ،‬‬
‫فإن المسافات ])𝐶𝐵𝐴( ‪ 𝑑[𝐷, (𝑃)] ، 𝑑[𝐷,‬و ])‪ 𝑑[𝐷, (∆′‬هي أطوال‬ ‫بما ّ‬
‫أضالع مثلث قائم طول وتره هو ])‪.𝑑[𝐷, (∆′‬‬
‫‪4 46‬‬ ‫‪46‬‬ ‫‪√138‬‬
‫‪𝑑2 [𝐷, (∆′)] = 𝑑2 [𝐷, (𝐴𝐵𝐶)] + 𝑑2 [𝐷, (𝑃)] = 14 +‬‬ ‫=‬ ‫= √ = ])‪⇒ 𝑑[𝐷, (∆′‬‬
‫‪3‬‬ ‫‪3‬‬ ‫‪3‬‬ ‫‪3‬‬

‫)𝑃(‬

‫)‪(∆′‬‬

‫)𝐶𝐵𝐴(‬

‫‪21‬‬
‫‪ .14‬ادرس الوضع النسبي للمستقيمين )∆( و )‪(∆′‬‬
‫⃗⃗⃗ شعاعي توجيه المستقيمين )∆( و )‪ (∆′‬على الترتيب‪.‬‬
‫𝑢 و ‪𝑢′‬‬
‫ليكن ⃗‬
‫‪2‬‬ ‫‪4‬‬ ‫‪2‬‬ ‫‪3‬‬
‫⃗⃗⃗ ; )‪⃗ (−3‬‬
‫𝑢‬ ‫‪𝑢′ ( 1 ) ; ≠ −‬‬
‫‪4‬‬ ‫‪1‬‬
‫‪1‬‬ ‫‪−5‬‬
‫فإن المستقيمين )∆( و )‪ (∆′‬متقاطعان أو ال ينتميان لنفس المستوي‪.‬‬ ‫⃗⃗⃗ غير مرتبطين خ ّ‬
‫طيا ‪ّ ،‬‬ ‫𝑢 و ‪𝑢′‬‬ ‫بما ّ‬
‫أن الشعاعين ⃗‬
‫لتحديد الوضع النسبي للمستقيمين )∆( و )‪ ، (∆′‬نح ّل الجملة التالية حيث )∆( ∈ 𝑀 و) ‪:𝑀′ ∈ (∆′‬‬
‫‪′‬‬
‫‪1‬‬
‫‪𝑥𝑀 = 𝑥𝑀′‬‬ ‫‪′‬‬ ‫‪′‬‬ ‫𝑡‬ ‫=‬ ‫‪−‬‬
‫‪2𝑡 = 4𝑡 … ‬‬ ‫𝑡‪2𝑡 = 4‬‬
‫{ ⇒ 𝑦 = 𝑦{‬ ‫⇒‬ ‫{‬ ‫{ ⇒ ‪⇒ { 𝑡 = 2𝑡′‬‬ ‫‪7‬‬
‫𝑀‬ ‫‪𝑀′‬‬ ‫‪ 4 + 12𝑡 = −4𝑡 ′‬نضرب المعادلة ‪ ‬في ‪−1 − 3𝑡 = 𝑡 …  −4‬‬
‫‪′‬‬
‫‪14𝑡 = −4‬‬ ‫‪2‬‬
‫‪𝑡=−‬‬
‫‪7‬‬
‫‪4‬‬ ‫‪1 12‬‬ ‫‪4‬‬ ‫‪1 19‬‬
‫بتعويض 𝑡 و‪ 𝑡′‬في التمثيلين الوسيطيين لـ )∆( و )‪ (∆′‬نحصل على النقطتين ) ‪ (− 7 ; − 7 ; 7‬و ) ‪(− 7 ; − 7 ; 7‬‬
‫أن ‪ ، 𝑧𝑀 ≠ 𝑧𝑀′‬نستنتج ّ‬
‫أن المستقيمين )∆( و )‪ (∆′‬ال ينتميان إلى نفس المستوي‪.‬‬ ‫بما ّ‬
‫‪ .15‬اعط المعادلة الديكارتية لـ )‪ (Q‬المستوي المحوري للقطعة ]‪ [EF‬حيث )‪ 𝐸(1; 0; 0‬و )‪𝐹(−1; 0; 2‬‬
‫المستوي المحوري للقطعة ]𝑩𝑨[ هو المستوي الذي يعامد هذه القطعة في منتصفها ‪ ،‬فهو إذن يعامد ⃗⃗⃗⃗⃗⃗‬
‫𝑩𝑨 ويشمل 𝑰‬
‫منتصف ]𝑩𝑨[‬
‫⃗⃗⃗⃗⃗‬
‫‪𝐸𝐹 (−2; 0; 2) ; 𝐼(0; 0; 1) ; 𝑀(𝑥; 𝑦; 𝑧) ∈ (𝑄) ⇒ −2𝑥 + 2𝑧 + 𝑑 = 0 ; 𝐼 ∈ (𝑄) ⇒ 2 + 𝑑 = 0‬‬
‫‪⇒ 𝑑 = −2 ⇒ (𝑄): −2𝑥 + 2𝑧 − 2 = 0‬‬
‫‪ .16‬عيّن تقاطع المستويات الثالث )‪ (Q) ، (P‬و )‪(ABC‬‬
‫لتعيين تقاطع المستويات الثالث )‪ (Q) ، (P‬و )‪ ، (ABC‬ندرس تقاطع المستوي )𝑄( والمستقيم )‪(∆′‬‬
‫‪⏟ ∩ (𝐴𝐵𝐶) ⇒ 𝑀 ∈ (𝑄) ∩ (∆′ ) ; −2(4𝑡 ′ ) + 2(1 − 5𝑡 ′ ) − 2 = 0 ⇒ −18𝑡 ′ = 0‬‬
‫)𝑃( ∩ )𝑄( ∈ 𝑀‬
‫) ‪(∆′‬‬

‫‪⇒ 𝑡′ = 0 ⇒ ′‬‬ ‫})‪(𝑄) ∩ (𝑃) ∩ (𝐴𝐵𝐶) = {𝐼(0; 0; 1‬‬


‫نعوض ‪ 𝑡 ′‬في التمثيل الوسيطي لـ ) ∆(‬
‫ّ‬
‫‪ .17‬عيّن إحداثيات النقطتين ‪ G‬و '‪ G‬حيث ‪ G‬مركز ثقل المثلث ‪ ABC‬و '‪ G‬مرجح الجملة })‪{(𝐴; 3), (𝐵; −1); (𝐶; 1‬‬

‫إذا كان 𝟎 ≠ 𝜸 ‪ ، 𝜶 + 𝜷 +‬فإنّ الجملة المثقلة })𝜸 ;𝑪( ;)𝜷 ;𝑩( ‪ {(𝑨; 𝜶),‬تقبل مرجحا وحيدا 𝑮 حيث ‪:‬‬
‫‪‬‬ ‫𝑩𝑮𝜷 ‪⃗⃗⃗⃗⃗ +‬‬
‫𝑨𝑮𝜶‬ ‫𝑪𝑮𝜸 ‪⃗⃗⃗⃗⃗⃗ +‬‬
‫𝟎 = ⃗⃗⃗⃗⃗‬
‫⃗‬
‫𝑪𝒛𝜸‪𝜶𝒙𝑨 +𝜷𝒙𝑩 +𝜸𝒙𝑪 𝜶𝒚𝑨 +𝜷𝒚𝑩 +𝜸𝒚𝑪 𝜶𝒛𝑨 +𝜷𝒛𝑩 +‬‬
‫‪‬‬ ‫(𝑮‬ ‫;‬ ‫;‬ ‫)‬
‫𝜸‪𝜶+𝜷+‬‬ ‫𝜸‪𝜶+𝜷+‬‬ ‫𝜸‪𝜶+𝜷+‬‬

‫‪‬‬ ‫𝑩𝑴𝜷 ‪⃗⃗⃗⃗⃗⃗⃗ +‬‬


‫𝑨𝑴𝜶 ; 𝑴∀‬ ‫𝑮𝑴)𝜸 ‪⃗⃗⃗⃗⃗⃗ = (𝜶 + 𝜷 +‬‬
‫𝑪𝑴𝜸 ‪⃗⃗⃗⃗⃗⃗⃗ +‬‬ ‫⃗⃗⃗⃗⃗⃗⃗‬
‫𝑮𝑴‖ هي النقطة 𝑮‬‫مجموعة النقط 𝑴 التي تحقّق ‪⃗⃗⃗⃗⃗⃗⃗ ‖ = 𝟎 :‬‬ ‫‪‬‬
‫𝑮𝑴‖ هي سطح كرة مركزها 𝑮 ونصف قطرها 𝒌‬ ‫مجموعة النقط 𝑴 التي تحقّق ‪⃗⃗⃗⃗⃗⃗⃗ ‖ = 𝒌(𝒌 > 𝟎) :‬‬ ‫‪‬‬
‫𝑮𝑴‖ هي المستوي المحوري للقطعة ] ‪[𝑮𝑮′‬‬ ‫مجموعة النقط 𝑴 التي تحقّق ‪⃗⃗⃗⃗⃗⃗⃗⃗ ‖ :‬‬
‫‪⃗⃗⃗⃗⃗⃗⃗ ‖ = ‖𝑴𝑮′‬‬ ‫‪‬‬
‫⃗⃗⃗⃗⃗⃗⃗ هي سطح كرة قطرها ] ‪[𝑮𝑮′‬‬ ‫مجموعة النقط 𝑴 التي تحقّق ‪𝑴𝑮′ = 𝟎 :‬‬
‫⃗⃗⃗⃗⃗⃗⃗ ‪𝑴𝑮.‬‬ ‫‪‬‬
‫مجموعة النقط 𝑴 التي تحقّق ‪⃗⃗⃗⃗⃗⃗ = 𝟎 :‬‬
‫𝑩𝑨 ‪⃗⃗⃗⃗⃗⃗⃗ .‬‬
‫𝑮𝑴 هي المستوي الذي يشمل النقطة 𝑮 ويعامد ⃗⃗⃗⃗⃗⃗‬
‫𝑩𝑨‬ ‫‪‬‬
‫مرجح الجملة })𝟏 ;𝑪( ;)𝟏 ;𝑩( ‪ {(𝑨; 𝟏),‬هو مركز ثقل المثلث ‪ABC‬‬ ‫‪‬‬
‫مرجح الجملة })𝟏 ;𝑩( ‪ {(𝑨; 𝟏),‬هو منتصف القطعة ]𝑩𝑨[‬ ‫‪‬‬
‫𝑩𝑴𝜷 ‪⃗⃗⃗⃗⃗⃗⃗ +‬‬
‫𝑨𝑴𝜶 مستقل عن 𝑴‬ ‫إذا كان 𝟎 = 𝜸 ‪ ، 𝜶 + 𝜷 +‬فإنّ الشعاع ⃗⃗⃗⃗⃗⃗‬
‫𝑪𝑴𝜸 ‪⃗⃗⃗⃗⃗⃗⃗ +‬‬

‫𝐶𝑧 ‪𝑥𝐴 + 𝑥𝐵 + 𝑥𝐶 𝑦𝐴 + 𝑦𝐵 + 𝑦𝐶 𝑧𝐴 + 𝑧𝐵 +‬‬


‫( 𝐺 ⇒ })‪𝐺{(𝐴; 1), (𝐵; 1); (𝐶; 1‬‬ ‫;‬ ‫;‬ ‫)‪) ⇒ 𝐺(2; 1; 0‬‬
‫‪3‬‬ ‫‪3‬‬ ‫‪3‬‬

‫‪21‬‬
‫𝐶𝑧 ‪3𝑥𝐴 − 𝑥𝐵 + 𝑥𝐶 3𝑦𝐴 − 𝑦𝐵 + 𝑦𝐶 3𝑧𝐴 − 𝑧𝐵 +‬‬
‫( 𝐺 ⇒ })‪𝐺′{(𝐴; 3), (𝐵; −1); (𝐶; 1‬‬ ‫;‬ ‫;‬ ‫)‬
‫‪3‬‬ ‫‪3‬‬ ‫‪3‬‬
‫‪4‬‬ ‫‪1‬‬ ‫‪8‬‬
‫) ‪⇒ 𝐺′ ( ; − ; −‬‬
‫‪3‬‬ ‫‪3‬‬ ‫‪3‬‬
‫‪ .18‬عيّن في كل حالة من الحاالت التالية مجموعة النقط ‪ M‬من الفضاء التي تحقق ‪:‬‬
‫𝐵𝑀 ‪⃗⃗⃗⃗⃗⃗ +‬‬
‫𝐴𝑀‖‬ ‫أ‪⃗⃗⃗⃗⃗⃗ ‖ = 6 …  .‬‬
‫𝐶𝑀 ‪⃗⃗⃗⃗⃗⃗ +‬‬

‫⃗⃗⃗⃗⃗⃗‬
‫𝐴𝑀‖‬
‫𝐵𝑀 ‪⏟ +‬‬ ‫𝐺𝑀‪⃗⃗⃗⃗⃗⃗ ‖ = 6 ⇒ ‖3‬‬
‫𝐶𝑀 ‪⃗⃗⃗⃗⃗⃗ +‬‬ ‫𝐺𝑀‖‪⃗⃗⃗⃗⃗⃗ ‖ = 6 ⇒ 3‬‬
‫𝐺𝑀‖ ⇒ ‪⃗⃗⃗⃗⃗⃗ ‖ = 6‬‬
‫‪⃗⃗⃗⃗⃗⃗ ‖ = 2‬‬
‫⃗⃗⃗⃗⃗⃗‬
‫𝐺𝑀‪3‬‬
‫مجموعة النقط 𝑀 التي تحقّق المعادلة ‪ ‬هي سطح كرة مركزها 𝐺 ونصف قطرها ‪2‬‬
‫⃗⃗⃗⃗⃗⃗ ‪⃗⃗⃗⃗⃗⃗ +‬‬
‫𝐴𝑀‖‬ ‫⃗⃗⃗⃗⃗⃗ ‪𝑀𝐵 +‬‬ ‫⃗⃗⃗⃗⃗⃗ ‪⃗⃗⃗⃗⃗⃗ −‬‬
‫𝐴𝑀‪𝑀𝐶 ‖ = ‖3‬‬ ‫⃗⃗⃗⃗⃗⃗ ‪𝑀𝐵 +‬‬
‫ب‪𝑀𝐶 ‖ …  .‬‬

‫⃗⃗⃗⃗⃗⃗‬
‫𝐴𝑀‖‬
‫𝐵𝑀 ‪⏟ +‬‬ ‫𝐴𝑀‪⃗⃗⃗⃗⃗⃗ ‖ = ‖3‬‬
‫𝐶𝑀 ‪⃗⃗⃗⃗⃗⃗ +‬‬ ‫𝐵𝑀 ‪⏟⃗⃗⃗⃗⃗⃗ −‬‬ ‫𝐺𝑀‪⃗⃗⃗⃗⃗⃗ ‖ ⇒ ‖3‬‬
‫𝐶𝑀 ‪⃗⃗⃗⃗⃗⃗ +‬‬ ‫𝐺𝑀‖ ⇒ ‪⃗⃗⃗⃗⃗⃗⃗ ‖ = 6‬‬
‫‪⃗⃗⃗⃗⃗⃗ ‖ = ‖3𝑀𝐺′‬‬ ‫‖ ⃗⃗⃗⃗⃗⃗⃗‬
‫‪⃗⃗⃗⃗⃗⃗ ‖ = ‖𝑀𝐺′‬‬
‫⃗⃗⃗⃗⃗⃗‬
‫𝐺𝑀‪3‬‬ ‫⃗⃗⃗⃗⃗⃗⃗⃗‬
‫‪3𝑀𝐺′‬‬
‫مجموعة النقط 𝑀 التي تحقّق المعادلة ‪ ‬هي المستوي المحوري للقطعة ] ‪[𝐺𝐺 ′‬‬

‫𝐵𝑀 ‪⃗⃗⃗⃗⃗⃗ +‬‬


‫𝐴𝑀‖‬ ‫𝐴𝑀‪⃗⃗⃗⃗⃗⃗ ‖ = ‖2‬‬
‫𝐶𝑀 ‪⃗⃗⃗⃗⃗⃗ +‬‬ ‫𝐵𝑀 ‪⃗⃗⃗⃗⃗⃗ −‬‬ ‫ج‪⃗⃗⃗⃗⃗⃗ ‖ …  .‬‬
‫𝐶𝑀 ‪⃗⃗⃗⃗⃗⃗ −‬‬

‫⃗⃗⃗⃗⃗⃗‬
‫𝐴𝑀‖‬
‫𝐵𝑀 ‪⏟ +‬‬ ‫𝐴𝑀‪⃗⃗⃗⃗⃗⃗ ‖ = ‖2‬‬
‫𝐶𝑀 ‪⃗⃗⃗⃗⃗⃗ +‬‬ ‫𝐵𝑀 ‪⏟⃗⃗⃗⃗⃗⃗ −‬‬ ‫𝐺𝑀‪⃗⃗⃗⃗⃗⃗ ‖ ⇒ ‖3‬‬
‫𝐶𝑀 ‪⃗⃗⃗⃗⃗⃗ −‬‬ ‫𝐴𝑀‪⃗⃗⃗⃗⃗⃗ ‖ = ‖2‬‬
‫𝐴𝑀( ‪⃗⃗⃗⃗⃗⃗ −‬‬
‫𝐵𝐴 ‪⃗⃗⃗⃗⃗⃗ +‬‬
‫𝐴𝑀( ‪⃗⃗⃗⃗⃗ ) −‬‬
‫𝐶𝐴 ‪⃗⃗⃗⃗⃗⃗ +‬‬
‫‖) ⃗⃗⃗⃗⃗‬
‫⃗⃗⃗⃗⃗⃗‬
‫𝐺𝑀‪3‬‬ ‫شعاع مستقل عن 𝑀‬
‫𝐵𝐴(‪⃗⃗⃗⃗⃗⃗ ‖ = ‖−‬‬
‫𝐺𝑀‖‪⇒ 3‬‬ ‫⃗⃗⃗⃗⃗ ‪⃗⃗⃗⃗⃗ +‬‬
‫𝐺𝑀‖‪𝐴𝐶 )‖ ⇒ 3‬‬ ‫𝐵𝐴‖ = ‖ ⃗⃗⃗⃗⃗⃗‬ ‫⃗⃗⃗⃗⃗ ‪⃗⃗⃗⃗⃗ +‬‬
‫‖ 𝐶𝐴‬
‫‪1‬‬ ‫‪2‬‬ ‫‪3‬‬
‫⃗⃗⃗⃗⃗‬
‫⃗⃗⃗⃗⃗ ; )‪𝐴𝐵 (2‬‬
‫⃗⃗⃗⃗⃗ ⇒ ) ‪𝐴𝐶 ( 1‬‬ ‫⃗⃗⃗⃗⃗ ‪𝐴𝐵 +‬‬
‫𝐵𝐴‖ ⇒ )‪𝐴𝐶 (3‬‬ ‫⃗⃗⃗⃗⃗ ‪⃗⃗⃗⃗⃗ +‬‬
‫𝐺𝑀‖ ⇒ ‪𝐴𝐶 ‖ = 3√3‬‬ ‫‪⃗⃗⃗⃗⃗⃗ ‖ = √3‬‬
‫‪4‬‬ ‫‪−1‬‬ ‫‪3‬‬
‫مجموعة النقط 𝑀 التي تح ّقق المعادلة ‪ ‬هي سطح كرة مركزها 𝐺 ونصف قطرها ‪√3‬‬
‫⃗⃗⃗⃗⃗⃗ ‪⃗⃗⃗⃗⃗⃗ +‬‬
‫𝐴𝑀(‬ ‫⃗⃗⃗⃗⃗⃗ ‪𝑀𝐵 +‬‬
‫𝐴𝑀‪𝑀𝐶 )(3‬‬ ‫⃗⃗⃗⃗⃗⃗ ‪⃗⃗⃗⃗⃗⃗ −‬‬
‫⃗⃗⃗⃗⃗⃗ ‪𝑀𝐵 +‬‬
‫د‪𝑀𝐶 ) = 0 …  .‬‬
‫⃗⃗⃗⃗⃗⃗ ‪⃗⃗⃗⃗⃗⃗ +‬‬
‫𝐴𝑀(‬ ‫⃗⃗⃗⃗⃗⃗ ‪𝑀𝐵 +‬‬ ‫⃗⃗⃗⃗⃗⃗ ‪⃗⃗⃗⃗⃗⃗ −‬‬
‫𝐴𝑀‪𝑀𝐶 )(3‬‬ ‫⃗⃗⃗⃗⃗⃗ ‪𝑀𝐵 +‬‬
‫𝐺𝑀‪𝑀𝐶 ) = 0 ⇒ (3‬‬ ‫𝐺𝑀‪⃗⃗⃗⃗⃗⃗ )(3‬‬
‫⃗⃗⃗⃗⃗⃗ ⇒ ‪⃗⃗⃗⃗⃗⃗⃗⃗′ ) = 0‬‬ ‫⃗⃗⃗⃗⃗⃗⃗ ‪𝑀𝐺 .‬‬
‫‪𝑀𝐺′ = 0‬‬
‫مجموعة النقط 𝑀 التي تح ّقق المعادلة ‪ ‬هي سطح كرة قطرها ] 𝐺𝐺[‬
‫‪′‬‬

‫⃗⃗⃗⃗⃗⃗ ‪⃗⃗⃗⃗⃗⃗ −‬‬


‫𝐴𝑀‪(3‬‬ ‫⃗⃗⃗⃗⃗⃗ ‪𝑀𝐵 +‬‬
‫𝐴𝑀() 𝐶𝑀‬ ‫⃗⃗⃗⃗⃗⃗ ‪⃗⃗⃗⃗⃗⃗ −‬‬
‫ه‪𝑀𝐵 ) = 0 …  .‬‬
‫⃗⃗⃗⃗⃗⃗ ‪⃗⃗⃗⃗⃗⃗ −‬‬
‫𝐴𝑀‪(3‬‬ ‫⃗⃗⃗⃗⃗⃗ ‪𝑀𝐵 +‬‬ ‫⃗⃗⃗⃗⃗⃗ ‪⃗⃗⃗⃗⃗⃗ −‬‬
‫𝐴𝑀() 𝐶𝑀‬ ‫⇒ ‪𝑀𝐵 ) = 0‬‬ ‫𝐵𝐴‪⃗⃗⃗⃗⃗⃗⃗ ) (−‬‬
‫‪(3𝑀𝐺′‬‬ ‫⃗⃗⃗⃗⃗⃗⃗ ⇒ ‪⃗⃗⃗⃗⃗ ) = 0‬‬
‫⃗⃗⃗⃗⃗ ‪𝑀𝐺′.‬‬
‫‪𝐴𝐵 = 0‬‬
‫⃗⃗⃗⃗⃗⃗⃗‬
‫𝐴𝑀=𝐵𝑀‬ ‫𝐵𝐴‪⃗⃗⃗⃗⃗⃗ +‬‬
‫⃗⃗⃗⃗⃗‬

‫مجموعة النقط 𝑀 التي تح ّقق المعادلة ‪ ‬هي المستوي الذي يشمل النقطة ‪ 𝐺′‬ويعامد 𝐵𝐴‬
‫⃗⃗⃗⃗⃗‬

‫‪29‬‬
‫و‪𝑀𝐴2 + 𝑀𝐵 2 + 𝑀𝐶 2 = 30 …  .‬‬
‫𝐵𝑀 ‪⃗⃗⃗⃗⃗⃗ 2 +‬‬
‫𝐴𝑀 ⇒ ‪𝑀𝐴2 + 𝑀𝐵 2 + 𝑀𝐶 2 = 30‬‬ ‫‪⃗⃗⃗⃗⃗⃗ 2 = 30‬‬
‫𝐶𝑀 ‪⃗⃗⃗⃗⃗⃗ 2 +‬‬
‫‪2‬‬ ‫‪2‬‬ ‫‪2‬‬
‫𝐴𝐺 ‪⃗⃗⃗⃗⃗⃗ +‬‬
‫𝐺𝑀( ⇒‬ ‫𝐺𝑀( ‪⃗⃗⃗⃗⃗ ) +‬‬
‫𝐵𝐺 ‪⃗⃗⃗⃗⃗⃗ +‬‬
‫𝐺𝑀( ‪⃗⃗⃗⃗⃗ ) +‬‬
‫𝐶𝐺 ‪⃗⃗⃗⃗⃗⃗ +‬‬
‫‪⃗⃗⃗⃗⃗ ) = 30‬‬

‫⃗⃗⃗⃗⃗ ‪⃗⃗⃗⃗⃗⃗ 2 +‬‬


‫𝐺𝑀‪⇒ 3‬‬ ‫⃗⃗⃗⃗⃗ ‪𝐺𝐴2 +‬‬
‫⃗⃗⃗⃗⃗ ‪𝐺𝐵 2 +‬‬ ‫𝐴𝐺( ⃗⃗⃗⃗⃗⃗‬
‫𝐺𝑀‪𝐺𝐶 2 + 2‬‬ ‫⃗⃗⃗⃗⃗‬
‫⃗⃗⃗⃗⃗ ‪⏟ +‬‬
‫⃗⃗⃗⃗⃗ ‪𝐺𝐵 +‬‬
‫‪𝐺𝐶 ) = 30‬‬
‫⃗‬
‫‪0‬‬
‫‪2‬‬ ‫‪2‬‬ ‫‪2‬‬ ‫‪2‬‬
‫𝐶𝐺 ‪⇒ 3𝑀𝐺 = 30 − 𝐺𝐴 − 𝐺𝐵 −‬‬
‫)‪⃗⃗⃗⃗⃗ (−1; −1; −1‬‬
‫𝐴𝐺‬ ‫‪𝐺𝐴2 = (−1)2 + (−1)2 + (−1)2 = 3‬‬
‫⃗⃗⃗⃗⃗ {‬‫{ ⇒ )‪𝐺𝐵 (0; 1; 3‬‬ ‫‪𝐺𝐵 2 = (1)2 + (3)2 = 10‬‬ ‫‪⇒ 3𝑀𝐺 2 = 12 ⇒ 𝑀𝐺 2 = 4‬‬
‫)‪⃗⃗⃗⃗⃗ (1; 0; −2‬‬
‫𝐶𝐺‬ ‫‪𝐺𝐶 2 = (1)2 + (−2)2 = 5‬‬
‫مجموعة النقط 𝑀 التي تح ّقق المعادلة ‪ ‬هي سطح كرة مركزها 𝐺 ونصف قطرها ‪2‬‬
‫‪ .19‬ادرس الوضع النسبي لسطح الكرة )𝑆( و المجموعة (أ) (السؤال السابق)‬

‫لدراسة الوضع النسبي لسطحي كرة )𝑺( و)‪ (𝑺′‬نقارن المسافة بين مركزيهما ) ‪ 𝒅(𝝎; 𝝎′‬مع مجموع نصفي قطريهما ‪𝒓 + 𝒓′‬‬
‫‪ (𝑺) : 𝒅(𝝎; 𝝎′ ) > 𝒓 + 𝒓′ ‬و)‪ (𝑺′‬منفصلتان‬
‫‪ (𝑺) : 𝒅(𝝎; 𝝎′ ) = 𝒓 + 𝒓′ ‬و)‪ (𝑺′‬متماستان‬
‫‪ (𝑺) : 𝒅(𝝎; 𝝎′ ) < 𝒓 + 𝒓′ ‬و)‪ (𝑺′‬متقاطعتان‬

‫لدينا ‪ :‬سطح الكرة )𝑆( مركزها 𝜔 ونصف قطرها ‪ ، 5‬و سطح الكرة )‪ (𝑆′‬مركزها 𝐺 ونصف قطرها ‪ ، 2‬منه ‪:‬‬
‫‪⃗⃗⃗⃗⃗⃗ ‖ = √22 + 22 + (−3)2 = √17 ; 𝑟 + 𝑟 ′ = 7‬‬
‫𝐺𝜔‖ = )𝐺 ;𝜔(𝑑‬
‫أن ‪ّ ، 𝑑(𝜔; 𝐺) < 𝑟 + 𝑟′‬‬
‫فإن )𝑆( و)‪ (𝑆′‬متقاطعتان‬ ‫بما ّ‬
‫‪ .21‬بيّن أن مجموعة النقط ) 𝑧 ; 𝑦 ; 𝑥(𝑀 من الفضاء التي تحقق ‪(𝑥 + 2𝑦 − 𝑧 + 3) + (3𝑥 + 𝑦 + 2𝑧 − 1) = 0‬‬
‫‪2‬‬ ‫‪2‬‬

‫هي مستقيم )𝐷( يطلب تعيين شعاع توجيه له‬


‫المعادلة 𝟎 = 𝟐𝒚 ‪ 𝒙𝟐 +‬تكافئ 𝟎 = 𝒙 و 𝟎 = 𝒚‬
‫‪𝑥 + 2𝑦 − 𝑧 + 3 = 0‬‬ ‫‪2𝑥 + 4𝑦 − 2𝑧 + 6 = 0 … ‬‬
‫{ ⇒ ‪(𝑥 + 2𝑦 − 𝑧 + 3)2 + (3𝑥 + 𝑦 + 2𝑧 − 1)2 = 0‬‬ ‫{⇒‬
‫‪3𝑥 + 𝑦 + 2𝑧 − 1 = 0‬‬ ‫‪3𝑥 + 𝑦 + 2𝑧 − 1 = 0 … ‬‬
‫‪5𝑥 + 5𝑦 + 5 = 0‬‬ ‫‪𝑥 = −𝑦 − 1‬‬ ‫‪𝑥 = −𝑦 − 1‬‬ ‫‪𝑥 = −𝑦 − 1‬‬
‫{ ⇒‬ ‫{⇒‬ ‫{⇒‬ ‫{⇒‬
‫‪+ 3𝑥 + 𝑦 + 2𝑧 − 1 = 0‬‬ ‫‪3(−𝑦 − 1) + 𝑦 + 2𝑧 − 1 = 0‬‬ ‫‪2𝑧 = 2𝑦 + 4‬‬ ‫‪𝑧 =𝑦+2‬‬
‫أن مجموعة النقط 𝑀 من الفضاء التي تحقق ‪(𝑥 + 2𝑦 − 𝑧 + 3) + (3𝑥 + 𝑦 + 2𝑧 − 1) = 0‬‬
‫‪2‬‬ ‫‪2‬‬ ‫بوضع ‪ ، 𝑦 = 𝑡 :‬نستنتج ّ‬
‫‪−1‬‬ ‫𝑡 ‪𝑥 = −1 −‬‬
‫𝑢‬ ‫{ والموجّه بالشعاع ) ‪(𝐷) ( 1‬‬
‫⃗⃗⃗⃗⃗⃗⃗⃗‬ ‫المعرف بتمثيله الوسيطي ‪𝑦 = 𝑡 ; 𝑡 ∈ ℝ :‬‬
‫ّ‬ ‫هي المستقيم )𝐷(‬
‫‪1‬‬ ‫𝑡‪𝑧 =2+‬‬
‫‪ .21‬بين أن مجموعة النقط )𝑧 ; 𝑦 ; 𝑥(𝑀 من الفضاء التي تحقق ‪(𝑥 + 2𝑦 − 𝑧 + 2)2 − (3𝑥 + 𝑦 + 2𝑧 − 1)2 = 0‬‬
‫هي اتحاد مستويين )‪ (𝑃′‬و )‪ (𝑄′‬يطلب إعطاء معادلتين ديكارتيتين لهما‬
‫‪(𝑥 + 2𝑦 − 𝑧 + 2) − (3𝑥 + 𝑦 + 2𝑧 − 1)2 = 0‬‬
‫‪2‬‬

‫‪⇒ [(𝑥 + 2𝑦 − 𝑧 + 2) − (3𝑥 + 𝑦 + 2𝑧 − 1)][(𝑥 + 2𝑦 − 𝑧 + 2) + (3𝑥 + 𝑦 + 2𝑧 − 1)] = 0‬‬


‫‪⇒ (−2𝑥 + 𝑦 − 3𝑧 + 3)(4𝑥 + 3𝑦 + 𝑧 + 1) = 0‬‬
‫) ‪ 4𝑥 + 3𝑦 + 𝑧 + 1 = 0 … (𝑄 ′ ) ⇒ 𝑀 ∈ (𝑃′ ) ∪ (𝑄 ′‬أو ) ‪⇒ −2𝑥 + 𝑦 − 3𝑧 + 3 = 0 … (𝑃′‬‬
‫المعرفة بالمعادلة ‪ (2 − 𝑚)𝑥 + 𝑦 + 𝑚𝑧 + 6𝑚 − 6 = 0 :‬حيث ‪𝑚 ∈ ℝ‬‬ ‫ّ‬ ‫‪ .22‬لتكن المستويات ) 𝑚𝜋(‬
‫أ‪ .‬بيّن ّ‬
‫أن المستويات ) 𝑚𝜋( تشمل مستقيما ثابتا )‪ (𝐷′‬يُطلب تعيين معادلته الديكارتية وتمثيله الوسيطي‬
‫العبارة ‪ :‬من أجل كل ‪ 𝒂𝒎 + 𝒃 = 𝟎 : 𝒎 ∈ ℝ‬تكافئ 𝟎 = 𝒂 و 𝟎 = 𝒃‬
‫‪∀𝑚 ∈ ℝ ∶ (2 − 𝑚)𝑥 + 𝑦 + 𝑚𝑧 + 6𝑚 − 6 = 0‬‬
‫‪−𝑥 + 𝑧 + 6 = 0‬‬
‫{ ‪∀𝑚 ∈ ℝ ∶ 𝑚(−𝑥 + 𝑧 + 6) + (2𝑥 + 𝑦 − 6) = 0 ⇒ (𝐷′):‬‬
‫‪2𝑥 + 𝑦 − 6 = 0‬‬

‫‪11‬‬
‫‪−𝑥 + 𝑧 + 6 = 0‬‬ ‫‪𝑥 =𝑧+6‬‬ ‫‪𝑥 =𝑧+6‬‬ ‫𝑡‪𝑥 =6+‬‬
‫{ ⇒ )‪𝑀 ∈ (𝐷′‬‬ ‫{⇒‬ ‫{⇒‬ ‫‪⇒ (𝐷′): {𝑦 = −6 − 2𝑡 ; 𝑡 ∈ ℝ‬‬
‫‪2𝑥 + 𝑦 − 6 = 0‬‬ ‫‪𝑦 = −2𝑥 + 6‬‬ ‫‪𝑦 = −2𝑧 − 6‬‬
‫𝑡=𝑧‬
‫ب‪ .‬بيّن ّ‬
‫أن المستويات ) 𝑚𝜋( تقطع المستوي )𝑧𝑂𝑦( وفق مستقيم يُطلب تعيين تمثيله الوسيطي‬
‫𝟎 = 𝒛 ∶ )𝒚𝑶𝒙( ; 𝟎 = 𝒚 ∶ )𝒛𝑶𝒙( ; 𝟎 = 𝒙 ∶ )𝒛𝑶𝒚(‬
‫‪(2 − 𝑚)𝑥 + 𝑦 + 𝑚𝑧 + 6𝑚 − 6 = 0‬‬ ‫‪𝑦 + 𝑚𝑧 + 6𝑚 − 6 = 0‬‬
‫{ ⇒ )𝑧𝑂𝑦( ∩ ) 𝑚𝜋( ∈ )𝑧 ;𝑦 ;𝑥(𝑀‬ ‫{⇒‬
‫‪𝑥=0‬‬ ‫‪𝑥=0‬‬
‫‪𝑥=0‬‬
‫{ ⇒‬ ‫𝑦‬ ‫=‬ ‫‪6‬‬ ‫‪− 6𝑚 − 𝑚𝑡′ ; 𝑡′ ∈ ℝ‬‬
‫‪𝑧 = 𝑡′‬‬
‫ج‪ .‬اكتب معادلة سطح الكرة )"𝑆( التي مركزها )‪ ω"(2; 1; 2‬ونصف قطرها ‪3‬‬
‫‪𝑀(𝑥; 𝑦; 𝑧) ∈ (𝑆") ⇒ (𝑥 − 2)2 + (𝑦 − 1)2 + (𝑧 − 2)2 = 32 ⇒ 𝑥 2 + 𝑦 2 + 𝑧 2 − 4𝑥 − 2𝑦 − 4𝑧 = 0‬‬
‫د‪ .‬ناقش حسب قيم 𝑚 الوضع النسبي للمستويات ) 𝑚𝜋( بالنسبة إلى )"𝑆( ث ّم استنتج المستويات المماسية لـ )"𝑆(‬
‫|‪|2(2 − 𝑚) + 1 + 2𝑚 + 6𝑚 − 6‬‬ ‫|‪|6𝑚 − 1‬‬
‫= ]) 𝑚𝜋( ‪𝑑[𝜔",‬‬ ‫=‬ ‫)‪; (2𝑚2 − 4𝑚 + 5 > 0‬‬
‫‪2‬‬ ‫‪2‬‬ ‫‪2‬‬
‫‪√2𝑚 − 4𝑚 + 5‬‬
‫𝑚 ‪√(2 − 𝑚) + 1 +‬‬
‫‪(6𝑚 − 1)2‬‬
‫⇒ ‪𝑑[𝜔", (𝜋𝑚 )] = 3 ⇒ 𝑑2 [𝜔", (𝜋𝑚 )] = 9‬‬ ‫‪= 9 ⇒ 9(2𝑚2 − 4𝑚 + 5) = (6𝑚 − 1)2‬‬
‫‪2𝑚2 − 4𝑚 + 5‬‬
‫‪−2 − √26‬‬ ‫‪−2 + √26‬‬
‫= ‪⇒ 18𝑚2 + 24𝑚 − 44 = 0 ; 𝑚′‬‬ ‫= ‪; 𝑚′′‬‬
‫‪3‬‬ ‫‪3‬‬
‫‪−2−√26‬‬ ‫‪−2+√26‬‬
‫‪‬‬ ‫; ∞‪𝑚 ∈ ]−‬‬ ‫]∪[‬ ‫المستويات ) 𝑚𝜋( منفصلة عن )"𝑆( ⇒ ‪; +∞[ : 𝑑[𝜔", (𝜋𝑚 )] > 3‬‬
‫‪3‬‬ ‫‪3‬‬
‫‪−2−√26‬‬ ‫‪−2+√26‬‬
‫‪‬‬ ‫]∈𝑚‬ ‫;‬ ‫المستويات ) 𝑚𝜋( متقاطعة مع )"𝑆( ⇒ ‪[ : 𝑑[𝜔", (𝜋𝑚 )] < 3‬‬
‫‪3‬‬ ‫‪3‬‬
‫‪−2−√26 −2+√26‬‬
‫‪‬‬ ‫{∈𝑚‬ ‫;‬ ‫المستويات ) 𝑚𝜋( مماسية لـ )"𝑆( ⇒ ‪} : 𝑑[𝜔", (𝜋𝑚 )] = 3‬‬
‫‪3‬‬ ‫‪3‬‬

‫𝑡‪𝑥 = 1 + 2‬‬
‫‪ .23‬نعتبر النقطة )‪ 𝐴(−1; 1; 3‬والمستقيم )∆( الممثل وسيطيا بالجملة ‪{ 𝑦 = 2 − 𝑡 ; 𝑡 ∈ ℝ :‬‬
‫𝑡‪𝑧 = 2 + 2‬‬
‫أ‪ .‬لتكن 𝑡𝑀 نقطة كيفية من المستقيم )∆(‪ .‬عبّر عن ‪ 𝐴𝑀𝑡2‬بداللة 𝑡 ‪ ،‬ث ّم احسب المسافة 𝑑‬
‫𝑡‪2 + 2‬‬
‫⃗⃗⃗⃗⃗⃗⃗⃗‬
‫‪𝐴𝑀𝑡 ( 1 − 𝑡 ) ⇒ 𝐴𝑀𝑡2 = (2 + 2𝑡)2 + (1 − 𝑡)2 + (−1 + 2𝑡)2 = 9𝑡 2 + 2𝑡 + 6‬‬
‫𝑡‪−1 + 2‬‬
‫حساب المسافة 𝒅‬
‫‪𝐴𝑀𝑡 = √9𝑡 2 + 2𝑡 + 6‬‬
‫المعرفة على ‪ ℝ‬بـ ‪ .𝑓(𝑡) = √9𝑡 2 + 2𝑡 + 6 :‬لدينا ‪:‬‬
‫ّ‬ ‫لتكن الدالة 𝑓‬
‫𝑡‪18‬‬ ‫‪+‬‬ ‫‪2‬‬ ‫𝑡‪9‬‬ ‫‪+‬‬ ‫‪1‬‬ ‫‪1‬‬
‫= )𝑡( ‪𝑓 ′‬‬ ‫=‬ ‫‪; 𝑓 ′ (𝑡) = 0 ⇒ 9𝑡 + 1 = 0 ⇒ 𝑡 = −‬‬
‫‪2√9𝑡 2 + 2𝑡 + 6 √9𝑡 2 + 2𝑡 + 6‬‬ ‫‪9‬‬
‫‪1‬‬
‫المسافة 𝑑 هي القيمة الحدّية الصغرى للدالة )𝑡(𝑓 ‪ ،‬أي )‪ ، 𝑓 (− 9‬منه ‪:‬‬

‫‪1‬‬ ‫‪1 2‬‬ ‫‪1‬‬ ‫‪1 2 54‬‬ ‫‪53‬‬ ‫‪√53‬‬


‫√‬
‫‪𝑑 = 𝑓 (− ) = 9 (− ) + 2 (− ) + 6 = √ − +‬‬ ‫=𝑑 ⇒ √=‬
‫‪9‬‬ ‫‪9‬‬ ‫‪9‬‬ ‫‪9 9 9‬‬ ‫‪9‬‬ ‫‪3‬‬
‫ب‪ .‬عيّن إحداثيات المسقط العمودي 𝐻 للنقطة 𝐴 على المستقيم )∆( ‪ ،‬ث ّم احسب المسافة 𝑑 مرة ثانية‬
‫𝑡‪2 + 2‬‬
‫⃗⃗⃗⃗⃗⃗ ⇒ )𝑡‪𝐻 ∈ (∆) ⇒ 𝐻(1 + 2𝑡; 2 − 𝑡; 2 + 2‬‬ ‫) 𝑡 ‪𝐴𝐻 ( 1 −‬‬
‫𝑡‪−1 + 2‬‬

‫‪10‬‬
‫‪1‬‬ ‫‪7 19 16‬‬
‫⃗⃗⃗⃗⃗⃗‬ ‫⃗⃗⃗⃗⃗⃗ ‪𝑢(∆) .‬‬
‫⃗⃗⃗⃗⃗⃗⃗ ⇒ )∆( ⊥ 𝐻𝐴‬ ‫) ; ; ( 𝐻 ⇒ ‪𝐴𝐻 = 0 ⇒ 2(2 + 2𝑡) − (1 − 𝑡) + 2(−1 + 2𝑡) = 0 ⇒ 𝑡 = −‬‬
‫‪9‬‬ ‫‪9 9 9‬‬
‫حساب المسافة 𝒅‬
‫‪2‬‬ ‫‪2‬‬
‫‪7‬‬ ‫‪2‬‬
‫‪19‬‬ ‫‪16‬‬ ‫‪16 2‬‬ ‫‪10 2‬‬ ‫‪11 2‬‬ ‫‪√53‬‬
‫√‬ ‫√‬
‫= 𝑑 ⇒ ) ‪𝑑 = 𝐴𝐻 = ( + 1) + ( − 1) + ( − 3) = ( ) + ( ) + (−‬‬
‫‪9‬‬ ‫‪9‬‬ ‫‪9‬‬ ‫‪9‬‬ ‫‪9‬‬ ‫‪9‬‬ ‫‪3‬‬
‫ج‪ .‬اكتب معادلة ديكارتية للمستوي )𝑃( الذي يشمل النقطة 𝐴 ويعامد )∆(‬
‫‪2‬‬
‫ليكن ⃗𝑛 شعاعا ناظميا للمستوي )𝑃(‪ .‬لدينا ‪𝑢(∆) ⇒ 𝑛⃗ (−1) :‬‬
‫⃗⃗⃗⃗⃗⃗⃗ = ⃗𝑛‬
‫‪2‬‬
‫⃗⃗⃗⃗⃗⃗‬
‫‪𝑀(𝑥; 𝑦; 𝑧) ∈ (𝑃) ⇒ 𝑛⃗. 𝐴𝑀 = 0 ⇒ 2(𝑥 + 1) − (𝑦 − 1) + 2(𝑧 − 3) = 0 ⇒ 2𝑥 − 𝑦 + 2𝑧 − 3 = 0‬‬
‫‪ ‬بيّن ّ‬
‫أن النقطة ‪ 𝑀0‬تنتمي إلى )∆( ‪ ،‬ث ّم احسب ‪𝐴𝑀02‬‬
‫)‪ 𝑀0 (1; 2; 2‬هي النقطة من )∆( من أجل ‪𝑡 = 0‬‬
‫‪2‬‬
‫⃗⃗⃗⃗⃗⃗⃗⃗‬
‫‪𝐴𝑀0 ( 1 ) ⇒ 𝐴𝑀02 = (2)2 + (1)2 + (−1)2 = 6‬‬
‫‪−1‬‬
‫‪ ‬استنتج المسافة 𝑑 مرة ثالثة‬
‫نسمي ‪ 𝑑1‬المسافة ‪ 𝐴𝑀0‬و ‪ 𝑑2‬المسافة بين النقطة ‪ 𝑀0‬والمستوي )𝑃(‪ .‬لدينا ‪:‬‬
‫‪|2(1) − 2 + 2(2) − 3| 1‬‬
‫= ‪𝑑2‬‬ ‫=‬
‫‪√9‬‬ ‫‪3‬‬
‫‪1 53‬‬ ‫‪√53‬‬ ‫‪𝑑1‬‬
‫‪𝑑2‬‬
‫‪𝑑 2 = 𝑑1 2 − 𝑑2 2 = 6 −‬‬ ‫=‬ ‫=𝑑 ⇒‬
‫‪9‬‬ ‫‪9‬‬ ‫‪3‬‬ ‫𝑑‬ ‫)𝑃(‬

‫)∆(‬

‫‪ .24‬من أجل كل عدد حقيقي ‪ α‬من المجال ]𝜋 ;𝜋‪ ، ]−‬نعتبر ) 𝛼𝑆( مجموعة النقط )𝑧 ;𝑦 ;𝑥(𝑀 من الفضاء التي إحداثياتها‬
‫تحقق العالقة ‪𝑥 2 + 𝑦 2 + 𝑧 2 − 2𝑥 cos 𝛼 − 2𝑦 sin 𝛼 + 2𝑧 = 0 :‬‬
‫أن ) 𝛼𝑆( سطح كرة يُطلب تعيين مركزها 𝛼𝜔 ونصف قطرها 𝑟‬ ‫أ‪ .‬بيّن ّ‬
‫‪𝑥 2 + 𝑦 2 + 𝑧 2 − 2𝑥 cos 𝛼 − 2𝑦 sin 𝛼 + 2𝑧 = 0‬‬
‫‪⇒ (𝑥 − cos 𝛼)2 − cos 2 𝛼 + (𝑦 − sin 𝛼)2 − sin2 𝛼 + (𝑧 + 1)2 − 1 = 0‬‬
‫‪⇒ (𝑥 − cos 𝛼)2 + (𝑦 − sin 𝛼)2 + (𝑧 + 1)2 = 2 ⇒ 𝜔𝛼 (cos 𝛼 ; sin 𝛼 ; −1) ; 𝑟 = √2‬‬

‫ب‪ .‬عيّن حسب قيم ‪ α‬تقاطع سطح الكرة ) 𝛼𝑆( والمستوي )"𝑃( ذي المعادلة ‪𝑦 − 𝑧 + 2 = 0 :‬‬
‫‪|sin 𝛼 + 3| √2‬‬
‫= ])"𝑃( ; 𝛼𝜔[𝑑‬ ‫=‬ ‫)‪(sin 𝛼 + 3‬‬
‫‪√2‬‬ ‫‪2‬‬
‫𝜋‪3‬‬
‫‪√2‬‬ ‫𝜋‪3‬‬ ‫𝛼‬ ‫=‬ ‫𝜋𝑘‪+ 2‬‬ ‫𝜋‬
‫‪(sin 𝛼 + 3) = √2 ⇒ sin 𝛼 = −1 ⇒ sin 𝛼 = sin‬‬ ‫{⇒‬ ‫‪2‬‬ ‫‪⇒ 𝛼=−‬‬
‫‪2‬‬ ‫‪2‬‬ ‫𝜋‪3‬‬ ‫‪2‬‬
‫𝜋𝑘‪𝛼 = (𝜋 − ) + 2‬‬
‫‪2‬‬
‫𝜋‬
‫) 𝛼𝑆( و)"𝑃( متماسان ⇒ 𝑟 = ])"𝑃( ; 𝛼𝜔[𝑑 ⇒ ‪ 𝛼 = − ⇒ sin 𝛼 = −1‬‬
‫‪2‬‬
‫𝜋‬
‫‪‬‬ ‫) 𝛼𝑆( و)"𝑃( منفصالن ⇒ 𝑟 > ])"𝑃( ; 𝛼𝜔[𝑑 ⇒ ‪𝛼 ∈ ]−𝜋; 𝜋] − 2 ⇒ 𝑠𝑖𝑛 𝛼 > −1‬‬

‫‪‬‬
‫‪‬‬

‫‪12‬‬
‫الشكل الجبري لعدد مر ّكب ‪𝒛 = 𝒙 + 𝒊𝒚 :‬‬
‫‪) ; 𝑖 2 = −1‬الجزء التخيّلي(𝑦 = )𝑧(𝑚𝐼 ; )الجزء الحقيقي(𝑥 = )𝑧(𝑒𝑅‬
‫‪𝑥=0‬‬ ‫‪𝑥 = 𝑥′‬‬
‫{⇒‪𝑧=0‬‬ ‫{ ⇒ ‪; 𝑧 ′ = 𝑥 ′ + 𝑖𝑦 ′ ; 𝑧 = 𝑧′‬‬
‫‪𝑦=0‬‬ ‫‪𝑦 = 𝑦′‬‬
‫مرافق عدد مركّب ‪𝒛̅ = 𝒙 − 𝒊𝒚 :‬‬
‫‪𝑧 + 𝑧̅ = 2𝑥 ; 𝑧. 𝑧̅ = 𝑥 2 + 𝑦 2‬‬
‫𝑧 تخيّلي صرف ⇒ 𝑧‪ 𝑧 ; 𝑧̅ = −‬حقيقي ⇒ 𝑧 = ̅𝑧‬
‫طويلة عدد مركّب ‪𝒓 = |𝒛| = √𝒙𝟐 + 𝒚𝟐 :‬‬
‫𝑧‬ ‫|𝑧|‬
‫= | ‪|𝑧̅| = |𝑧| ; |𝑧|2 = 𝑧. 𝑧̅ ; |𝑧. 𝑧′| = |𝑧| × |𝑧′| ; |𝑧 𝑛 | = |𝑧|𝑛 ; | ′‬‬
‫𝑧‬ ‫|‪|𝑧′‬‬
‫عمدة عدد مركّب ‪𝒂𝒓𝒈(𝒛) = 𝜽 + 𝟐𝒌𝝅 :‬‬
‫𝑥‬ ‫𝑦‬
‫= 𝜃 𝑛𝑖𝑠 ; = 𝜃 𝑠𝑜𝑐‬
‫𝑟‬ ‫𝑟‬
‫𝑧‬
‫)‪𝑎𝑟𝑔(𝑧̅) = −𝑎𝑟𝑔(𝑧) ; 𝑎𝑟𝑔(𝑧. 𝑧 ′ ) = 𝑎𝑟𝑔(𝑧) + 𝑎𝑟𝑔(𝑧 ′ ) ; 𝑎𝑟𝑔 ( ′ ) = 𝑎𝑟𝑔(𝑧) − 𝑎𝑟𝑔(𝑧′‬‬
‫𝑧‬
‫)𝑧(𝑔𝑟𝑎 ‪𝑎𝑟𝑔(𝑧 𝑛 ) = 𝑛.‬‬
‫𝜋‬
‫𝑧 تخيّلي صرف ⇒ 𝜋𝑘 ‪ 𝑧 ; 𝑎𝑟𝑔(𝑧) = +‬حقيقي ⇒ 𝜋𝑘 = )𝑧(𝑔𝑟𝑎‬
‫‪2‬‬
‫الشكل المثلّثي لعدد مركّب ‪𝒛 = 𝒓(𝒄𝒐𝒔 𝜽 + 𝒊 𝒔𝒊𝒏 𝜽) :‬‬
‫𝑥‬ ‫𝑦‬
‫)𝜃𝑛( 𝑛𝑖𝑠 𝑖 ‪𝑧 = 𝑥 + 𝑖𝑦 = 𝑟 ( + 𝑖 ) = 𝑟(𝑐𝑜𝑠 𝜃 + 𝑖 𝑠𝑖𝑛 𝜃) ; (𝑐𝑜𝑠 𝜃 + 𝑖 𝑠𝑖𝑛 𝜃)𝑛 = 𝑐𝑜𝑠 (𝑛𝜃) +‬‬
‫𝑟‬ ‫𝑟‬
‫سي لعدد مركّب ‪𝒛 = 𝒓𝒆𝒊𝜽 :‬‬
‫الشكل األ ّ‬
‫‪′‬‬ ‫𝑧‬ ‫𝑟‬ ‫‪′‬‬ ‫𝑛‬
‫𝜃𝑛𝑖 𝑒 ‪𝑧̅ = 𝑟𝑒 −𝑖𝜃 ; 𝑧. 𝑧 ′ = 𝑟. 𝑟 ′ 𝑒 𝑖(𝜃+𝜃 ) ; ′ = ′ 𝑒 𝑖(𝜃−𝜃 ) ; (𝑟𝑒 𝑖𝜃 ) = 𝑟 𝑛 .‬‬
‫𝑧‬ ‫𝑟‬
‫𝜋‬
‫𝑟‪𝑟𝑒 𝑖 2 = 𝑖 𝑟 ; 𝑟𝑒 𝑖𝜋 = −‬‬
‫التفسير الهندسي لألعداد المركّبة ‪:‬‬
‫𝐵𝑧 ‪𝑧𝐴 +‬‬
‫⃗⃗⃗⃗⃗‬
‫= 𝐼𝑧 ; | 𝐴𝑧 ‪⃗⃗⃗⃗⃗ = 𝑧𝐵 − 𝑧𝐴 ; ‖𝐴𝐵 ‖ = |𝑧𝐵 −‬‬
‫𝐵𝐴𝑧‬ ‫)𝐼 منتصف ]𝐵𝐴[(‬
‫‪2‬‬
‫𝐶𝑧𝛾 ‪𝛼𝑧𝐴 + 𝛽𝑧𝐵 +‬‬
‫= 𝐺𝑧‬ ‫)}𝛾 ;𝐶( ‪(𝐺{(𝐴; 𝛼), (𝐵; 𝛽),‬‬
‫𝛾‪𝛼+𝛽+‬‬
‫𝐴𝑧 ‪𝑧𝐶 −‬‬ ‫𝐶𝐴‬ ‫𝐴𝑧 ‪𝑧𝐶 −‬‬
‫|‬ ‫=|‬ ‫( ‪; arg‬‬ ‫𝐵𝐴( = )‬‫⃗⃗⃗⃗⃗ ; ⃗⃗⃗⃗⃗‬
‫) 𝐶𝐴‬
‫𝐴𝑧 ‪𝑧𝐵 −‬‬ ‫𝐵𝐴‬ ‫𝐴𝑧 ‪𝑧𝐵 −‬‬
‫𝐴𝑧 ‪𝑧𝐶 −‬‬ ‫𝐴𝑧 ‪𝑧𝐶 −‬‬
‫; 𝐴 ‪ 𝐶, 𝐵,‬على استقامة واحدة ⇒ حقيقي‬ ‫⃗⃗⃗⃗⃗ ⇒ تخيّلي صرف‬ ‫⃗⃗⃗⃗⃗ ⊥ 𝐵𝐴‬
‫𝐶𝐴‬
‫𝐴𝑧 ‪𝑧𝐵 −‬‬ ‫𝐴𝑧 ‪𝑧𝐵 −‬‬
‫مثال ‪: 1‬‬
‫𝐴 ‪ 𝐶, 𝐵,‬ثالث نقط من المستوي لواحقها على الترتيب ‪ 𝑧𝐵 = 2𝑖 ، 𝑧𝐴 = 1 :‬و 𝑖 ‪𝑧𝐶 = −1 −‬‬
‫𝑧– 𝑧‬
‫عيّن طويلة وعمدة العدد المر ّكب 𝐴𝑧‪ ، 𝑧 𝐵−‬ث ّم استنتج طبيعة المثلث 𝐶𝐵𝐴‪.‬‬
‫𝐶‬ ‫𝐴‬
‫𝐴𝑧 – 𝐵𝑧‬ ‫)𝑖 ‪2𝑖 − 1 (−1 + 2𝑖)(−2 +‬‬ ‫𝑖‪5‬‬
‫=‬ ‫=‬ ‫𝑖‪= − = −‬‬
‫𝑖 ‪𝑧𝐶 − 𝑧𝐴 −2 −‬‬ ‫)𝑖 ‪(−2 − 𝑖)(−2 +‬‬ ‫‪5‬‬

‫‪11‬‬
‫𝐴𝑧 – 𝐵𝑧‬
‫|‬ ‫‪|=1‬‬ ‫𝐶𝐴 = 𝐵𝐴‬
‫𝐴𝑧 ‪𝑧𝐶 −‬‬ ‫𝜋‬
‫{‬ ‫𝐴𝑧 – 𝐵𝑧‬ ‫𝜋‬ ‫⇒‬ ‫{‬ ‫المثلث 𝐶𝐵𝐴 متساوي الساقين و قائم في 𝐴 ⇒ ‪⃗⃗⃗⃗⃗ ) = −‬‬
‫𝐵𝐴 ; ⃗⃗⃗⃗⃗‬
‫𝐶𝐴(‬
‫( ‪arg‬‬ ‫‪)=−‬‬ ‫‪2‬‬
‫𝐴𝑧 ‪𝑧𝐶 −‬‬ ‫‪2‬‬
‫مثال ‪: 2‬‬
‫𝐴 ‪ 𝐸, 𝐷, 𝐶, 𝐵,‬نقط من المستوي لواحقها على الترتيب ‪:‬‬
‫𝑖‪𝑧𝐴 = √3𝑖 ; 𝑧𝐵 = −√3𝑖 ; 𝑧𝐶 = 3 + 2√3𝑖 ; 𝑧𝐷 = 3 − 2√3𝑖 ; 𝑧𝐸 = −3 + 2√3‬‬
‫مثّل النقط ‪: E ، D ، C ، B ، A‬‬

‫اثبت أنّ النقط ‪ D ، C ، B ، A‬تنتمي إلى دائرة يطلب تعيين مركزها ونصف قطرها‬
‫𝐴𝑧 ‪𝑧𝐶 −‬‬ ‫𝑖‪3 + √3‬‬ ‫‪√3‬‬ ‫𝜋 𝑖 ‪√3‬‬
‫=‬ ‫=‬ ‫=𝑖‬ ‫المثلث 𝐷𝐶𝐴 قائم في 𝐴 ⇒ ‪𝑒 2‬‬
‫𝑖‪𝑧𝐷 − 𝑧𝐴 3 − 3√3‬‬ ‫‪3‬‬ ‫‪3‬‬
‫𝑖‪𝑧𝐶 − 𝑧𝐵 3 + 3√3‬‬ ‫𝜋‬
‫=‬ ‫المثلث 𝐷𝐶𝐵 قائم في 𝐵 ⇒ ‪= √3𝑖 = √3𝑒 𝑖 2‬‬
‫𝐵𝑧 ‪𝑧𝐷 −‬‬ ‫𝑖‪3 − √3‬‬
‫بما أ ّن المثلثين 𝐷𝐶𝐴 و 𝐷𝐶𝐵 قائمان ولهما نفس الوتر )𝐷𝐶( ‪ ،‬نستنتج أ ّن النقط 𝐴 ‪ 𝐷, 𝐶, 𝐵,‬تنتمي إلى الدائرة التي‬
‫‪1‬‬
‫مركزها 𝜔 منتصف ]𝐷𝐶[ ونصف قطرها 𝐷𝐶 ‪.𝑟 = 2‬‬
‫𝑖‪𝑧𝐶 + 𝑧𝐷 3 + 2√3𝑖 + 3 − 2√3‬‬
‫= 𝜔𝑧‬ ‫=‬ ‫)‪= 3 ⇒ 𝜔(3; 0‬‬
‫‪2‬‬ ‫‪2‬‬
‫‪1‬‬ ‫‪1‬‬ ‫‪1‬‬ ‫‪1‬‬ ‫‪1‬‬
‫‪𝑟 = 𝐶𝐷 = |𝑧𝐷 − 𝑧𝑐 | = |3 − 2√3𝑖 − 3 − 2√3𝑖| = |−4√3𝑖| = (4√3) ⇒ 𝑟 = 2√3‬‬
‫‪2‬‬ ‫‪2‬‬ ‫‪2‬‬ ‫‪2‬‬ ‫‪2‬‬
‫𝝅‬
‫𝒛‪𝒛 −‬‬
‫بيّن أنّ 𝟑𝒊‪ ، 𝒛𝑪 −𝒛𝑩 = 𝒆−‬ث ّم استنتج طبيعة المثلث ‪BEC‬‬
‫𝑬‬ ‫𝑩‬

‫𝐵𝑧 ‪𝑧𝐶 −‬‬ ‫𝑖‪3 + 3√3‬‬ ‫𝑖‪1 + √3‬‬ ‫‪1 √3‬‬ ‫𝐵𝑧 ‪𝑧𝐶 −‬‬ ‫𝜋‬
‫=‬ ‫=‬ ‫‪= −‬‬ ‫⇒𝑖‬ ‫=‬ ‫‪𝑒 −𝑖 3‬‬
‫‪𝑧𝐸 − 𝑧𝐵 −3 + 3√3𝑖 −1 + √3𝑖 2‬‬ ‫‪2‬‬ ‫𝐵𝑧 ‪𝑧𝐸 −‬‬
‫𝐵𝑧 ‪𝑧𝐶 −‬‬
‫|‬ ‫‪|=1‬‬ ‫𝐶𝐵 = 𝐸𝐵‬
‫𝐵𝑧 ‪𝑧𝐶 −‬‬ ‫𝑖‪−‬‬
‫𝜋‬ ‫𝐵𝑧 ‪𝑧𝐸 −‬‬ ‫𝜋‬
‫𝐵𝑧 ‪𝑧𝐸 −‬‬
‫{⇒ ‪=𝑒 3‬‬ ‫𝐵𝑧 ‪𝑧𝐶 −‬‬ ‫𝜋‬ ‫⇒‬ ‫{‬ ‫المثلث 𝐶𝐸𝐵 متقايس األضالع ⇒ ‪⃗⃗⃗⃗⃗ ) = −‬‬
‫𝐶𝐵 ; ⃗⃗⃗⃗⃗‬
‫𝐸𝐵(‬
‫( ‪arg‬‬ ‫‪)=−‬‬ ‫‪3‬‬
‫𝐵𝑧 ‪𝑧𝐸 −‬‬ ‫‪3‬‬

‫‪11‬‬
‫مثال ‪: 3‬‬
‫‪𝑧̅ +2‬‬
‫= 𝐿 و ‪ z‬عدد مركب حيث ‪ .z ≠ -2‬و لتكن ‪ M‬صورة العدد المركب ‪ z‬في المستوي المنسوب إلى‬ ‫ليكن ‪ L‬عدد مركب حيث ‪:‬‬
‫‪𝑧+2‬‬
‫𝑢 ‪.(𝑜,‬‬‫معلم متعامد و متجانس ) 𝑣 ‪⃗ ,‬‬
‫‪ .1‬عيّن الجزء الحقيقي و التخيلي للعدد المركب ‪L‬‬
‫)‪𝑧̅ + 2 𝑥 − 𝑖𝑦 + 2 (𝑥 + 2 − 𝑖𝑦)(𝑥 + 2 − 𝑖𝑦) (𝑥 + 2 − 𝑖𝑦)2 (𝑥 + 2)2 − 𝑦 2 − 2𝑖𝑦(𝑥 + 2‬‬
‫=𝐿‬ ‫=‬ ‫=‬ ‫=‬ ‫=‬
‫‪𝑧 + 2 𝑥 + 𝑖𝑦 + 2 (𝑥 + 2 + 𝑖𝑦)(𝑥 + 2 − 𝑖𝑦) (𝑥 + 2)2 + 𝑦 2‬‬ ‫‪(𝑥 + 2)2 + 𝑦 2‬‬
‫‪(𝑥 + 2)2 − 𝑦 2‬‬ ‫)‪2𝑦(𝑥 + 2‬‬
‫=𝐿‬ ‫‪−‬‬ ‫𝑖‬
‫‪(𝑥 + 2)2 + 𝑦 2 (𝑥 + 2)2 + 𝑦 2‬‬
‫‪ .2‬عيّن مجموعة النقط ‪ M‬من المستوي بحيث يكون ‪ L‬حقيقيا‬
‫‪2𝑦(𝑥 + 2) = 0‬‬ ‫)∆( … ‪𝑦 = 0‬‬ ‫) ‪𝑥 = −2 … (∆′‬‬
‫{ ⇒ 𝐿 حقيقي‬ ‫⇒‬ ‫{‬ ‫أو‬ ‫{‬
‫‪(𝑥 + 2)2 + 𝑦 2 ≠ 0‬‬ ‫)‪(𝑥; 𝑦) ≠ (−2; 0‬‬ ‫)‪(𝑥; 𝑦) ≠ (−2; 0‬‬
‫})‪⇒ 𝑀 ∈ (∆) ∪ (∆′) − {(−2; 0‬‬
‫‪ .3‬عيّن مجموعة النقط ‪ M‬من المستوي بحيث يكون ‪ L‬تخيليا صرفا‬
‫‪(𝑥 + 2) − 𝑦 = 0‬‬ ‫‪2‬‬ ‫‪2‬‬
‫‪𝑦 2 = (𝑥 + 2)2‬‬ ‫)𝐷( … ‪𝑦 = 𝑥 + 2‬‬ ‫) ‪𝑦 = −𝑥 − 2 … (𝐷′‬‬
‫{ ⇒ 𝐿 تخيلي صرف‬ ‫⇒‬ ‫{‬ ‫⇒‬ ‫{‬ ‫أو‬ ‫{‬
‫‪(𝑥 + 2)2 + 𝑦 2 ≠ 0‬‬ ‫)‪(𝑥; 𝑦) ≠ (−2; 0‬‬ ‫)‪(𝑥; 𝑦) ≠ (−2; 0‬‬ ‫)‪(𝑥; 𝑦) ≠ (−2; 0‬‬
‫})‪⇒ 𝑀 ∈ (𝐷) ∪ (𝐷′) − {(−2; 0‬‬

‫التحويالت النقطية ‪:‬‬


‫𝑼⃗ ; ) 𝟎𝒛(𝝎 ; ) ‪𝑴(𝒛) ; 𝑴′ (𝒛′‬‬
‫)𝒃( ⃗‬
‫العبارة المركّبة‬ ‫التحويل النقطي‬
‫‪′‬‬ ‫‪′‬‬ ‫االنسحاب 𝑇 الذي شعاعه ⃗‬
‫𝒃 ‪𝑻(𝑴) = 𝑴 ⇒ 𝒛 = 𝒛 +‬‬ ‫𝑈‬
‫) 𝟎𝒛 ‪𝒉(𝑴) = 𝑴′ ⇒ 𝒛′ − 𝒛𝟎 = 𝒌(𝒛 −‬‬ ‫التحاكي ‪ ℎ‬الذي مركزه 𝜔 و نسبته 𝑘‬
‫) 𝟎𝒛 ‪𝑹(𝑴) = 𝑴′ ⇒ 𝒛′ − 𝒛𝟎 = 𝒆𝒊𝜽 (𝒛 −‬‬ ‫الدوران 𝑅 الذي مركزه 𝜔 و زاويته 𝜃‬
‫‪′‬‬ ‫‪′‬‬
‫) 𝟎𝒛 ‪𝑺(𝑴) = 𝑴 ⇒ 𝒛 − 𝒛𝟎 = 𝒌𝒆 (𝒛 −‬‬ ‫𝜽𝒊‬ ‫التشابه المباشر 𝑆 الذي مركزه 𝜔 ‪ ،‬نسبته 𝑘 و زاويته 𝜃‬

‫مثال ‪ :‬نعتبر في المستوي المر ّكب المنسوب إلى معلم متعامد ومتجانس )𝑣 ‪⃗ ,‬‬
‫𝑢 ;𝑂( النقطتين 𝐴 و 𝐵 الحقتيهما على الترتيب ‪:‬‬
‫𝑖‪ 𝑧𝐴 = 1 − 2‬و 𝑖 ‪𝑧𝐵 = −3 +‬‬
‫𝑈‬‫‪ .1‬عيّن النقطة 𝐶 صورة النقطة 𝐵 باالنسحاب 𝑇 الذي شعاعه )‪⃗ (2‬‬
‫‪1‬‬
‫𝑖‪𝐶 = 𝑇(𝐵) ⇒ 𝑧𝐶 = 𝑧𝐵 + 𝑧𝑈⃗ = −3 + 𝑖 + 2 + 𝑖 ⇒ 𝑧𝐶 = −1 + 2‬‬
‫‪ .2‬عيّن النقطة 𝐷 صورة النقطة 𝐶 بالتحاكي ‪ ℎ‬الذي مركزه 𝐴 و نسبته ‪2‬‬
‫𝑖‪𝐷 = ℎ(𝐶) ⇒ 𝑧𝐷 − 𝑧𝐴 = 2(𝑧𝐶 − 𝑧𝐴 ) ⇒ 𝑧𝐷 = 2𝑧𝐶 − 𝑧𝐴 = 2(−1 + 2𝑖) − 1 + 2𝑖 ⇒ 𝑧𝐷 = −3 + 6‬‬
‫𝜋‬
‫‪ .3‬عيّن النقطة 𝐸 صورة النقطة 𝐷 بالدوران 𝑅 الذي مركزه 𝑂 و زاويته ‪− 2‬‬
‫𝜋‬
‫𝑖‪𝐸 = 𝑅(𝐷) ⇒ 𝑧𝐸 − 𝑧𝑂 = 𝑒 −𝑖 2 (𝑧𝐷 − 𝑧𝑂 ) ⇒ 𝑧𝐸 = 𝑖𝑧𝐷 = −𝑖(−3 + 6𝑖) ⇒ 𝑧𝐸 = 6 + 3‬‬
‫𝜋‬ ‫‪1‬‬
‫‪ .4‬عيّن النقطة 𝐹 صورة النقطة 𝐸 بالتشابه المباشر 𝑆 الذي مركزه )‪ ، 𝜔(2; 1‬نسبته ‪ 2‬و زاويته‬
‫‪2‬‬
‫𝜋𝑖 ‪1‬‬ ‫‪1‬‬ ‫‪1‬‬
‫= 𝜔𝑧 ‪𝐹 = 𝑆(𝐸) ⇒ 𝑧𝐹 −‬‬ ‫𝑖 ‪𝑒 2 (𝑧𝐸 − 𝑧𝜔 ) ⇒ 𝑧𝐹 = 𝑖(𝑧𝐸 − 𝑧𝜔 ) + 𝑧𝜔 = 𝑖(6 + 3𝑖 − 2 − 𝑖) + 2 +‬‬
‫‪2‬‬ ‫‪2‬‬ ‫‪2‬‬
‫‪1‬‬
‫=‬ ‫𝑖‪𝑖(4 + 2𝑖) + 2 + 𝑖 = 2𝑖 − 1 + 2 + 𝑖 ⇒ 𝑧𝐹 = 1 + 3‬‬
‫‪2‬‬

‫‪11‬‬
‫المعرف بعبارته المركّبة ‪ 𝒛′ = 𝒂𝒛 + 𝒃 :‬وذكر عناصره المم ّيزة‬
‫ّ‬ ‫تعيين طبيعة التحويل النقطي‬
‫العناصر المميّزة للتحويل‬ ‫طبيعة التحويل‬ ‫𝑎‬
‫𝑏 = ⃗𝑈𝑧‬ ‫𝑈‬‫انسحاب شعاعه ⃗‬ ‫‪𝑎=1‬‬
‫𝑏‬
‫= 𝜔𝑧‬ ‫𝑎=𝑘 ;‬ ‫تحاكي مركزه 𝜔 ونسبته 𝑘‬ ‫}‪𝑎 ∈ ℝ − {1‬‬
‫𝑎‪1−‬‬
‫𝑏‬
‫= 𝜔𝑧‬ ‫)𝑎(‪; 𝜃 = arg‬‬ ‫دوران مركزه 𝜔 وزاويته 𝜃‬ ‫‪𝑎 ∈ ℂ ; |𝑎| = 1‬‬
‫𝑎‪1−‬‬
‫𝑏‬
‫= 𝜔𝑧‬ ‫‪ 𝑎 ∈ ℂ ; |𝑎| ≠ 1‬تشابه مباشر مركزه 𝜔 ‪ ،‬نسبته 𝑘 وزاويته 𝜃 )𝑎(‪; 𝑘 = |𝑎| ; 𝜃 = arg‬‬
‫𝑎‪1−‬‬
‫المعرفة بالعبارات المر ّكبة التالية واذكر عناصرها المميّزة‬
‫ّ‬ ‫مثال ‪ :‬عيّن طبيعة التحويالت النقطية‬
‫‪𝒛′ = 𝒛 + 𝟐 − 𝒊 .1‬‬
‫𝑈 حيث 𝑖 ‪ ، 𝑧𝑈⃗ = 2 −‬أي ) ‪⃗ ( 2‬‬
‫𝑈‬ ‫‪ : 𝑎 = 1‬التحويل هو انسحاب شعاعه ⃗‬
‫‪−1‬‬
‫‪𝒛′ = 𝟐𝒛 + 𝟏 + 𝟑𝒊 .2‬‬
‫𝑏‬ ‫𝑖‪1+3‬‬
‫= 𝑎‪ ، 𝑧𝜔 = 1−‬أي )‪𝜔(−1; −3‬‬ ‫‪ : 𝑎 = 2‬التحويل هو تحاكي مركزه 𝜔 ونسبته ‪ 2‬حيث ‪= −1 − 3𝑖 :‬‬
‫‪1−2‬‬
‫𝟏‬ ‫𝟑√‬
‫‪𝒛′ = (𝟐 +‬‬ ‫‪𝒊) 𝒛 +‬‬ ‫‪𝟐𝒊 .3‬‬
‫𝟐‬
‫𝑖‪2‬‬ ‫‪1‬‬ ‫‪√3‬‬
‫‪ ، 𝑧𝜔 = 1‬أي )‪ 𝜔(−√3; 1‬وزاويته‬ ‫‪√3‬‬
‫‪ : |𝑎| = |2 +‬التحويل هو دوران مركزه 𝜔 حيث ‪= −√3 + 𝑖 :‬‬ ‫‪𝑖| = 1‬‬
‫‪−‬‬ ‫𝑖‬ ‫‪2‬‬
‫‪2‬‬ ‫‪2‬‬
‫‪1‬‬ ‫‪√3‬‬ ‫𝜋‬
‫= 𝜃 ‪(cos 𝜃 = 2 ; sin‬‬ ‫)‬ ‫𝜃 حيث ‪، 𝜃 = arg 𝑎 = 3 + 2𝑘𝜋 :‬‬
‫‪2‬‬
‫‪𝒛′ = 𝟐𝒊𝒛 + 𝟏 − 𝟐𝒊 .4‬‬
‫𝑖‪1−2‬‬
‫‪ : |𝑎| = |2𝑖| = 2‬التحويل هو تشابه مباشر مركزه 𝜔 حيث ‪ ، 𝑧𝜔 = 1−2𝑖 = 1 :‬أي )‪ ، 𝜔(1; 0‬نسبته ‪𝑘 = |𝑎| = 2‬‬
‫𝜋‬
‫وزاويته 𝜃 حيث ‪(cos 𝜃 = 0 ; sin 𝜃 = 1) ، 𝜃 = arg 𝑎 = + 2𝑘𝜋 :‬‬
‫‪2‬‬
‫العمليات على الشكل الجبري ‪:‬‬
‫مثال ‪: 1‬‬
‫ّ‬
‫اكتب األعداد المركبة التالية على شكلها الجبري‬
‫𝑖‪ 𝑧1 = (2 + 𝑖)2 = 2² + 𝑖² + 4𝑖 = 4 − 1 + 4𝑖 = 3 + 4‬‬
‫‪ 𝑧2 = (4 + 2𝑖)(4 − 2𝑖) = 42 − (2𝑖)2 = 16 − 4𝑖 2 = 16 − 4(−1) = 20‬‬
‫‪ 𝑧3 = (2 − 𝑖)2 (1 + 2𝑖)2 = [(2 − 𝑖)(1 + 2𝑖)]2 = (2 + 4𝑖 − 𝑖 − 2𝑖 2 )2 = (4 + 3𝑖)2‬‬
‫𝑖‪= 16 − 9 + 24𝑖 = 7 + 24‬‬
‫𝑖‪ 𝑧4 = (3 − 2𝑖)3 = 33 − 3(3)2 (2𝑖) + 3(3)(2𝑖)2 − (2𝑖)3 = 27 − 54𝑖 − 36 + 8𝑖 = −9 − 46‬‬
‫‪3‬‬ ‫‪2‬‬ ‫‪3‬‬
‫‪1‬‬ ‫‪√3‬‬ ‫‪1 3‬‬ ‫‪1 2‬‬ ‫‪√3‬‬ ‫‪1‬‬ ‫‪√3‬‬ ‫‪√3‬‬
‫𝑖 ‪ 𝑧5 = (− 2 +‬‬ ‫)‬ ‫𝑖( )‪= (− 2) + 3 (− 2‬‬ ‫)‬ ‫𝑖( )‪+ 3 (− 2‬‬ ‫)‬ ‫𝑖( ‪+‬‬ ‫)‬
‫‪2‬‬ ‫‪2‬‬ ‫‪2‬‬ ‫‪2‬‬
‫‪1 3√3‬‬ ‫‪9 3√3‬‬ ‫‪8‬‬
‫‪=− +‬‬ ‫‪𝑖+ −‬‬ ‫‪𝑖= = 1‬‬
‫‪8‬‬ ‫‪8‬‬ ‫‪8‬‬ ‫‪8‬‬ ‫‪8‬‬
‫𝑖‪4−6‬‬ ‫)𝑖‪(4−6𝑖)(3−2‬‬ ‫‪12−8𝑖−18𝑖−12‬‬ ‫𝑖‪−26‬‬ ‫𝑖‪−26‬‬
‫𝑖‪ 𝑧6 = 3+2𝑖 = (3+2𝑖)(3−2𝑖) = 32 −(2𝑖)2 = 9+4 = 13 = −2‬‬
‫𝑖‪1+‬‬ ‫)‪(1+𝑖)(3+𝑖√2‬‬ ‫‪3+𝑖√2+3𝑖+𝑖 2 √2‬‬ ‫𝑖)‪3−√2+(3+√2‬‬ ‫𝑖)‪3−√2+(3+√2‬‬ ‫‪3−√2‬‬ ‫)‪(3+√2‬‬
‫𝑖‪ 𝑧7 = 3−𝑖√2 = (3−‬‬ ‫=‬ ‫‪2‬‬ ‫=‬ ‫=‬ ‫=‬ ‫‪+‬‬ ‫𝑖‬
‫‪√2)(3+𝑖√2‬‬ ‫)‬ ‫)‪32 −(𝑖√2‬‬ ‫‪9+2‬‬ ‫‪11‬‬ ‫‪11‬‬ ‫‪11‬‬

‫𝑛‪(1+𝑖)(1+𝑖) 4‬‬ ‫𝑛‪4‬‬


‫𝑛‪1+𝑖 4‬‬ ‫‪1+𝑖+𝑖+𝑖 2‬‬ ‫𝑛‪2𝑖 4‬‬
‫)𝑖‪ 𝑧8 = (1−‬‬ ‫])𝑖‪= [(1−𝑖)(1+‬‬ ‫(=‬ ‫)‬ ‫)‪= (2‬‬ ‫‪= (𝑖)4𝑛 = 1‬‬
‫‪1−𝑖 2‬‬
‫)𝜃 ‪(cos 𝜃+𝑖 sin‬‬ ‫)𝜃 ‪(cos 𝜃+𝑖 sin 𝜃)(cos 𝜃+𝑖 sin‬‬ ‫)𝜃 ‪(cos 𝜃+𝑖 sin 𝜃)(cos 𝜃+𝑖 sin‬‬
‫= ‪ 𝑧9‬‬ ‫)𝜃 ‪(cos 𝜃−𝑖 sin‬‬
‫)𝜃 ‪= (cos 𝜃−𝑖 sin 𝜃)(cos 𝜃+𝑖 sin‬‬ ‫=‬ ‫‪(cos 𝜃)2 +(sin 𝜃)2‬‬
‫=‬ ‫‪(cos 2 (sin 2‬‬
‫‪𝜃) −‬‬ ‫𝜃‪𝜃) + 2𝑖 cos 𝜃 sin 𝜃 = cos 2𝜃 + 𝑖 sin 2‬‬

‫‪11‬‬
‫تعيين الجذرين التربيعيين لعدد مركب ‪:‬‬
‫𝑥 = ‪𝒛𝟐 = 𝟖 − 𝟔𝒊 ; 𝑧 = 𝑥 + 𝑖𝑦 ⇒ 𝑧 2 = (𝑥 + 𝑖𝑦)2‬‬
‫𝑦𝑥‪⏟2 − 𝑦 2 + 2‬‬
‫‪⏟ 𝑖; |𝑧 2 | = 𝑥 2 + 𝑦 2 = √82 + 62 = 10‬‬
‫‪8‬‬ ‫‪−6‬‬
‫‪2‬‬ ‫‪2‬‬
‫‪𝑥 + 𝑦 = 10‬‬ ‫‪𝑥 =9‬‬ ‫‪2‬‬
‫‪2𝑥 2 = 18‬‬ ‫‪𝑥=3‬‬ ‫‪𝑥 = −3‬‬
‫{ ⇒ ‪𝑧 2 = 8 − 6𝑖 ⇒ { 𝑥 2 − 𝑦 2 = 8‬‬ ‫{⇒‬ ‫{⇒‪3‬‬ ‫{ أو‬
‫‪𝑥𝑦 = −3‬‬ ‫‪𝑦=−‬‬ ‫‪𝑦 = −1‬‬ ‫‪𝑦=1‬‬
‫‪2𝑥𝑦 = −6‬‬ ‫𝑥‬
‫الجذرين التربيعيين للعدد 𝑖‪ 8 − 6‬هما ‪ 𝑧1 = 3 − 𝑖 :‬و 𝑖 ‪(𝑧12 = 𝑧22 = 8 − 6𝑖) .𝑧2 = −3 +‬‬
‫𝑥 = ‪𝒛𝟐 = −𝟏𝟓 + 𝟖𝒊 ; 𝑧 = 𝑥 + 𝑖𝑦 ⇒ 𝑧 2 = (𝑥 + 𝑖𝑦)2‬‬
‫𝑦𝑥‪⏟2 − 𝑦 2 + 2‬‬
‫‪⏟ 𝑖; |𝑧 2 | = √(−15)2 + 82 = 17‬‬
‫‪−15‬‬ ‫‪8‬‬
‫‪𝑥 2 + 𝑦 2 = 17‬‬ ‫‪𝑥2 = 1‬‬
‫‪2‬‬ ‫‪2𝑥 2 = 2‬‬ ‫‪𝑥=1‬‬ ‫‪𝑥 = −1‬‬
‫‪2‬‬ ‫‪2‬‬
‫{ ⇒ ‪𝑧 = −15 + 8𝑖 ⇒ {𝑥 − 𝑦 = −15‬‬ ‫{⇒‬ ‫{⇒ ‪4‬‬ ‫{ أو‬
‫‪𝑥𝑦 = 4‬‬ ‫=𝑦‬ ‫‪𝑦=4‬‬ ‫‪𝑦 = −4‬‬
‫‪2𝑥𝑦 = 8‬‬ ‫𝑥‬
‫الجذرين التربيعيين للعدد 𝑖‪ −15 + 8‬هما ‪ 𝑧1 = 1 + 4𝑖 :‬و 𝑖‪(𝑧12 = 𝑧22 = −15 + 8𝑖 ) .𝑧2 = −1 − 4‬‬

‫حل معادالت من الدرجة األولى ‪:‬‬


‫𝑖‪𝟑𝒛 − 𝟐 + 𝒊 = (𝟏 + 𝒊)𝒛 − 𝟏 − 𝟐𝒊 ⇒ 3𝑧 − (1 + 𝑖)𝑧 = −1 − 2𝑖 + 2 − 𝑖 ⇒ (3 − 1 − 𝑖)𝑧 = 1 − 3‬‬
‫𝑖‪1 − 3𝑖 (1 − 3𝑖)(2 + 𝑖) 2 + 𝑖 − 6𝑖 − 3𝑖 2 5 − 5‬‬
‫= 𝑧 ⇒ 𝑖‪⇒ (2 − 𝑖)𝑧 = 1 − 3‬‬ ‫=‬ ‫=‬ ‫=‬ ‫𝑖‪= 1−‬‬
‫𝑖‪2−‬‬ ‫)𝑖 ‪(2 − 𝑖)(2 +‬‬ ‫‪22 − 𝑖 2‬‬ ‫‪5‬‬
‫‪ (3 − 4𝑖)𝑧 − 𝑖 = 0‬أو ‪(𝟑 − 𝟒𝒊)𝒛𝟐 = 𝒊𝒛 ⇒ (3 − 4𝑖)𝑧 2 − 𝑖𝑧 = 0 ⇒ 𝑧[(3 − 4𝑖)𝑧 − 𝑖] = 0 ⇒ 𝑧 = 0‬‬
‫𝑖‬ ‫)𝑖‪𝑖(3 + 4‬‬ ‫‪4‬‬ ‫‪3‬‬ ‫‪4‬‬ ‫‪3‬‬
‫= 𝑧 ⇒ 𝑖 = 𝑧)𝑖‪(3 − 4‬‬ ‫=‬ ‫‪=−‬‬ ‫}𝑖 ‪+ 𝑖 ; 𝑆 = {0 ; − +‬‬
‫𝑖‪3 − 4‬‬ ‫‪25‬‬ ‫‪25 25‬‬ ‫‪25 25‬‬
‫𝟏‪𝒛+‬‬ ‫𝑖‪−1 − 2‬‬
‫= 𝑧 ⇒ 𝑖‪= 𝟐𝒊 ⇒ 𝑧 + 1 = 2𝑖(𝑧 − 1) ⇒ 𝑧 − 2𝑖𝑧 = −1 − 2𝑖 ⇒ 𝑧(1 − 2𝑖) = −1 − 2‬‬
‫𝟏‪𝒛−‬‬ ‫𝑖‪1 − 2‬‬
‫𝑖‪(−1 − 2𝑖)(1 + 2𝑖) −1 − 2𝑖 − 2𝑖 + 4 3 − 4‬‬ ‫‪3 4‬‬
‫=𝑧‬ ‫=‬ ‫=‬ ‫𝑖 ‪= −‬‬
‫)𝑖‪(1 − 2𝑖)(1 + 2‬‬ ‫‪5‬‬ ‫‪5‬‬ ‫‪5 5‬‬

‫حل معادالت من الدرجة الثانية ‪:‬‬


‫‪−2‬‬ ‫‪−‬‬ ‫𝑖‪4‬‬ ‫‪−2‬‬ ‫𝑖‪+ 4‬‬
‫= ‪𝒛𝟐 + 𝟐𝒛 + 𝟓 = 𝟎 ; ∆= −16 = (4𝑖)2 ; 𝑧1‬‬ ‫= ‪= −1 − 2𝑖 ; 𝑧2‬‬ ‫𝑖‪= −1 + 2‬‬
‫‪2‬‬ ‫‪2‬‬
‫‪𝒛𝟐 − 𝟒𝒛̅ − 𝟓 = 𝟎 ⇒ (𝑥 + 𝑖𝑦)2 − 4(𝑥 − 𝑖𝑦) − 5 = 0 ⇒ 𝑥 2 − 𝑦 2 + 2𝑖𝑥𝑦 − 4𝑥 + 4𝑖𝑦 − 5 = 0‬‬
‫‪𝑥 2 − 𝑦 2 − 4𝑥 − 5 = 0‬‬
‫{ ⇒ ‪⇒ 𝑥 2 − 𝑦 2 − 4𝑥 − 5 + 2𝑖𝑦(𝑥 + 2) = 0‬‬
‫‪2𝑦(𝑥 + 2) = 0‬‬
‫‪ 𝑥 = 5 ⇒ 𝑧1 = −1 ; 𝑧2 = 5‬أو ‪ 𝑦 = 0 ⇒ 𝑥 − 4𝑥 − 5 = 0 ⇒ 𝑥 = −1‬‬
‫‪2‬‬

‫𝑖‪ 𝑦 = −√7 ⇒ 𝑧3 = −2 + √7𝑖 ; 𝑧4 = −2 − √7‬أو ‪ 𝑥 = −2 ⇒ 𝑦 2 = 7 ⇒ 𝑦 = √7‬‬


‫}𝑖‪𝑆 = {−1 ; 5 ; −2 + √7𝑖 ; −2 − √7‬‬

‫االنتقال بين األشكال الثالثة (الجبري ‪ ،‬المثلّثي ‪ ،‬األ ّ‬


‫سي) لعدد مركب ‪:‬‬
‫مثال ‪: 1‬‬
‫𝑥‬ ‫‪2‬‬ ‫‪√2‬‬ ‫𝑦‬ ‫‪2‬‬ ‫‪√2‬‬
‫= 𝜃 ‪𝒛𝟏 = 𝟐 + 𝟐𝒊 ; |𝑧| = √22 + 22 = √8 = 2√2 ; cos‬‬ ‫=‬ ‫=‬ ‫= 𝜃 ‪; sin‬‬ ‫=‬ ‫=‬
‫‪|𝑧| 2√2‬‬ ‫‪2‬‬ ‫‪|𝑧| 2√2‬‬ ‫‪2‬‬
‫‪√2‬‬
‫= 𝜃 ‪cos‬‬
‫‪2 ⇒ 𝜃 = 𝜋 + 2𝑘𝜋 ⇒ 𝑧 = 2√2 (cos 𝜋 + 𝑖 sin 𝜋) = 2√2𝑒 𝑖𝜋4‬‬
‫‪√2‬‬ ‫‪4‬‬ ‫‪4‬‬ ‫‪4‬‬
‫= 𝜃 ‪sin‬‬
‫{‬ ‫‪2‬‬

‫‪11‬‬
‫‪√𝟑 −𝒊𝝅 √3‬‬ ‫𝜋‬ ‫𝜋‬ ‫‪√3 √3 1‬‬ ‫‪3 √3‬‬
‫= 𝟐𝒛‬ ‫=𝟔 𝒆‬ ‫= ]) ‪[𝑐𝑜𝑠 (− ) + 𝑖 𝑠𝑖𝑛(−‬‬ ‫‪[ − 𝑖 ] ⇒ 𝑧2 = −‬‬ ‫𝑖‬
‫𝟐‬ ‫‪2‬‬ ‫‪6‬‬ ‫‪6‬‬ ‫‪2 2‬‬ ‫‪2‬‬ ‫‪4‬‬ ‫‪4‬‬
‫𝝅‬ ‫𝝅‬ ‫𝜋‬ ‫𝜋‬ ‫𝜋‬
‫‪𝒛𝟑 = 𝟒 (𝐜𝐨𝐬 − 𝒊 𝐬𝐢𝐧 ) = 4 [cos (− ) + 𝑖 sin (− )] = 4𝑒 −𝑖 4‬‬
‫⏟‬ ‫𝟒‬ ‫𝟒‬ ‫‪4‬‬ ‫‪4‬‬
‫ليس شكال مثلّثيا‬

‫‪√2 √2‬‬
‫( ‪𝑧3 = 4‬‬ ‫‪−‬‬ ‫𝑖‪𝑖) = 2√2 − 2√2‬‬
‫‪2‬‬ ‫‪2‬‬
‫𝝅‬ ‫𝝅‬ ‫𝜋‬ ‫𝜋‬ ‫𝜋‬ ‫𝜋‬
‫]) ‪𝒛𝟒 = −𝟑 (𝐜𝐨𝐬 + 𝒊 𝐬𝐢𝐧 ) = 3 (− cos − 𝑖 sin ) = 3 [𝑐𝑜𝑠 (𝜋 + ) + 𝑖 𝑠𝑖𝑛 (𝜋 +‬‬
‫⏟‬ ‫𝟑‬ ‫𝟑‬ ‫‪3‬‬ ‫‪3‬‬ ‫‪3‬‬ ‫‪3‬‬
‫ليس شكال مثلّثيا‬

‫𝜋‪4‬‬ ‫𝜋‪4‬‬ ‫𝜋‪4‬‬ ‫‪1 √3‬‬ ‫‪3 3√3‬‬


‫‪= 3 (cos‬‬ ‫‪+ 𝑖 sin ) = 3𝑒 𝑖 3 ; 𝑧4 = 3 (− −‬‬ ‫‪𝑖) = − −‬‬ ‫𝑖‬
‫‪3‬‬ ‫‪3‬‬ ‫‪2‬‬ ‫‪2‬‬ ‫‪2‬‬ ‫‪2‬‬
‫𝝅‬ ‫𝝅‬ ‫𝜋 𝜋‬ ‫𝜋 𝜋‬ ‫𝜋‬ ‫𝜋‬ ‫𝜋‬
‫‪𝒛𝟓 = √𝟓 (𝐬𝐢𝐧 + 𝒊 𝐜𝐨𝐬 ) = √5 [cos ( − ) + 𝑖 sin ( − )] = √5 (𝑐𝑜𝑠 + 𝑖 𝑠𝑖𝑛 ) = √5𝑒 𝑖 3‬‬
‫⏟‬ ‫𝟔‬ ‫𝟔‬ ‫‪2 6‬‬ ‫‪2 6‬‬ ‫‪3‬‬ ‫‪3‬‬
‫ليس شكال مثلّثيا‬

‫‪1 √3‬‬ ‫‪√5 √15‬‬


‫‪𝑧5 = √5 ( +‬‬ ‫= )𝑖‬ ‫‪+‬‬ ‫𝑖‬
‫‪2‬‬ ‫‪2‬‬ ‫‪2‬‬ ‫‪2‬‬
‫طريقة ثانية لحساب 𝟓𝒛 ‪:‬‬
‫𝝅‬ ‫𝝅‬ ‫‪1 √3‬‬ ‫𝜋‬ ‫𝜋‬ ‫𝜋‬ ‫‪√5 √15‬‬
‫𝐧𝐢𝐬( 𝟓√ = 𝟓𝒛‬ ‫‪+ 𝒊 𝐜𝐨𝐬 ) = √5 ( +‬‬ ‫= ‪𝑖) = √5 (𝑐𝑜𝑠 + 𝑖 𝑠𝑖𝑛 ) = √5𝑒 𝑖 3‬‬ ‫‪+‬‬ ‫𝑖‬
‫𝟔‬ ‫𝟔‬ ‫‪2‬‬ ‫‪2‬‬ ‫‪3‬‬ ‫‪3‬‬ ‫‪2‬‬ ‫‪2‬‬
‫مثال ‪: 2‬‬
‫نعتبر العددين المر ّكبين ‪ 𝑧1‬و ‪ 𝑧2‬حيث ‪𝑧2 = √2 − √2𝑖 ، 𝑧1 = −√3 + 𝑖 :‬‬
‫سي‬‫اكتب 𝟏𝒛 و 𝟐𝒛 على الشكل األ ّ‬
‫‪|𝑧1 | = √3 + 1 = 2‬‬ ‫‪|𝑧1 | = 2‬‬ ‫𝜋‪5‬‬
‫𝑖‬
‫{‬ ‫‪√3‬‬ ‫‪1‬‬ ‫⇒‬ ‫{‬ ‫𝜋‪5‬‬ ‫⇒‬ ‫𝑧‬‫‪1‬‬ ‫=‬ ‫𝑒‪2‬‬ ‫‪6‬‬
‫‪cos 𝜃1 = −‬‬ ‫= ‪; sin 𝜃1‬‬ ‫= ) ‪arg(𝑧1‬‬
‫‪2‬‬ ‫‪2‬‬ ‫‪6‬‬
‫‪|𝑧2 | = √2 + 2 = 2‬‬ ‫‪|𝑧2 | = 2‬‬ ‫𝜋‬
‫𝜋‬ ‫𝑖‪−‬‬
‫{‬ ‫‪√2‬‬ ‫‪√2‬‬ ‫⇒‬ ‫{‬ ‫⇒‬ ‫𝑧‬‫‪2‬‬ ‫=‬ ‫𝑒‪2‬‬ ‫‪4‬‬
‫= ‪cos 𝜃2‬‬ ‫‪; sin 𝜃2 = −‬‬ ‫‪arg(𝑧2 ) = −‬‬
‫‪2‬‬ ‫‪2‬‬ ‫‪4‬‬
‫𝒊‪−√𝟑+‬‬
‫=𝑳‬ ‫استنتج الطويلة وعمدة للعدد المركّب 𝑳 حيث ‪:‬‬
‫𝒊𝟐√‪√𝟐−‬‬

‫| ‪|𝑧1‬‬ ‫‪2‬‬
‫‪𝑧1‬‬ ‫= |𝐿|‬ ‫‪|𝐿| = 1‬‬
‫= |𝐿|‬ ‫‪2‬‬
‫{⇒ =𝐿‬ ‫| ‪|𝑧2‬‬ ‫{⇒‬ ‫{ ⇒‬ ‫𝜋‪13‬‬
‫‪𝑧2‬‬ ‫𝜋 𝜋‪5‬‬ ‫= )𝐿(‪arg‬‬
‫‪arg(𝐿) = 𝜃1 − 𝜃2‬‬ ‫(‬ ‫)‬
‫= 𝐿 ‪arg‬‬ ‫‪+‬‬ ‫‪12‬‬
‫‪6‬‬ ‫‪4‬‬
‫اكتب العدد المركّب 𝑳 على الشكل الجبري‬
‫𝑖 ‪−√3 +‬‬ ‫)𝑖‪(−√3 + 𝑖)(√2 + √2‬‬ ‫‪−√6 − √2 −√6 + √2‬‬
‫=𝐿‬ ‫=‬ ‫=𝐿 ⇒‬ ‫‪+‬‬ ‫𝑖‬
‫𝑖‪√2 − √2‬‬ ‫)𝑖‪(√2 − √2𝑖)(√2 + √2‬‬ ‫‪4‬‬ ‫‪4‬‬
‫𝝅𝟑𝟏‬ ‫𝝅𝟑𝟏‬
‫𝐧𝐢𝐬‬ ‫𝐬𝐨𝐜 و‬ ‫استنتج قيمتي‬
‫𝟐𝟏‬ ‫𝟐𝟏‬

‫‪13𝜋 −√6 − √2‬‬


‫‪−√6 − √2 −√6 + √2‬‬ ‫𝜋‪13‬‬ ‫𝜋‪13‬‬ ‫=‬ ‫‪cos‬‬
‫=𝐿‬ ‫‪+‬‬ ‫‪𝑖 = cos‬‬ ‫‪+ 𝑖 sin‬‬ ‫⇒‬ ‫‪12‬‬ ‫‪4‬‬
‫‪4‬‬ ‫‪4‬‬ ‫‪12‬‬ ‫‪12‬‬ ‫‪13𝜋 −√6 + √2‬‬
‫‪sin‬‬ ‫=‬
‫{‬ ‫‪12‬‬ ‫‪4‬‬

‫‪11‬‬
‫‪ -I‬المتتالية الحسابية ‪:‬‬
‫‪ .1‬العالقة التراجعية ‪:‬‬
‫𝑟 ‪𝑢𝑛+1 = 𝑢𝑛 +‬‬
‫‪ .2‬عبارة الحد العام ‪:‬‬
‫𝑟)‪𝑢𝑛 = 𝑢𝑝 + (𝑛 − 𝑝)𝑟 ; 𝑢𝑛 = 𝑢0 + 𝑛𝑟 ; 𝑢𝑛 = 𝑢1 + (𝑛 − 1‬‬
‫‪ .3‬الوسط الحسابي ‪:‬‬
‫فإن ‪𝑎 + 𝑐 = 2𝑏 :‬‬ ‫إذا كانت األعداد 𝑎 ‪ 𝑐, 𝑏,‬حدود متتابعة لمتتالية حسابية ‪ّ ،‬‬
‫تنبيه ‪ :‬لحل مسائل الوسط الحسابي ‪ ،‬ننطلق دائما من معادلة المجموع ) ⋯ = 𝑐 ‪ (𝑎 + 𝑏 +‬وتعويضها بالمعادلة‬
‫نعوض 𝑏 في معادلة الجداء ) ⋯ = 𝑐 × 𝑏 × 𝑎( فنحصل على معادلة من الشكل‬ ‫) ⋯ = 𝑏‪ (3‬لتعيين قيمة 𝑏 ‪ ،‬ث ّم ّ‬
‫) ⋯ = 𝑐 × 𝑎( ‪ ،‬ث ّم نكتب 𝑎 و 𝑐 بداللة 𝑏 و 𝑟 ‪ ،‬فنحصل على معادلة من الدرجة الثانية ذات المجهول 𝑟 من الشكل‬
‫] ⋯ = )𝑟 ‪ [(𝑏 − 𝑟)(𝑏 +‬التي تكافئ ) ⋯ = ‪ ، (𝑏 2 − 𝑟 2‬ونعيّن قيمتي 𝑟 (الموجبة والسالبة) مع التركيز جيّدا‬
‫على طبيعة المتتالية المعطاة في السؤال (متزايدة أم متناقصة) لمعرفة أي القيمتين نأخذ‪ ،‬ث ّم نحسب الحد ّين 𝑎 و 𝑐‪.‬‬
‫مثال ‪: 1‬‬
‫‪𝑢 + 𝑢1 + 𝑢2 = 3‬‬
‫‪{ 0‬‬ ‫عيّن الحدود الثالثة األولى ‪ 𝑢2 ، 𝑢1 ، 𝑢0‬لمتتالية حسابية حيث ‪:‬‬
‫‪𝑢0 × 𝑢1 × 𝑢2 = −24‬‬
‫‪𝑢0 + 𝑢1 + 𝑢2 = 3 ⇒ 3𝑢1 = 3 ⇒ 𝑢1 = 1‬‬
‫⏟ × ‪𝑢0‬‬ ‫𝑢 ⇒ ‪𝑢1 × 𝑢2 = −24‬‬ ‫𝑢 × ‪⏟0‬‬ ‫‪⏟2 = −24 ⇒ (1 − 𝑟)(1 + 𝑟) = −24 ⇒ 1 − 𝑟 2 = −24 ⇒ 𝑟 2 = 25‬‬
‫‪=1‬‬ ‫𝑟‪𝑢1 −‬‬ ‫𝑟‪𝑢1 +‬‬
‫بما أ ّن السؤال لم يحد ّد طبيعة المتتالية ) 𝑛𝑢( ‪ ،‬فإ ّن ‪ 𝑟 = −5‬أو ‪𝑟 = 5‬‬
‫‪‬‬ ‫)‪𝑟 = −5 ⇒ 𝑢0 = 1 − (−5) = 6 ; 𝑢2 = 1 + (−5) = −4 ⇒ (𝑢0 , 𝑢1 , 𝑢2 ) = (6,1, −4‬‬
‫‪‬‬ ‫)‪𝑟 = 5 ⇒ 𝑢0 = 1 − 5 = −4 ; 𝑢2 = 1 + 5 = 6 ⇒ (𝑢0 , 𝑢1 , 𝑢2 ) = (−4,1,6‬‬
‫التحقيق ‪:‬‬
‫‪𝑢0 + 𝑢1 + 𝑢2 = 6 + 1 − 4 = 3 ; 𝑢0 × 𝑢1 × 𝑢2 = 6 × 1 × (−4) = −24‬‬
‫مثال ‪: 2‬‬
‫‪𝑢1 + 𝑢2 + 𝑢3 = 24‬‬
‫‪{ 2‬‬ ‫عيّن الحدود ‪ 𝑢3 ، 𝑢2 ، 𝑢1‬لمتتالية حسابية متناقصة حيث ‪:‬‬
‫‪𝑢1 + 𝑢22 + 𝑢32 = 210‬‬
‫‪𝑢1 + 𝑢2 + 𝑢3 = 24 ⇒ 3𝑢2 = 24 ⇒ 𝑢2 = 8‬‬
‫‪𝑢12 + 𝑢22 + 𝑢32 = 210 ⇒ 𝑢12 + 𝑢32 = 210 − 64 ⇒ (8 − 𝑟)2 + (8 + 𝑟)2 = 146 ⇒ 𝑟 2 = 9‬‬
‫بما أ ّن المتتالية ) 𝑛𝑢( متناقصة ‪ ،‬فإ ّن ‪ ، 𝑟 = −3‬منه ‪ 𝑢1 = 𝑢2 − 𝑟 = 11 :‬و ‪𝑢3 = 𝑢2 + 𝑟 = 5‬‬
‫التحقيق ‪:‬‬
‫‪2‬‬ ‫‪2‬‬ ‫‪2‬‬
‫‪𝑢1 + 𝑢2 + 𝑢3 = 11 + 8 + 5 = 24 ; 11 + 8 + 5 = 121 + 64 + 25 = 210‬‬
‫‪ .4‬حساب المجاميع ‪:‬‬
‫‪𝑛−𝑝+1‬‬
‫= 𝑛𝑢 ‪𝑆𝑛 = 𝑢𝑝 + 𝑢𝑝+1 + ⋯ +‬‬ ‫) 𝑛𝑢 ‪(𝑢𝑝 +‬‬
‫‪2‬‬
‫‪𝑛+1‬‬
‫= 𝑛𝑢 ‪𝑢0 + 𝑢1 + ⋯ +‬‬ ‫) 𝑛𝑢 ‪(𝑢0 +‬‬
‫‪2‬‬
‫𝑛‬
‫) 𝑛𝑢 ‪𝑢1 + 𝑢2 + ⋯ + 𝑢𝑛 = (𝑢1 +‬‬
‫‪2‬‬
‫مثال ‪:‬‬
‫‪10‬‬
‫= ‪𝑢0 = 1 ; 𝑟 = 3 ; 𝑆 = 𝑢0 + ⋯ + 𝑢9‬‬ ‫𝑢 ‪(𝑢0 +‬‬‫‪⏟9 ) = 5(1 + 28) = 5 × 29 = 145‬‬
‫‪2‬‬
‫𝑟‪𝑢0 +9‬‬

‫‪19‬‬
‫‪ -II‬المتتالية الهندسية ‪:‬‬
‫‪ .1‬العالقة التراجعية ‪:‬‬
‫𝑞 × 𝑛𝑣 = ‪𝑣𝑛+1‬‬
‫‪ .2‬عبارة الحد العام ‪:‬‬
‫𝑝‪𝑛−‬‬ ‫𝑛‬ ‫‪𝑛−1‬‬
‫𝑞 × 𝑝𝑣 = 𝑛𝑣‬ ‫𝑞 × ‪; 𝑣𝑛 = 𝑣0 × 𝑞 ; 𝑣𝑛 = 𝑣1‬‬
‫‪ .3‬الوسط الهندسي ‪:‬‬
‫إذا كانت األعداد 𝑎 ‪ 𝑐, 𝑏,‬حدود متتابعة لمتتالية هندسية ‪ّ ،‬‬
‫فإن ‪𝑎 × 𝑐 = 𝑏 :‬‬ ‫‪2‬‬

‫تنبيه ‪ :‬لحل مسائل الوسط الهندسي ‪ ،‬ننطلق دائما من معادلة الجداء ) ⋯ = 𝑐 × 𝑏 × 𝑎( وتعويضها بالمعادلة‬
‫نعوض 𝑏 في معادلة المجموع ) ⋯ = 𝑐 ‪ (𝑎 + 𝑏 +‬فنحصل على معادلة من الشكل‬ ‫) ⋯ = ‪ (𝑏 3‬لتعيين قيمة 𝑏 ‪ ،‬ث ّم ّ‬
‫) ⋯ = 𝑐 ‪ ، (𝑎 +‬ث ّم نكتب 𝑎 و 𝑐 بداللة 𝑏 و 𝑞 ‪ ،‬فنحصل على معادلة من الدرجة الثانية ذات المجهول 𝑞 من الشكل‬
‫𝑏‬
‫] ⋯ = 𝑞𝑏 ‪ [𝑞 +‬التي تكافئ ‪( 𝑏𝑞 2 − ⋯ 𝑞 + 𝑏 = 0‬نضرب طرفي المعادلة في 𝑞)‪ ،‬ونعيّن قيمتي 𝑞 مع التركيز‬
‫جيّدا على طبيعة المتتالية المعطاة في السؤال ]رتيبة )‪ ، (𝑞 > 0‬غير رتيبة )‪ ، (𝑞 < 0‬متزايدة )‪ ، (𝑞 > 1‬متناقصة‬
‫)‪ [(0 > 𝑞 > 1‬وإشارة حدودها لمعرفة أي القيمتين نأخذ‪ ،‬ث ّم نحسب الحد ّين 𝑎 و 𝑐‪.‬‬
‫مثال ‪: 1‬‬
‫‪𝑣1 × 𝑣3 = 256‬‬
‫{‬ ‫عيّن الحدود ‪ 𝑣3 ، 𝑣2 ، 𝑣1‬لمتتالية هندسية متزايدة ‪ ،‬حدودها موجبة حيث ‪:‬‬
‫‪𝑣1 + 𝑣2 + 𝑣3 = 56‬‬
‫)ألن حدود المتتالية ) 𝑛𝑣( موجبة( ‪𝑣1 × 𝑣3 = 256 ⇒ 𝑣22 = 256 ⇒ 𝑣2 = 16‬‬ ‫ّ‬
‫‪16‬‬
‫⇒ ‪𝑣1 + 𝑣⏟2 + 𝑣3 = 56 ⇒ 𝑣⏟1 + 𝑣⏟3 = 40‬‬ ‫‪+ 16𝑞 = 40 ⇒ 16𝑞 2 − 40𝑞 + 16 = 0‬‬
‫‪𝑢2‬‬ ‫𝑞‬
‫‪16‬‬ ‫𝑞‪𝑢2 .‬‬
‫𝑞‬
‫‪𝑣2‬‬
‫ّ‬
‫)ألن المتتالية ) 𝑛𝑣( متزايدة( ‪⇒ 2𝑞 2 − 5𝑞 + 2 = 0 ⇒ 𝑞 = 2‬‬ ‫= ‪⇒ 𝑣1‬‬ ‫‪= 8 ; 𝑣3 = 𝑣2 . 𝑞 = 32‬‬
‫𝑞‬
‫التحقيق ‪:‬‬
‫‪𝑣1 × 𝑣3 = 8 × 32 = 256 ; 𝑣1 + 𝑣2 + 𝑣3 = 8 + 16 + 32 = 56‬‬
‫مثال ‪: 2‬‬
‫‪𝑣1 × 𝑣5 = 16‬‬
‫{‬ ‫) 𝑛𝑣( متتالية هندسية حيث ‪:‬‬
‫‪𝑣2 + 𝑣3 + 𝑣4 = 6‬‬
‫أن ‪ ، 𝑣1 × 𝑣5 = 𝑣32‬ث ّم احسب الحدود ‪𝑣5 , 𝑣4 , 𝑣3 , 𝑣2 , 𝑣1‬‬ ‫اثبت ّ‬
‫‪𝑣3‬‬
‫= ‪𝑣1 × 𝑣5‬‬ ‫‪× 𝑣3 . 𝑞 2 = 𝑣32‬‬
‫‪𝑞2‬‬
‫‪ 𝑣3 = −4‬أو ‪𝑣1 × 𝑣5 = 16 ⇒ 𝑣32 = 16 ⇒ 𝑣3 = 4‬‬
‫𝑣‬ ‫‪4‬‬
‫‪ 𝑣3 = 4 ∶ 𝑣2 + 𝑣3 + 𝑣4 = 6 ⇒ 𝑣2 + 𝑣4 = 2 ⇒ 𝑞3 + 𝑣3 . 𝑞 = 2 ⇒ 𝑞 + 4𝑞 − 2 = 0‬‬
‫)حالة مرفوضة( ‪⇒ 4𝑞 2 − 2𝑞 + 4 = 0 ⇒ 2𝑞 2 − 𝑞 + 2 = 0 ; ∆= −15‬‬
‫‪𝑣3‬‬ ‫‪4‬‬
‫‪‬‬ ‫⇒ ‪𝑣3 = −4 ∶ 𝑣2 + 𝑣3 + 𝑣4 = 6 ⇒ 𝑣2 + 𝑣4 = 10‬‬ ‫‪+ 𝑣3 . 𝑞 = 10 ⇒ − 𝑞 − 4𝑞 − 10 = 0‬‬
‫𝑞‬
‫‪1‬‬
‫‪⇒ 4𝑞 2 + 10𝑞 + 4 = 0 ⇒ 2𝑞 2 + 5𝑞 + 2 = 0 ⇒ 𝑞 = −‬‬ ‫‪ 𝑞 = −2‬أو‬
‫‪2‬‬
‫‪1‬‬ ‫𝑣‬ ‫‪𝑣3‬‬
‫= ‪ 𝑞 = − 2 ∶ 𝑣1 = 𝑞32 = −16 ; 𝑣2‬‬ ‫‪= 8 ; 𝑣3 = −4 ; 𝑣4 = 𝑣3 . 𝑞 = 2 ; 𝑣5 = 𝑣3 . 𝑞 2 = −1‬‬
‫𝑞‬
‫‪𝑣3‬‬ ‫‪𝑣3‬‬
‫= ‪ 𝑞 = −2 ∶ 𝑣1 = 𝑞2 = −1 ; 𝑣2‬‬ ‫‪= 2 ; 𝑣3 = −4 ; 𝑣4 = 𝑣3 . 𝑞 = 8 ; 𝑣5 = 𝑣3 . 𝑞 2 = −16‬‬
‫𝑞‬
‫‪ .4‬حساب المجاميع ‪:‬‬
‫‪𝑛+1‬‬ ‫‪𝑛+1‬‬
‫𝑞‬ ‫‪−1‬‬ ‫𝑞‪1−‬‬
‫( ‪𝑆𝑛 = 𝑣𝑝 + 𝑣𝑝+1 + ⋯ + 𝑣𝑛 = 𝑣0‬‬ ‫( ‪) = 𝑣0‬‬ ‫)‬
‫‪𝑞−1‬‬ ‫𝑞‪1−‬‬
‫‪𝑞 𝑛−𝑝+1 − 1‬‬ ‫‪1 − 𝑞 𝑛−𝑝+1‬‬
‫( ‪𝑣0 + 𝑣1 + ⋯ + 𝑣𝑛 = 𝑣0‬‬ ‫( 𝑝𝑣 = )‬ ‫)‬
‫‪𝑞−1‬‬ ‫𝑞‪1−‬‬

‫‪11‬‬
‫‪𝑞𝑛 − 1‬‬ ‫𝑛𝑞 ‪1 −‬‬
‫( ‪𝑣1 + ⋯ + 𝑣𝑛 = 𝑣1‬‬ ‫( ‪) = 𝑣1‬‬ ‫)‬
‫‪𝑞−1‬‬ ‫𝑞‪1−‬‬
‫‪ .5‬حساب الجداءات ‪:‬‬
‫)‪𝑛(𝑛+1‬‬
‫𝑞 ‪𝑃𝑛 = 𝑣0 × 𝑣1 × … × 𝑣𝑛 = 𝑣0 × 𝑣0 . 𝑞 × 𝑣0 . 𝑞 2 × … × 𝑣0 . 𝑞 𝑛 = 𝑣0𝑛+1 . 𝑞1+2+⋯+𝑛 = 𝑣0𝑛+1 .‬‬ ‫‪2‬‬

‫سية أو اللوغاريتمية ‪:‬‬


‫‪ -III‬المتتالية المشتملة على الدالة األ ّ‬
‫تنبيه ‪ :‬دراسة هذا النوع من المتتاليات يتطلب معرفة جيّدة بخواص الدالتين األسّية واللوغاريتمية ‪ ،‬فراجعها‪.‬‬

‫مثال ‪: 1‬‬
‫) 𝑛𝑢( متتالية هندسية حدودها موجبة تماما حيث ‪:‬‬
‫‪𝑙𝑛 𝑢3 + 𝑙𝑛𝑢4 = 5 ; 𝑙𝑛𝑢3 − 𝑙𝑛 𝑢4 = −1‬‬
‫‪ .1‬عيّن أساس المتتالية )‪ (un‬و حدها األول ‪.u1‬‬
‫‪ .2‬أكتب ‪ un‬بداللة ‪ ، n‬ثم احسب الجداء ‪.𝑃𝑛 = 𝑢1 × 𝑢2 × … × 𝑢𝑛 :‬‬
‫)‪ (vn‬متتالية عددية معرفة على *‪ N‬بـ ‪𝑣𝑛 = 𝑙𝑛 𝑢𝑛+1 − 2𝑙𝑛 𝑢𝑛 :‬‬
‫‪ .3‬أثبت أن )‪ (vn‬متتالية حسابية ‪ ،‬يُطلب تعيين أساسها و حدها األول‪.‬‬
‫‪ .4‬احسب بداللة ‪ n‬المجموع ‪.𝑆𝑛 = 𝑣1 + 𝑣2 + ⋯ + 𝑣𝑛 :‬‬

‫‪ .1‬تعيين أساس المتتالية )‪ (un‬و حدها األول ‪u1‬‬


‫‪𝑢3‬‬ ‫‪𝑢3‬‬ ‫𝑢‬‫‪4‬‬
‫⇒ ‪𝑙𝑛𝑢3 − 𝑙𝑛 𝑢4 = −1 ⇒ ln ( ) = −1‬‬ ‫⇒ ‪= 𝑒 −1‬‬ ‫𝑒=𝑞 ⇒𝑒=‬
‫‪𝑢4‬‬ ‫‪𝑢4‬‬ ‫‪𝑢3‬‬
‫‪𝑙𝑛𝑢3 + ln 𝑢4 = 5 ⇒ 𝑙𝑛(𝑢1 . 𝑒 2 ) + 𝑙𝑛(𝑢1 . 𝑒 3 ) = 5 ⇒ ln 𝑢1 + ln 𝑒 2 + ln 𝑢1 + ln 𝑒 3 = 5‬‬
‫‪⇒ 2 ln 𝑢1 + 5 = 5 ⇒ 2 ln 𝑢1 = 0 ⇒ 𝑢1 = 1‬‬
‫‪ .2‬كتابة ‪ un‬بداللة ‪ ، n‬ثم حساب الجداء ‪𝑷𝒏 = 𝒖𝟏 × 𝒖𝟐 × … × 𝒖𝒏 :‬‬
‫‪𝑢𝑛 = 𝑢1 . 𝑞 𝑛−1 ⇒ 𝑢𝑛 = 𝑒 𝑛−1‬‬
‫)‪𝑛(𝑛−1‬‬
‫𝑒 = 𝑛𝑃 ⇒ ‪𝑃𝑛 = 𝑢1 × 𝑢2 × … × 𝑢𝑛 = 1 × 𝑒 × … × 𝑒 𝑛−1 = 𝑒 1+2+⋯+𝑛−1‬‬ ‫‪2‬‬

‫‪ .3‬اثبات أ ّن ) 𝒏𝒗( متتالية حسابية ‪ ،‬يُطلب تعيين أساسها و حدها األول‬


‫‪𝑣𝑛+1‬‬ ‫𝑒 ‪= 𝑙𝑛 𝑢𝑛+2 − 2𝑙𝑛 𝑢𝑛+1 = 𝑙𝑛 (𝑢𝑛+1 . 𝑒) − 2𝑙𝑛 (𝑢𝑛 . 𝑒) = 𝑙𝑛 𝑢𝑛+1 + ln 𝑒 − 2𝑙𝑛 𝑢𝑛 − 2 ln‬‬
‫‪= 𝑙𝑛 𝑢𝑛+1 − 2𝑙𝑛 𝑢𝑛 − 1 ⇒ 𝑣𝑛+1 = 𝑣𝑛 − 1‬‬
‫منه نستنتج أ ّن المتتالية ) 𝑛𝑣( حسابية أساسها ‪ 𝑟 = −1‬وحد ّها األول ‪𝑣1 = 𝑙𝑛 𝑢2 − 2𝑙𝑛 𝑢1 = ln 𝑒 − 2 ln 1 = 1‬‬
‫𝑛 ‪𝑣𝑛 = 𝑣1 + (𝑛 − 1)𝑟 ⇒ 𝑣𝑛 = 2 −‬‬
‫‪ .4‬حساب بداللة 𝒏 المجموع ‪𝑺𝒏 = 𝒗𝟏 + 𝒗𝟐 + ⋯ + 𝒗𝒏 :‬‬
‫𝑛‬ ‫𝑛‬ ‫𝑛‬
‫)𝑛 ‪𝑆𝑛 = 𝑣1 + 𝑣2 + ⋯ + 𝑣𝑛 = (𝑣1 + 𝑣𝑛 ) = (1 + 2 − 𝑛) ⇒ 𝑆𝑛 = (3 −‬‬
‫‪2‬‬ ‫‪2‬‬ ‫‪2‬‬
‫مثال ‪: 2‬‬
‫𝑒‪𝑢1 + 𝑢3 = 30‬‬
‫{‬ ‫) 𝑛𝑢( متتالية هندسية حدودها موجبة تماما حيث ‪:‬‬
‫‪ln(𝑢2 ) − ln(𝑢4 ) + 2 ln 3 = 0‬‬
‫‪ .1‬عيّن ‪ 𝑢1‬و 𝑞 أساس المتتالية ) 𝑛𝑢(‬
‫‪ .2‬عبّر عن 𝑛𝑢 بداللة 𝑛‬
‫‪ .3‬احسب بداللة 𝑛 المجموع ‪𝑆𝑛 = 𝑢1 + 𝑢2 + ⋯ + 𝑢𝑛 :‬‬
‫المعرفة على ‪ ℕ‬كما يلي ‪𝑣𝑛 = ln(𝑢𝑛+2 ) + ln(𝑢𝑛+1 ) :‬‬ ‫ّ‬ ‫‪ .4‬نعتبر المتتالية ) 𝑛𝑣(‬
‫أ‪ .‬اكتب 𝑛𝑣 بداللة 𝑛 ‪ ،‬ث ّم بيّن ّ‬
‫أن المتتالية ) 𝑛𝑣( حسابية يُطلب تعيين حدها األول وأساسها 𝑟‬
‫ب‪ .‬عيّن العدد الطبيعي 𝑛 حيث ‪.𝑣0 + 𝑣1 + ⋯ + 𝑣𝑛 = 12 + 48 ln 3 :‬‬

‫‪10‬‬
‫‪ .1‬تعيين 𝟏𝒖 و𝒒 أساس المتتالية ) 𝒏𝒖(‬
‫‪𝑢4‬‬
‫‪ln(𝑢2 ) − ln(𝑢4 ) + 2 ln 3 = 0 ⇒ ln(𝑢4 ) − ln(𝑢2 ) = 2 ln 3 ⇒ ln ( ) = 2 ln 3‬‬
‫‪𝑢2‬‬
‫‪2‬‬
‫𝑞 ‪𝑢2 .‬‬
‫( ‪⇒ ln‬‬ ‫‪) = 2 ln 3 ⇒ ln 𝑞 2 = 2 ln 3 ⇒ 2 ln 𝑞 = 2 ln 3 ⇒ 𝑞 = 3‬‬
‫‪𝑢2‬‬
‫𝑒‪𝑢1 + 𝑢3 = 30𝑒 ⇒ 𝑢1 + 𝑢1 . 𝑞 2 = 30𝑒 ⇒ 10𝑢1 = 30𝑒 ⇒ 𝑢1 = 3‬‬
‫‪ .2‬عبارة 𝒏𝒖 بداللة 𝒏‬
‫‪𝑛−1‬‬ ‫‪𝑛−1‬‬ ‫𝑛‬
‫𝑞 ‪𝑢𝑛 = 𝑢1 .‬‬ ‫‪⇒ 𝑢𝑛 = 3𝑒. 3‬‬ ‫𝑒 ‪⇒ 𝑢𝑛 = 3 .‬‬
‫‪ .3‬حساب بداللة 𝒏 المجموع ‪𝑺𝒏 = 𝒖𝟏 + 𝒖𝟐 + ⋯ + 𝒖𝒏 :‬‬
‫𝑛‬
‫‪𝑞 −1‬‬ ‫𝑛 𝑒‪3‬‬
‫( ‪𝑆𝑛 = 𝑢1 + 𝑢2 + ⋯ + 𝑢𝑛 = 𝑢1‬‬ ‫= 𝑛𝑆 ⇒ )‬ ‫)‪(3 − 1‬‬
‫‪𝑞−1‬‬ ‫‪2‬‬
‫‪𝒗𝒏 = 𝐥𝐧(𝒖𝒏+𝟐 ) + 𝐥𝐧(𝒖𝒏+𝟏 ) .4‬‬
‫أ‪ .‬كتابة 𝒏𝒗 بداللة 𝒏‬
‫)𝑒(‪𝑣𝑛 = ln(𝑢𝑛+2 ) + ln(𝑢𝑛+1 ) = ln(3 . 𝑒) + ln(3 . 𝑒) = ln(3 ) + ln(𝑒) + ln(3𝑛+1 ) + ln‬‬
‫‪𝑛+2‬‬ ‫‪𝑛+1‬‬ ‫‪𝑛+2‬‬

‫‪𝑣𝑛 = (𝑛 + 2) ln 3 + (𝑛 + 1) ln 3 + 2 ⇒ 𝑣𝑛 = (2𝑛 + 3) ln 3 + 2‬‬


‫بيان أنّ المتتالية ) 𝒏𝒗( حسابية يطلب تعيين حدها األول وأساسها 𝒓‬
‫‪𝑣𝑛+1‬‬ ‫‪= (2𝑛 + 5) ln 3 + 2 = (2𝑛 + 3) ln 3 + 2 + 2 ln 3 = 𝑣𝑛 + 2 ln 3‬‬
‫منه ‪ ،‬نستنتج ّ‬
‫أن المتتالية ) 𝑛𝑣( حسابية أساسها ‪ 𝑟 = 2 ln 3‬وحدها األول ‪𝑣0 = 3 ln 3 + 2‬‬
‫ب‪ .‬تعيين العدد الطبيعي 𝒏 حيث ‪𝒗𝟎 + 𝒗𝟏 + ⋯ + 𝒗𝒏 = 𝟏𝟐 + 𝟒𝟖 𝐥𝐧 𝟑 :‬‬
‫‪𝑛+1‬‬
‫⇒ ‪𝑣0 + 𝑣1 + ⋯ + 𝑣𝑛 = 12 + 48 ln 3‬‬ ‫‪(𝑣0 + 𝑣𝑛 ) = 12 + 48 ln 3‬‬
‫‪2‬‬
‫‪𝑛+1‬‬
‫⇒‬ ‫‪[3 ln 3 + 2 + (2𝑛 + 3) ln 3 + 2] = 12 + 48 ln 3‬‬
‫‪2‬‬
‫‪𝑛+1‬‬
‫⇒‬ ‫‪[2(𝑛 + 3) ln 3 + 4] = 12 + 48 ln 3‬‬
‫‪2‬‬
‫‪2(𝑛 + 1) = 12‬‬
‫{ ⇒ ‪⇒ (𝑛 + 1)[(𝑛 + 3) ln 3 + 2] = 12 + 48 ln 3‬‬ ‫‪⇒ 𝑛=5‬‬
‫‪(𝑛 + 1)(𝑛 + 3) = 48‬‬

‫‪ -IV‬دراسة تغيّرات متتالية وتقاربها ‪:‬‬


‫لدراسة تغيّرات متتالية عددية ‪ ،‬نتبع إحدى الطرق التالية ‪:‬‬
‫المتتالية ) 𝑛𝑢( متزايدة ⇒ ‪𝑢𝑛+1 − 𝑢𝑛 > 0‬‬
‫‪ .0‬ندرس إشارة الفرق 𝑛𝑢 ‪ 𝑢𝑛+1 −‬ونستنتج ما يلي ‪ :‬المتتالية ) 𝑛𝑢( متناقصة ⇒ ‪{𝑢𝑛+1 − 𝑢𝑛 < 0‬‬
‫المتتالية ) 𝑛𝑢( ثابتة ⇒ ‪𝑢𝑛+1 − 𝑢𝑛 = 0‬‬
‫‪𝑢𝑛+1‬‬
‫المتتالية ) 𝑛𝑢( متزايدة ⇒ ‪> 1‬‬
‫𝑢‬ ‫𝑛‬
‫‪𝑢𝑛+1‬‬
‫<‪0‬‬ ‫‪ .2‬نقارن النسبة ‪ 𝑢𝑛+1‬مع ‪( 0‬ل ّما ‪ )𝑢𝑛 > 0‬ونستنتج ما يلي ‪ :‬المتتالية ) 𝑛𝑢( متناقصة ⇒ ‪< 1‬‬
‫𝑛𝑢‬ ‫𝑢‬ ‫𝑛‬
‫‪𝑢𝑛+1‬‬
‫المتتالية ) 𝑛𝑢( ثابتة ⇒ ‪= 1‬‬
‫{‬ ‫𝑛𝑢‬
‫‪ .1‬ندرس تغيّرات الدالة 𝑛𝑢 = )𝑛(𝑓 على المجال [∞‪ [0; +‬ونستنتج تغيّرات المتتالية ) 𝑛𝑢( (نفس التغيّرات)‬
‫لبيان أ ّن المتتالية ) 𝒏𝒖( متقاربة ‪ ،‬نتبع إحدى الطرق التالية ‪:‬‬
‫‪ .0‬نبيّن أ ّن 𝑙 = 𝑛𝑢 ‪lim‬‬
‫∞→𝑛‬
‫‪ .2‬نبيّن أ ّن المتتالية ) 𝑛𝑢( محدودة من األعلى )𝑀 < 𝑛𝑢( ومتزايدة‬
‫‪ .1‬نبيّن أ ّن المتتالية ) 𝑛𝑢( محدودة من األسفل )𝑚 > 𝑛𝑢( ومتناقصة‬

‫‪12‬‬
‫جدول توضيحي يبيّن العالقة بين أساس المتتالية وتغيّراتها‬
‫المتتالية الهندسية‬ ‫المتتالية الحسابية‬
‫المتتالية ) 𝒏𝒗(‬ ‫األساس 𝒒‬ ‫المتتالية ) 𝒏𝒖(‬ ‫األساس 𝒓‬
‫متزايدة‬ ‫‪𝑞>1‬‬ ‫متزايدة‬ ‫‪𝑟>0‬‬
‫متناقصة‬ ‫‪0<𝑞<1‬‬ ‫متناقصة‬ ‫‪𝑟<0‬‬
‫ثابتة‬ ‫‪𝑞=1‬‬ ‫ثابتة‬ ‫‪𝑟=0‬‬
‫غير رتيبة‬ ‫‪𝑞<0‬‬

‫‪ -V‬االستدالل بالتراجع ‪:‬‬


‫لبيان أ ّن خاصية )𝑛(𝑃 محققة من أجل كل عدد طبيعي ‪ ، 𝑛 ≥ 𝑛0‬نستعمل االستدالل بالتراجع باتباع الخطوات التالية ‪:‬‬
‫‪ .0‬تحقيق التراجع ‪ :‬نتحقق أ ّن الخاصية )𝑛(𝑃 محققة من أجل ‪𝑛 = 𝑛0‬‬
‫‪ .2‬فرض التراجع ‪ :‬نفرض أ ّن الخاصية )𝑛(𝑃 محققة من أجل 𝑛‬
‫‪ .1‬برهان التراجع ‪ :‬نبرهن أ ّن الخاصية )𝑛(𝑃 محققة من أجل ‪𝑛 + 1‬‬

‫مثال ‪:‬‬
‫‪5‬‬
‫‪ ، 𝑢𝑛+1‬حيث ‪ α‬عدد حقيقي‬ ‫معرفة على ‪ ℕ‬بحدها األول 𝛼 = ‪ 𝑢0‬والعالقة التراجعية ‪= 6 𝑢𝑛 + 335 :‬‬
‫) 𝑛𝑢( متتالية عددية ّ‬
‫‪ .1‬عيّن العدد الحقيقي ‪ α‬حيث تكون المتتالية ) 𝑛𝑢( ثابتة‬
‫‪ .2‬نضع ‪𝛼 = 2009 :‬‬
‫أن من أجل كل عدد طبيعي 𝑛 ‪𝑢𝑛 ≤ 2010 :‬‬ ‫أ‪ .‬برهن بالتراجع ّ‬
‫ب‪ .‬بيّن ّ‬
‫أن المتتالية ) 𝑛𝑢( متزايدة على ‪ .ℕ‬ماذا تستنتج ؟‬

‫‪ .1‬تعيين العدد الحقيقي ‪ α‬حيث تكون المتتالية ) 𝒏𝒖( ثابتة‬


‫‪5‬‬
‫المتتالية ) 𝑛𝑢( ثابتة يعني 𝛼 = ‪ ، 𝑢𝑛+1 = 𝑢𝑛 = 𝑢0‬منه ‪ 𝛼 = 6 𝛼 + 335‬أي ‪𝛼 = 335 × 6 = 2010‬‬
‫‪𝜶 = 𝟐𝟎𝟎𝟗 .2‬‬
‫أ‪ .‬برهان بالتراجع أنّ من أجل كل عدد طبيعي 𝒏 ‪𝒖𝒏 ≤ 𝟐𝟎𝟏𝟎 :‬‬
‫تحقيق التراجع ‪( 𝑢0 ≤ 2010 :‬محقّقة أل ّن ‪)𝑢0 = 2009‬‬
‫أن ‪𝑢𝑛 ≤ 2010 :‬‬‫فرض التراجع ‪ :‬نفرض ّ‬
‫أن ‪𝑢𝑛+1 ≤ 2010 :‬‬ ‫برهان التراجع ‪ :‬نبرهن ّ‬
‫‪5‬‬ ‫‪5‬‬ ‫‪5‬‬ ‫‪5‬‬
‫‪𝑢𝑛 ≤ 2010 ⇒ 𝑢𝑛 ≤ (2010) ⇒ 𝑢𝑛 ≤ 1675 ⇒ 𝑢𝑛 + 335 ≤ 2010 ⇒ 𝑢𝑛+1 ≤ 2010‬‬
‫‪6‬‬ ‫‪6‬‬ ‫‪6‬‬ ‫‪6‬‬
‫ب‪ .‬بيان أنّ المتتالية ) 𝒏𝒖( متزايدة على ‪ℕ‬‬
‫‪5‬‬ ‫‪1‬‬ ‫𝑛𝑢 ‪2010 −‬‬
‫= 𝑛𝑢 ‪𝑢𝑛+1 − 𝑢𝑛 = 𝑢𝑛 + 335 − 𝑢𝑛 = 335 −‬‬
‫‪6‬‬ ‫‪6‬‬ ‫‪6‬‬
‫فإن ‪ ، 2010 − 𝑢𝑛 ≥ 0‬منه المتتالية ) 𝑛𝑢( متزايدة على ‪ℕ‬‬ ‫أن ‪ّ ، 𝑢𝑛 ≤ 2010‬‬ ‫بما ّ‬
‫االستنتاج ‪ :‬المتتالية ) 𝑛𝑢( محدودة من األعلى ومتزايدة ‪ ،‬فهي إذن متقاربة‪.‬‬

‫‪‬‬
‫‪‬‬

‫‪11‬‬
‫‪ -I‬الدوال األصلية ‪:‬‬

‫تعريف ‪:‬‬
‫ّ‬ ‫معرفة على مجال 𝐼‪ .‬نقول ّ‬
‫إن الدالة 𝐹 هي دالة أصلية للدالة 𝑓 على المجال 𝐼 إذا تحقق الشرطان التاليان ‪:‬‬ ‫لتكن 𝑓 دالة عددية ّ‬
‫‪ 𝐹 ‬قابلة لالشتقاق على المجال 𝐼‬
‫𝐹‬ ‫)𝑥( ‪′‬‬
‫‪ ‬من أجل كل 𝐼 ∈ 𝑥 ‪= 𝑓(𝑥) :‬‬

‫خواص ‪:‬‬
‫‪ ‬كل دالة مستمرة على مجال تقبل دالة أصلية على هذا المجال‬
‫‪ ‬إذا كانت 𝐹 دالة أصلية للدالة 𝑓 على المجال 𝐼 ‪ ،‬فإ ّن جميع الدوال األصلية للدالة 𝑓 ّ‬
‫معرفة على 𝐼 بما يلي ‪:‬‬
‫‪𝑥 ↦ 𝐹(𝑥) + 𝑘 ; 𝑘 ∈ ℝ‬‬
‫‪ ‬توجد دالة أصلية وحيدة 𝐹 للدالة 𝑓 على المجال 𝐼 تحقّق الشرط ‪ ، 𝐹(𝑥0 ) = 𝑦0 :‬حيث 𝐼 ∈ ‪ 𝑥0‬و ‪𝑦0 ∈ ℝ‬‬
‫‪ ‬إذا كانت 𝐹 و 𝐺 دالتين أصليتين للدالتين 𝑓 و 𝑔 على الترتيب على المجال 𝐼 ‪ ،‬فإ ّن ‪:‬‬
‫‪ 𝐹 + 𝐺 ‬دالة أصلية للدالة 𝑔 ‪ 𝑓 +‬على المجال 𝐼‬
‫‪ 𝑘𝐹 ‬دالة أصلية للدالة 𝑓𝑘 على المجال 𝐼‬

‫جدول الدوال األصلية لبعض الدوال المألوفة ‪:‬‬

‫المجال 𝐼‬ ‫الدالة األصلية )𝑥(𝐹‬ ‫الدالة )𝑥(𝑓‬


‫‪ℝ‬‬ ‫‪𝑐 ,𝑐 ∈ ℝ‬‬ ‫‪0‬‬
‫‪ℝ‬‬ ‫𝑐 ‪𝑘𝑥 +‬‬ ‫𝑘‬
‫‪1 2‬‬
‫‪ℝ‬‬ ‫𝑐‪𝑥 +‬‬ ‫𝑥‬
‫‪2‬‬
‫‪𝑥 𝑛+1‬‬
‫‪ℝ‬‬ ‫𝑐‪+‬‬ ‫‪𝑥 𝑛 , 𝑛 ≠ −1‬‬
‫‪𝑛+1‬‬
‫‪1‬‬
‫[‪ ]−∞; 0‬أو [∞‪]0; +‬‬ ‫𝑐 ‪ln|𝑥| +‬‬
‫𝑥‬
‫‪1‬‬ ‫‪1‬‬
‫[‪ ]−∞; 0‬أو [∞‪]0; +‬‬ ‫𝑐‪− +‬‬
‫𝑥‬ ‫‪𝑥2‬‬
‫‪1‬‬ ‫‪1‬‬
‫[‪ ]−∞; 0‬أو [∞‪]0; +‬‬ ‫‪−‬‬ ‫𝑐‪+‬‬ ‫‪,𝑛 ≠ 1‬‬
‫‪(𝑛 − 1)𝑥 𝑛−1‬‬ ‫𝑛𝑥‬
‫‪1‬‬
‫[∞‪]0; +‬‬ ‫𝑐 ‪2 √𝑥 +‬‬
‫𝑥√‬
‫‪ℝ‬‬ ‫𝑐 ‪− cos 𝑥 +‬‬ ‫𝑥 ‪sin‬‬
‫‪ℝ‬‬ ‫𝑐 ‪sin 𝑥 +‬‬ ‫𝑥 ‪cos‬‬
‫𝜋‬ ‫𝜋‬
‫‪]−‬‬ ‫[𝜋𝑘 ‪+ 𝑘𝜋; +‬‬ ‫𝑐 ‪tan 𝑥 +‬‬ ‫𝑥 ‪1 + tan2‬‬
‫‪2‬‬ ‫‪2‬‬
‫‪ℝ‬‬ ‫𝑐 ‪𝑒𝑥 +‬‬ ‫𝑥𝑒‬

‫‪11‬‬
‫استعمال صيغ االشتقاق لتحديد بعض الدوال األصلية ‪:‬‬

‫الدالة األصلية )𝑥(𝐹‬ ‫الدالة )𝑥(𝑓‬


‫𝑐‪𝑢+𝑣+‬‬ ‫‪𝑢′ + 𝑣′‬‬
‫‪′‬‬
‫𝑐 ‪𝑢𝑣 +‬‬ ‫‪𝑢 𝑣 + 𝑢𝑣′‬‬
‫𝑢‬ ‫‪𝑢′ 𝑣 − 𝑢𝑣′‬‬
‫𝑐‪−‬‬
‫𝑣‬ ‫‪𝑣2‬‬
‫‪𝑢𝑛+1‬‬
‫𝑐‪+‬‬ ‫𝑛𝑢 ‪𝑢′ .‬‬
‫‪𝑛+1‬‬
‫‪𝑢′‬‬
‫𝑐 ‪ln|𝑢| +‬‬
‫𝑢‬
‫‪1‬‬ ‫‪𝑢′‬‬
‫𝑐‪− +‬‬
‫𝑢‬ ‫‪𝑢2‬‬
‫‪𝑢′‬‬
‫𝑐 ‪2 √𝑢 +‬‬
‫𝑢√‬
‫𝑢‬
‫𝑐‪𝑒 +‬‬ ‫𝑢 𝑒‪𝑢′‬‬
‫‪1‬‬
‫𝑐 ‪sin(𝑎𝑥 + 𝑏) +‬‬ ‫)𝑏 ‪cos(𝑎𝑥 +‬‬
‫𝑎‬
‫‪1‬‬
‫𝑐 ‪− cos(𝑎𝑥 + 𝑏) +‬‬ ‫)𝑏 ‪sin(𝑎𝑥 +‬‬
‫𝑎‬

‫أمثلة على حساب الدوال األصلية ‪:‬‬


‫المجموعة األولى ‪:‬‬
‫𝑐 ‪1) 𝑓(𝑥) = 2𝑥 + 1 ⇒ 𝐹(𝑥) = 𝑥 2 + 𝑥 +‬‬
‫‪𝑥5‬‬ ‫‪𝑥4‬‬ ‫‪3‬‬
‫𝑐 ‪2) 𝑓(𝑥) = 10𝑥 4 + 6𝑥 3 − 1 ⇒ 𝐹(𝑥) = 10 ( ) + 6 ( ) − 𝑥 + 𝑐 = 2𝑥 5 + 𝑥 4 − 𝑥 +‬‬
‫‪5‬‬ ‫‪4‬‬ ‫‪2‬‬
‫‪1‬‬
‫𝑐 ‪3) 𝑓(𝑥) = (𝑥 − 1)(𝑥 + 3) = 𝑥 2 + 2𝑥 − 3 ⇒ 𝐹(𝑥) = 𝑥 3 + 𝑥 2 − 3𝑥 +‬‬
‫‪3‬‬
‫‪1‬‬ ‫‪1 1‬‬
‫𝑐 ‪4) 𝑓(𝑥) = 2 − 𝑥 2 ⇒ 𝐹(𝑥) = − − 𝑥 3 +‬‬
‫𝑥‬ ‫‪𝑥 3‬‬
‫‪4‬‬ ‫‪4 −5‬‬ ‫‪4 𝑥 −4‬‬ ‫‪1‬‬
‫( ‪5) 𝑓(𝑥) = − 5 = − 𝑥 ⇒ 𝐹(𝑥) = −‬‬ ‫= 𝑐‪)+‬‬ ‫𝑐‪+‬‬
‫𝑥‪3‬‬ ‫‪3‬‬ ‫‪3 −4‬‬ ‫‪3𝑥 4‬‬
‫‪1‬‬ ‫‪1‬‬
‫‪6) 𝑓(𝑥) = 𝑥 +‬‬ ‫𝑐 ‪⇒ 𝐹(𝑥) = 𝑥 2 + 2√𝑥 +‬‬
‫𝑥√‬ ‫‪2‬‬
‫𝑐 ‪7) 𝑓(𝑥) = sin 𝑥 − 2 cos 𝑥 ⇒ 𝐹(𝑥) = − cos 𝑥 − 2 sin 𝑥 +‬‬

‫المجموعة الثانية ‪ :‬من الشكل 𝒏𝒖 ‪𝒖′ .‬‬


‫‪4‬‬ ‫‪(3𝑥 + 1)5‬‬
‫𝑥‪3 (3‬‬
‫⏟ = )𝑥(𝑓 )‪1‬‬ ‫= )𝑥(𝐹 ⇒ )‪⏟ + 1‬‬ ‫𝑐‪+‬‬
‫‪𝑢′‬‬
‫‪5‬‬
‫𝑢‬
‫‪3‬‬ ‫‪(4𝑥 − 1)4‬‬
‫⏟ × ‪2) 𝑓(𝑥) = 16(4𝑥 − 1)3 = 4‬‬
‫𝑥‪4 (4‬‬
‫‪⏟ − 1) ⇒ 𝐹(𝑥) = 4‬‬ ‫𝑐 ‪+ 𝑐 = (4𝑥 − 1)4 +‬‬
‫‪𝑢′‬‬
‫‪4‬‬
‫𝑢‬
‫‪1‬‬ ‫‪1 (2𝑥 + 7)7‬‬ ‫‪(2𝑥 + 7)7‬‬
‫= ‪3) 𝑓(𝑥) = (2𝑥 + 7)6‬‬ ‫⏟×‬
‫⏟‪2‬‬‫× = )𝑥(𝐹 ⇒ ‪(2𝑥 + 7)6‬‬ ‫= 𝑐‪+‬‬ ‫𝑐‪+‬‬
‫‪2 𝑢′‬‬ ‫‪2‬‬ ‫‪7‬‬ ‫‪14‬‬
‫𝑢‬

‫‪11‬‬
‫‪5‬‬ ‫‪(3𝑥 2 − 2𝑥 + 3)6‬‬
‫𝑥‪4) 𝑓(𝑥) = (6‬‬ ‫= )𝑥(𝐹 ⇒ )‪⏟ 2 − 2𝑥 + 3‬‬
‫𝑥‪⏟ − 2) (3‬‬ ‫𝑐‪+‬‬
‫‪6‬‬
‫‪𝑢′‬‬ ‫𝑢‬
‫‪4‬‬
‫‪1‬‬ ‫‪1‬‬ ‫‪1‬‬ ‫‪1 4‬‬ ‫‪1‬‬ ‫‪1 5‬‬
‫𝑐 ‪5) 𝑓(𝑥) = 2 (1 + ) = − [− 2 (1 + ) ] ⇒ 𝐹(𝑥) = − (1 + ) +‬‬
‫𝑥‬ ‫𝑥‬ ‫⏟‬‫𝑥 ⏟ 𝑥‬ ‫‪5‬‬ ‫𝑥‬
‫‪𝑢′‬‬ ‫𝑢‬
‫‪1 2‬‬
‫‪6) 𝑓(𝑥) = sin‬‬
‫‪⏟𝑥 . cos‬‬
‫= )𝑥(𝐹 ⇒ 𝑥⏟‬ ‫𝑐 ‪sin 𝑥 +‬‬
‫𝑢‬ ‫‪𝑢′‬‬
‫‪2‬‬

‫‪𝒖′‬‬
‫المجموعة الثالثة ‪ :‬من الشكل‬
‫𝟐𝒖‬
‫‪4‬‬ ‫𝑐 ‪→ 𝑢′ ⇒ 𝐹(𝑥) = − 1 +‬‬
‫= )𝑥(𝑓 )‪1‬‬
‫‪(1 + 4𝑥)2 → 𝑢2‬‬ ‫𝑥‪1 + 4‬‬
‫‪6‬‬ ‫‪2‬‬ ‫𝑐 ‪→ 𝑢′ ⇒ 𝐹(𝑥) = − 3 +‬‬
‫= )𝑥(𝑓 )‪2‬‬ ‫‪=3‬‬
‫)‪(2𝑥 + 1‬‬ ‫‪2‬‬ ‫‪(2𝑥 + 1) → 𝑢2‬‬
‫‪2‬‬ ‫‪2𝑥 + 1‬‬
‫‪1‬‬ ‫‪1‬‬ ‫‪4‬‬ ‫‪→ 𝑢′ ⇒ 𝐹(𝑥) = −‬‬ ‫‪1‬‬
‫= )𝑥(𝑓 )‪3‬‬ ‫=‬ ‫×‬ ‫‪2‬‬ ‫𝑐‪+‬‬
‫𝑢 → ‪(4𝑥 + 3)2 4 (4𝑥 + 3)2‬‬ ‫)‪4(4𝑥 + 3‬‬
‫‪−1 → 𝑢′‬‬ ‫‪1‬‬
‫= )𝑥(𝑓 )‪4‬‬ ‫‪2‬‬ ‫⇒‬ ‫)𝑥(𝐹‬ ‫=‬ ‫‪−‬‬ ‫𝑐‪+‬‬
‫𝑢 → ‪(2 − 𝑥)2‬‬ ‫𝑥‪2−‬‬
‫‪2‬‬ ‫‪2‬‬ ‫‪−3‬‬ ‫‪→ 𝑢′‬‬ ‫‪2‬‬ ‫‪1‬‬ ‫‪2‬‬
‫= )𝑥(𝑓 )‪5‬‬ ‫‪2‬‬
‫=‬ ‫‪−‬‬ ‫×‬ ‫‪2‬‬ ‫‪2‬‬ ‫‪⇒ 𝐹(𝑥) = − (−‬‬ ‫=)‬ ‫𝑐‪+‬‬
‫)𝑥‪(4 − 3‬‬ ‫𝑢 → )𝑥‪3 (4 − 3‬‬ ‫‪3‬‬ ‫𝑥‪4 − 3‬‬ ‫)𝑥‪3(4 − 3‬‬
‫‪2𝑥 + 1‬‬ ‫‪→ 𝑢′ ⇒ 𝐹(𝑥) = −‬‬ ‫‪1‬‬
‫= )𝑥(𝑓 )‪6‬‬ ‫‪2‬‬ ‫𝑐‪+‬‬
‫‪(𝑥 2‬‬ ‫‪2‬‬
‫𝑢 → )‪+ 𝑥 + 1‬‬ ‫‪2‬‬
‫‪𝑥 +𝑥+1‬‬
‫‪4𝑥 − 10‬‬ ‫‪2𝑥 − 5‬‬ ‫‪→ 𝑢′ ⇒ 𝐹(𝑥) = −‬‬ ‫‪2‬‬
‫‪7) 𝑓(𝑥) = 2‬‬ ‫‪2‬‬
‫‪= 2× 2‬‬ ‫‪2‬‬ ‫‪2‬‬ ‫‪2‬‬
‫𝑐‪+‬‬
‫)‪(𝑥 − 5𝑥 + 6‬‬ ‫𝑢 → )‪(𝑥 − 5𝑥 + 6‬‬ ‫‪𝑥 − 5𝑥 + 6‬‬
‫‪cos 𝑥 → 𝑢′‬‬ ‫‪1‬‬
‫= )𝑥(𝑓 )‪8‬‬ ‫‪2‬‬ ‫‪2 ⇒ 𝐹(𝑥) = −‬‬ ‫𝑐‪+‬‬
‫𝑢 → 𝑥 ‪sin‬‬ ‫𝑥 ‪sin‬‬
‫𝑥 ‪sin‬‬ ‫‪− sin 𝑥 → 𝑢′‬‬ ‫‪1‬‬
‫= )𝑥(𝑓 )‪9‬‬ ‫‪2‬‬
‫‪=−‬‬ ‫‪2‬‬ ‫= )𝑥(𝐹 ⇒ ‪2‬‬ ‫𝑐‪+‬‬
‫𝑥 ‪cos‬‬ ‫𝑢 → 𝑥 ‪cos‬‬ ‫𝑥 ‪cos‬‬

‫‪𝒖′‬‬
‫المجموعة الرابعة ‪ :‬من الشكل‬
‫𝒖√‬
‫‪3‬‬
‫‪→ 𝑢′‬‬
‫= )𝑥(𝑓 )‪1‬‬ ‫𝑐 ‪⇒ 𝐹(𝑥) = 2√3𝑥 + 2 +‬‬
‫𝑢√ → ‪√3𝑥 + 2‬‬
‫‪2‬‬ ‫‪2‬‬ ‫‪−5 → 𝑢′‬‬ ‫‪2‬‬ ‫‪4‬‬
‫= )𝑥(𝑓 )‪2‬‬ ‫× ‪=−‬‬ ‫𝑐 ‪⇒ 𝐹(𝑥) = − × 2√2 − 5𝑥 + 𝑐 = − √2 − 5𝑥 +‬‬
‫𝑥‪√2 − 5‬‬ ‫𝑢√ → 𝑥‪5 √2 − 5‬‬ ‫‪5‬‬ ‫‪5‬‬
‫‪1‬‬ ‫‪1‬‬ ‫‪2‬‬ ‫‪→ 𝑢′‬‬ ‫‪1‬‬
‫= )𝑥(𝑓 )‪3‬‬ ‫× =‬ ‫𝑐 ‪⇒ 𝐹(𝑥) = × 2√2𝑥 − 3 + 𝑐 = √2𝑥 − 3 +‬‬
‫𝑢√ → ‪√2𝑥 − 3 2 √2𝑥 − 3‬‬ ‫‪2‬‬
‫‪2𝑥 + 1‬‬ ‫‪→ 𝑢′‬‬
‫= )𝑥(𝑓 )‪4‬‬ ‫𝑐 ‪⇒ 𝐹(𝑥) = 2√𝑥 2 + 𝑥 + 1 +‬‬
‫𝑢√ → ‪√𝑥 + 𝑥 + 1‬‬
‫‪2‬‬
‫𝑥‬ ‫‪1‬‬ ‫𝑥‪2‬‬ ‫‪→ 𝑢′‬‬ ‫‪1‬‬
‫= )𝑥(𝑓 )‪5‬‬ ‫× =‬ ‫𝑐 ‪⇒ 𝐹(𝑥) = × 2√𝑥 2 − 1 + 𝑐 = √𝑥 2 − 1 +‬‬
‫𝑢√ → ‪√𝑥 2 − 1 2 √𝑥 2 − 1‬‬ ‫‪2‬‬
‫𝑥 ‪cos‬‬ ‫‪→ 𝑢′‬‬
‫= )𝑥(𝑓 )‪6‬‬ ‫𝑐 ‪⇒ 𝐹(𝑥) = 2√2 + sin 𝑥 +‬‬
‫𝑢√ → 𝑥 ‪√2 + sin‬‬

‫‪11‬‬
𝒖′
‫ من الشكل‬: ‫المجموعة الخامسة‬
𝒖
1 1 5
1) 𝑓(𝑥) = 𝑥 2 − 5𝑥 + ⇒ 𝐹(𝑥) = 𝑥 3 − 𝑥 2 + ln|𝑥| + 𝑐
𝑥 3 2
2
𝑥 +𝑥+1 1 1
2) 𝑓(𝑥) = = 𝑥 + 1 + ⇒ 𝐹(𝑥) = 𝑥 2 + 𝑥 + ln|𝑥| + 𝑐
𝑥 𝑥 2
7 5 1 1
3) 𝑓(𝑥) = + + 2 ⇒ 𝐹(𝑥) = 7 ln|𝑥| + 10√𝑥 − + 𝑐
𝑥 √𝑥 𝑥 𝑥
3
4) 𝑓(𝑥) = ⇒ 𝐹(𝑥) = ln|3𝑥 − 4| + 𝑐
3𝑥 − 4
1
5) 𝑓(𝑥) = ; 𝐼 = ]−∞; −1[ ⇒ 𝐹(𝑥) = ln |𝑥⏟+ 1| + 𝑐 = ln(−𝑥 − 1) + 𝑐
𝑥+1 <0
1
6) 𝑓(𝑥) = ; 𝐼 = ]−1; +∞[ ⇒ 𝐹(𝑥) = ln |𝑥⏟+ 1| + 𝑐 = ln(𝑥 + 1) + 𝑐
𝑥+1 >0
2𝑥
7) 𝑓(𝑥) = 2 ; 𝐼 = ]−∞; −2[ ⇒ 𝐹(𝑥) = ln |𝑥⏟2 − 4| + 𝑐 = ln(𝑥 2 − 4) + 𝑐
𝑥 −4 >0
1 1 3 1 1
8) 𝑓(𝑥) = = × ; 𝐼 = [2; +∞[ ⇒ 𝐹(𝑥) = ln |3𝑥
⏟ − 5| + 𝑐 = ln(3𝑥 − 5) + 𝑐
3𝑥 − 5 3 3𝑥 − 5 3 >0
3
𝑥+1 1 2(𝑥 + 1) 1
9) 𝑓(𝑥) = 2 = × 2 ⇒ 𝐹(𝑥) = ln |𝑥⏟2 + 2𝑥 + 2| + 𝑐
𝑥 + 2𝑥 + 2 2 𝑥 + 2𝑥 + 2 2 >0
1
= ln(𝑥 2 + 2𝑥 + 2) + 𝑐
2
𝑥 1 2𝑥 1 1
10) 𝑓(𝑥) = 2 = × 2 ; 𝐼 = ]−1; 1[ ⇒ 𝐹(𝑥) = ln |𝑥⏟2 − 1| + 𝑐 = ln(1 − 𝑥 2 ) + 𝑐
𝑥 −1 2 𝑥 −1 2 <0
2
cos 𝑥 𝜋
11) 𝑓(𝑥) = ; 𝐼 = ]0; [ ⇒ 𝐹(𝑥) = ln |sin ⏟𝑥| + 𝑐 = ln(sin 𝑥) + 𝑐
sin 𝑥 2 >0
1
1
12) 𝑓(𝑥) = = 𝑥 ; 𝐼 = ]1; +∞[ ⇒ 𝐹(𝑥) = ln |ln ⏟ 𝑥| + 𝑐 = ln(ln 𝑥) + 𝑐
𝑥 ln 𝑥 ln 𝑥 >0
sin 𝑥 − sin 𝑥 𝜋
13) 𝑓(𝑥) = tan 𝑥 = =− ; 𝐼 = ] ; 𝜋[ ⇒ 𝐹(𝑥) = − ln |cos
⏟𝑥| + 𝑐 = − ln(− cos 𝑥) + 𝑐
cos 𝑥 cos 𝑥 2 <0

𝒖′ . 𝒆𝒖 ‫ من الشكل‬: ‫المجموعة السادسة‬


1 1
1) 𝑓(𝑥) = 𝑒 𝑥 ⇒ 𝐹(𝑥) = 𝑒 𝑥 + 𝑐
4 4
2) 𝑓(𝑥) = 𝑒 = −(−𝑒 ) ⇒ 𝐹(𝑥) = −𝑒 −𝑥 + 𝑐
−𝑥 −𝑥

1 1
3) 𝑓(𝑥) = 𝑒 2𝑥+3 = (2𝑒 2𝑥+3 ) ⇒ 𝐹(𝑥) = 𝑒 2𝑥+3 + 𝑐
2 2
2 1 2 1 2
4) 𝑓(𝑥) = 𝑥𝑒 𝑥 = (2𝑥𝑒 𝑥 ) ⇒ 𝐹(𝑥) = 𝑒 𝑥 + 𝑐
2 2
𝑒 𝑥 → 𝑢′
5) 𝑓(𝑥) = 𝑥 ⇒ 𝐹(𝑥) = ln |𝑒⏟𝑥 + 1| + 𝑐 = 𝐹(𝑥) = ln(𝑒 𝑥 + 1) + 𝑐
𝑒 +1→𝑢 >0

11
‫‪ -II‬الحساب التكاملي ‪:‬‬
‫‪ .1‬تعريف ‪:‬‬
‫لتكن 𝑓 دالة مستمرة على مجال ]𝑏 ;𝑎[ و 𝐹 دالة أصلية للدالة 𝑓 على المجال ]𝑏 ;𝑎[‪ .‬تكامل الدالة 𝑓 من 𝑎 إلى 𝑏 هو العدد‬
‫الحقيقي ‪:‬‬
‫𝑏‬
‫)𝑎(𝐹 ‪∫ 𝑓(𝑥)𝑑𝑥 = [𝐹(𝑥)]𝑏𝑎 = 𝐹(𝑏) −‬‬
‫𝑎‬
‫‪ .2‬خواص التكامل ‪:‬‬
‫‪ ‬الخ ّطية ‪:‬‬
‫𝑎‬
‫‪∫ 𝑓(𝑥)𝑑𝑥 = 0‬‬
‫𝑎‬
‫𝑏‬ ‫𝑎‬
‫𝑥𝑑)𝑥(𝑓 ∫ ‪∫ 𝑓(𝑥)𝑑𝑥 = −‬‬
‫𝑎‬ ‫𝑏‬
‫𝑏‬ ‫𝑏‬
‫‪∫ 𝑘𝑓(𝑥)𝑑𝑥 = 𝑘 ∫ 𝑓(𝑥)𝑑𝑥 ; 𝑘 ∈ ℝ‬‬
‫𝑎‬ ‫𝑎‬
‫𝑏‬ ‫𝑏‬ ‫𝑏‬
‫𝑥𝑑)𝑥(𝑔 ∫ ‪∫ [𝑓(𝑥) + 𝑔(𝑥)]𝑑𝑥 = ∫ 𝑓(𝑥)𝑑𝑥 +‬‬
‫𝑎‬ ‫𝑎‬ ‫𝑎‬
‫عالقة شال ‪:‬‬ ‫‪‬‬
‫𝑏‬ ‫𝑐‬ ‫𝑏‬
‫𝑥𝑑)𝑥(𝑓 ∫ ‪∫ 𝑓(𝑥)𝑑𝑥 = ∫ 𝑓(𝑥)𝑑𝑥 +‬‬
‫𝑎‬ ‫𝑎‬ ‫𝑐‬
‫‪ .3‬التكامل والترتيب ‪:‬‬
‫𝑏‬
‫‪∀𝑥 ∈ [𝑎; 𝑏]: 𝑓(𝑥) ≥ 0 ⇒ ∫ 𝑓(𝑥)𝑑𝑥 ≥ 0‬‬
‫𝑎‬
‫𝑏‬ ‫𝑏‬
‫𝑥𝑑)𝑥(𝑔 ∫ ≤ 𝑥𝑑)𝑥(𝑓 ∫ ⇒ )𝑥(𝑔 ≤ )𝑥(𝑓 ‪∀𝑥 ∈ [𝑎; 𝑏]:‬‬
‫𝑎‬ ‫𝑎‬
‫‪ .4‬القيمة المتوسطة ‪:‬‬
‫‪ ‬لتكن 𝑓 دالة مستمرة على مجال ]𝑏 ;𝑎[‪ .‬القيمة المتوسطة للدالة 𝑓 على المجال ]𝑏 ;𝑎[ هي العدد الحقيقي ‪:‬‬
‫𝑏‬
‫‪1‬‬
‫𝑥𝑑)𝑥(𝑓 ∫‬
‫𝑎 𝑎‪𝑏−‬‬
‫‪ ‬إذا ُوجد عددان حقيقيان 𝑚 و 𝑀 بحيث من أجل كل ]𝑏 ;𝑎[ ∈ 𝑥 ‪ّ 𝑚 ≤ 𝑓(𝑥) ≤ 𝑀 ،‬‬
‫فإن ‪:‬‬
‫𝑏‬
‫)𝑎 ‪𝑚(𝑏 − 𝑎) ≤ ∫ 𝑓(𝑥)𝑑𝑥 ≤ 𝑀(𝑏 −‬‬
‫𝑎‬
‫‪ .5‬المكاملة بالتجزئة ‪:‬‬
‫لتكن 𝑓 و 𝑔 دالتين قابلتين لالشتقاق على مجال ]𝑏 ;𝑎[ بحيث تكون ‪ 𝑓′‬و‪ 𝑔′‬مستمرتين على المجال ]𝑏 ;𝑎[‬
‫𝑏‬ ‫𝑏‬
‫𝑥𝑑)𝑥(‪∫ 𝑓 ′ (𝑥). 𝑔(𝑥)𝑑𝑥 = [𝑓(𝑥). 𝑔(𝑥)]𝑏𝑎 − ∫ 𝑓(𝑥). 𝑔′‬‬
‫𝑎‬ ‫𝑎‬
‫أمثلة على الحساب التكاملي ‪:‬‬
‫‪3‬‬ ‫‪3‬‬
‫‪1‬‬ ‫‪9‬‬ ‫‪15‬‬
‫‪1) ∫ (𝑥 − 4)𝑑𝑥 = [ 𝑥 2 − 4𝑥] = − 12 = −‬‬
‫‪0‬‬ ‫‪2‬‬ ‫‪0‬‬ ‫‪2‬‬ ‫‪2‬‬
‫‪2‬‬ ‫‪2‬‬
‫‪1‬‬ ‫‪1‬‬ ‫‪1‬‬ ‫‪3‬‬ ‫‪5‬‬
‫= ‪2) ∫ (2𝑥 − 1 + 2 ) 𝑑𝑥 = [𝑥 2 − 𝑥 − ] = (4 − 2 − ) − (1 − 1 − 1) = + 1‬‬
‫‪1‬‬ ‫𝑥‬ ‫‪𝑥1‬‬ ‫‪2‬‬ ‫‪2‬‬ ‫‪2‬‬
‫‪1‬‬ ‫‪1‬‬ ‫‪′‬‬
‫‪2‬‬ ‫‪1‬‬ ‫‪1‬‬ ‫‪1‬‬ ‫𝑢 ‪4‬‬
‫∫ )‪3‬‬ ‫‪2‬‬
‫𝑥𝑑‬ ‫=‬ ‫‪[−‬‬ ‫]‬ ‫=‬ ‫‪−‬‬ ‫‪−‬‬ ‫)‪(−1‬‬ ‫=‬ ‫‪−‬‬ ‫‪+‬‬ ‫‪1‬‬ ‫=‬ ‫)من الشكل (‬
‫)‪−1 (2𝑥 + 3‬‬ ‫‪2𝑥 + 3 −1‬‬ ‫‪5‬‬ ‫‪5‬‬ ‫‪5 𝑢2‬‬

‫‪11‬‬
4 4 4
𝑑𝑥 1 4 −3 1 1 1 1 1 1 1 1
4) ∫ 2
=− ∫ 2
𝑑𝑥 = − [− ] =− [ ] =− ( − )=
2 (4 − 3𝑥) 3 2 (4 − 3𝑥) 3 4 − 3𝑥 2 3 3𝑥 − 4 2 3 8 2 8
−1 −1
𝑑𝑥 1 −2 1 −1 1
5) ∫ =− ∫ 𝑑𝑥 = − [2√4 − 2𝑥]1 = [√4 − 2𝑥]−1 = √2 − √6
1 √4 − 2𝑥 2 1 √4 − 2𝑥 2
2 2
𝑑𝑥
6) ∫ ⏟+ 1|] = [ln(𝑥 + 1)]20 = ln 3 − ln 1 = ln 3
= [ln |𝑥
0 𝑥 + 1 >0 0
−3 −3 −3
3 1
7) ∫ 𝑑𝑥 = 3 ∫ 𝑑𝑥 = 3 [ln |𝑥
⏟+ 2|] = 3[ln(−𝑥 − 2)]−3
−4 = 3(ln 1 − ln 2) = −3 ln 2
−4 𝑥+2 −4 𝑥 + 2 <0 −4
2 2
𝑥 +𝑥−2 2
1 2 22
8) ∫ 𝑑𝑥 = ∫ (1 + − ) 𝑑𝑥 = [𝑥 + ln|𝑥| + ] = 3 + ln 2 − 3 = ln 2
1 𝑥2 1 𝑥 𝑥2 𝑥1
4 4
1 1 4 1
9) ∫ ( + − ) 𝑑𝑥 = [ 𝑥 − ln|2 − 𝑥| − 4 ln|𝑥 + 2|]
3 4 2−𝑥 𝑥+2 4 3
3 1
= (1 − ln 2 − 4 ln 6) − ( − ln 1 − 4 ln 5) = − ln 2 − 4 ln 6 + 4 ln 5
4 4
5
10) ∫ −𝑒 −𝑥 𝑑𝑥 = [𝑒 −𝑥 ]52 = 𝑒 −5 − 𝑒 −2
2
1 2 2
1 2𝑥
11) ∫ (𝑒 2𝑥 𝑥
+ 2𝑒 − 3)𝑑𝑥 = − ∫ (𝑒 2𝑥 𝑥
+ 2𝑒 − 3)𝑑𝑥 = − [ 𝑒 + 2𝑒 − 3𝑥]𝑥
2 1 2 1
1 4 1 1 3
= − ( 𝑒 + 2𝑒 2 − 6) + ( 𝑒 2 + 2𝑒 − 3) = − 𝑒 4 − 𝑒 2 + 2𝑒 + 3
2 2 2 2
ln 10 ln 10
(𝑒 𝑥 − 3)2 49 49
12) ∫ 𝑒 𝑥 (𝑒 𝑥 − 3)𝑑𝑥 = [ ] = −0= (𝑢′ . 𝑢 ‫)من الشكل‬
ln 3 2 ln 3
2 2
1 −𝑥 1 1
𝑒 −2 −𝑥 (𝑒 −𝑥
(𝑒 −𝑥 − 2)2 (𝑒 −1 − 2)2 1
13) ∫ 𝑑𝑥 = − ∫ −𝑒 − 2)𝑑𝑥 = − [ ] = − +
0 𝑒𝑥 0 2 0
2 2
0 0 0
ln(1 − 𝑥) 1 ln2 |1 − 𝑥| ln2 (2) ln2 (2)
14) ∫ 𝑑𝑥 = − ∫ ln(1 − 𝑥) 𝑑𝑥 = − [ ] = 0+ =
−1 𝑥 − 1 −1 𝑥 − 1 2 −1
2 2
3 2 3 2 3
1 1
15) ∫ |𝑥 − 2|𝑑𝑥 = ∫ (−𝑥 + 2)𝑑𝑥 + ∫ (𝑥 − 2)𝑑𝑥 = [− 𝑥 2 + 2𝑥] + [ 𝑥 2 − 2𝑥]
0 0 2 2 0 2 2
3 5
= 2 + (− + 2) =
2 2
3
𝜋 √3
𝜋 (2)
3 sin3 𝑥
2
3 √3
16) ∫ cos 𝑥 sin 𝑥 𝑑𝑥 = [ ] = =
0 3 0 3 2
0
17) ∫ (𝑥 + 3)𝑒 𝑥 𝑑𝑥 ; 𝑢 = 𝑥 + 3 ⇒ 𝑢′ = 1 ; 𝑣 ′ = 𝑒 𝑥 ⇒ 𝑣 = 𝑒 𝑥
−2
0 0
∫ (𝑥 + 3)𝑒 𝑥 𝑑𝑥 = [(𝑥 + 3)𝑒 𝑥 ]0−2 − ∫ 𝑒 𝑥 𝑑𝑥 = [(𝑥 + 3)𝑒 𝑥 ]0−2 − [𝑒 𝑥 ]0−2 = [(𝑥 + 2)𝑒 𝑥 ]0−2 = 2
−2 −2
𝜋
18) ∫ (𝑥 + 1) sin 𝑥 𝑑𝑥 ; 𝑢 = 𝑥 + 1 ⇒ 𝑢′ = 1 ; 𝑣 ′ = sin 𝑥 ⇒ 𝑣 = − cos 𝑥
0
𝜋 𝜋
∫ (𝑥 + 1) sin 𝑥 𝑑𝑥 = [−(𝑥 + 1) cos 𝑥]𝜋0 − ∫ − cos 𝑥 𝑑𝑥 = [−(𝑥 + 1) cos 𝑥]𝜋0 + [sin 𝑥]𝜋0
0 0
= 𝜋+2

19
‫‪1‬‬ ‫‪1‬‬ ‫‪1‬‬ ‫‪1‬‬ ‫‪2‬‬ ‫‪3‬‬
‫‪19) ∫ 𝑥√1 − 𝑥𝑑𝑥 = ∫ 𝑥(1 − 𝑥)2 𝑑𝑥 ; 𝑢 = 𝑥 ⇒ 𝑢′ = 1 ; 𝑣 ′ = (1 − 𝑥)2 ⇒ 𝑣 = − (1 − 𝑥)2‬‬
‫‪0‬‬ ‫‪0‬‬ ‫‪3‬‬
‫‪1‬‬ ‫‪1‬‬ ‫𝜋‬
‫‪2‬‬ ‫‪3‬‬ ‫‪2‬‬ ‫‪3‬‬
‫𝑥𝑑 ‪∫ 𝑥√1 − 𝑥𝑑𝑥 = [− 𝑥(1 − 𝑥)2 ] − ∫ − (1 − 𝑥)2‬‬
‫‪0‬‬ ‫‪3‬‬ ‫‪0‬‬ ‫‪0‬‬ ‫‪3‬‬
‫‪1‬‬ ‫‪5 1‬‬
‫‪2‬‬ ‫‪3‬‬ ‫‪4‬‬ ‫‪4‬‬
‫= ] ‪= [− 𝑥(1 − 𝑥)2 ] + [ (1 − 𝑥)2‬‬
‫‪3‬‬ ‫‪0‬‬ ‫‪15‬‬ ‫‪0‬‬ ‫‪15‬‬
‫‪2‬‬
‫𝑥‬ ‫𝑥‬ ‫‪1‬‬
‫( ‪20) ∫ ln‬‬ ‫( ‪) 𝑑𝑥 ; 𝑢 = ln‬‬ ‫= ‪) ⇒ 𝑢′‬‬ ‫𝑥 = 𝑣 ⇒ ‪; 𝑣′ = 1‬‬
‫‪1‬‬ ‫𝑥‬ ‫‪+‬‬ ‫‪1‬‬ ‫𝑥‬ ‫‪+‬‬ ‫‪1‬‬ ‫𝑥(𝑥‬ ‫‪+‬‬ ‫)‪1‬‬
‫‪2‬‬ ‫‪2‬‬ ‫‪2‬‬ ‫‪2‬‬
‫𝑥‬ ‫𝑥‬ ‫‪1‬‬ ‫𝑥‬
‫( ‪∫ ln‬‬ ‫( ‪) 𝑑𝑥 = [𝑥 ln‬‬ ‫∫ ‪)] −‬‬ ‫( ‪𝑑𝑥 = [𝑥 ln‬‬ ‫‪)] − [ln|𝑥 + 1|]12‬‬
‫‪1‬‬ ‫‪𝑥+1‬‬ ‫‪𝑥+1 1‬‬ ‫‪1 𝑥+1‬‬ ‫‪𝑥+1 1‬‬
‫‪2‬‬ ‫‪1‬‬
‫‪= 2 ln − ln − ln 3 + ln 2 = 2 ln 2 − 2 ln 3 + 2 ln 2 − ln 3 = 4 ln 2 − 3 ln 3‬‬
‫‪3‬‬ ‫‪2‬‬
‫‪0‬‬
‫𝑥‪21) ∫ (2𝑥 2 + 3𝑥)𝑒 −𝑥 𝑑𝑥 ; 𝑢1 = 2𝑥 2 + 3𝑥 ⇒ 𝑢1′ = 4𝑥 + 3 ; 𝑣1′ = 𝑒 −𝑥 ⇒ 𝑣1 = −𝑒 −‬‬
‫‪−1‬‬
‫‪0‬‬ ‫‪0‬‬
‫𝑥𝑑 𝑥‪∫ (2𝑥 2 + 3𝑥)𝑒 −𝑥 𝑑𝑥 = −[(2𝑥 2 + 3𝑥)𝑒 −𝑥 ]0−1 − ∫ −(4𝑥 + 3)𝑒 −‬‬
‫‪⏟−1‬‬ ‫‪⏟−1‬‬
‫𝐼‬ ‫𝐽‬
‫𝑥‪𝑢2 = 4𝑥 + 3 ⇒ 𝑢2′ = 4 ; 𝑣2′ = −𝑒 −𝑥 ⇒ 𝑣2 = 𝑒 −‬‬
‫‪0‬‬
‫𝑒‪𝐽 = [(4𝑥 + 3)𝑒 −𝑥 ]0−1 − 4 ∫ 𝑒 −𝑥 𝑑𝑥 = [(4𝑥 + 3)𝑒 −𝑥 ]0−1 + 4[𝑒 −𝑥 ]0−1 = 7 − 3‬‬
‫‪−1‬‬
‫‪𝐼 = −[(2𝑥 +‬‬ ‫‪2‬‬
‫‪3𝑥)𝑒 −𝑥 ]0−1‬‬
‫‪− (7 − 3𝑒) = −𝑒 − 7 + 3𝑒 = 2𝑒 − 7‬‬
‫𝑒‬
‫‪2 ln 𝑥 ′‬‬
‫= ‪22) ∫ (ln 𝑥)2 𝑑𝑥 ; 𝑢1 = (ln 𝑥)2 ⇒ 𝑢1′‬‬ ‫𝑥 = ‪; 𝑣1 = 1 ⇒ 𝑣1‬‬
‫‪1‬‬ ‫𝑥‬
‫𝑒‬ ‫𝑒‬
‫‪2‬‬
‫= 𝑥𝑑 )𝑥 ‪∫ (ln‬‬ ‫𝑒‪[𝑥(ln 𝑥)2 ]1‬‬ ‫𝑥𝑑 𝑥 ‪− 2 ∫ ln‬‬
‫‪⏟1‬‬ ‫‪⏟1‬‬
‫𝐼‬ ‫𝐽‬
‫‪1‬‬
‫𝑥 = ‪𝑢2 = ln 𝑥 ⇒ 𝑢2′ = ; 𝑣2′ = 1 ⇒ 𝑣2‬‬
‫𝑥‬
‫𝑒‬
‫‪𝐽 = [𝑥 ln 𝑥]1𝑒 − ∫ 𝑑𝑥 = [𝑥 ln 𝑥]1𝑒 − [𝑥]1𝑒 = 𝑒 − (𝑒 − 1) = 1‬‬
‫‪1‬‬
‫‪𝐼 = [𝑥(ln 𝑥)2 ]1𝑒 − 2 = 𝑒 − 2‬‬
‫‪ -III‬حساب المساحات ‪:‬‬
‫‪ .1‬وحدة المساحة ‪:‬‬
‫وحدة المساحة في مستوي منسوب إلى معلم متعامد )𝑗 ‪ (𝑂; 𝑖,‬هي مساحة الرباعي المحد ّد بالنقطة 𝑂 والشعاعين 𝑖 و 𝑗‬
‫‖𝑗‖ × ‖𝑖‖ = 𝐴 ‪1𝑢.‬‬

‫‪11‬‬
‫‪ .2‬حساب مساحة ح ّيز ‪:‬‬
‫‪ ‬لتكن 𝑓 دالة مستمرة على مجال ]𝑏 ;𝑎[‪ .‬مساحة الحيّز المحصور بين المنحنى ) ‪ ،(Cf‬محور الفواصل‬
‫والمستقيمين اللذين معادلتاهما ‪ 𝑥 = 𝑎 :‬و 𝑏 = 𝑥 هي ‪:‬‬
‫𝑏‬
‫𝐴 ‪(∫ |𝑓(𝑥)|𝑑𝑥) 𝑢.‬‬
‫𝑎‬
‫لتكن 𝑓 و 𝑔 دالتين مستمرتين على مجال ]𝑏 ;𝑎[‪ .‬مساحة الحيّز المحصور بين المنحنيين ) ‪ (Cf‬و ) ‪ ، (Cg‬محور‬ ‫‪‬‬
‫الفواصل والمستقيمين اللذين معادلتاهما ‪ 𝑥 = 𝑎 :‬و 𝑏 = 𝑥 هي ‪:‬‬
‫𝑏‬
‫𝐴 ‪(∫ |𝑓(𝑥) − 𝑔(𝑥)|𝑑𝑥) 𝑢.‬‬
‫𝑎‬
‫‪ .3‬حاالت خاصة ‪:‬‬
‫مساحة الحيّز الرمادي في الرسم‬ ‫مالحظات‬ ‫الرسم‬

‫) ‪(Cf‬‬
‫𝑏‬
‫𝐴 ‪(∫ 𝑓(𝑥)𝑑𝑥) 𝑢.‬‬ ‫𝑓 موجبة على المجال ]𝑏 ;𝑎[‬
‫𝑎‬

‫𝑏‬
‫𝐴 ‪(∫ −𝑓(𝑥)𝑑𝑥) 𝑢.‬‬ ‫𝑓 سالبة على المجال ]𝑏 ;𝑎[‬
‫𝑎‬
‫) ‪(Cf‬‬

‫𝑐‬ ‫𝑏‬
‫𝑓 موجبة على المجال ]𝑐 ;𝑎[‬ ‫) ‪(Cf‬‬
‫𝐴 ‪(∫ 𝑓(𝑥)𝑑𝑥 + ∫ −𝑓(𝑥)𝑑𝑥) 𝑢.‬‬
‫𝑎‬ ‫𝑐‬ ‫وسالبة على المجال ]𝑏 ;𝑐[‬

‫𝑏‬
‫) ‪(Cg‬‬
‫) ‪ (Cf‬فوق ) ‪ (Cg‬على المجال‬
‫𝐴 ‪(∫ [𝑓(𝑥) − 𝑔(𝑥)]𝑑𝑥 ) 𝑢.‬‬
‫𝑎‬ ‫]𝑏 ;𝑎[‬

‫) ‪(Cf‬‬

‫𝑐‬
‫) ‪(Cg‬‬
‫𝑥𝑑])𝑥(𝑔 ‪∫ [𝑓(𝑥) −‬‬ ‫) ‪ (Cf‬فوق ) ‪ (Cg‬على المجال‬
‫𝑎‬
‫𝑏‬ ‫𝐴 ‪𝑢.‬‬ ‫]𝑐 ;𝑎[ و) ‪ (Cf‬تحت ) ‪(Cg‬‬
‫𝑥𝑑])𝑥(𝑓 ‪+ ∫ [𝑔(𝑥) −‬‬ ‫على المجال ]𝑏 ;𝑐[‬
‫(‬ ‫𝑐‬ ‫)‬
‫) ‪(Cf‬‬

‫‪10‬‬
‫‪ .4‬حساب الحجوم ‪:‬‬
‫حجم المجسّم المولّد بدوران المنحنى ) ‪ (Cf‬حول محور الفواصل دورة كاملة في مجال ]𝑏 ;𝑎[ هو ‪:‬‬
‫𝑏‬
‫‪2‬‬
‫𝑣 ‪𝑉 = [∫ 𝜋(𝑓(𝑥)) 𝑑𝑥] 𝑢.‬‬
‫𝑎‬

‫) ‪(Cf‬‬

‫مثال على حساب المساحات ‪:‬‬


‫‪𝑥 2 −2𝑥−3‬‬
‫‪; 𝑥 ≥ −1‬‬
‫{ = )𝑥(𝑓 وليكن ) ‪ (Cf‬تمثيلها البياني المبيّن أسفله‪.‬‬ ‫‪2‬‬ ‫المعرفة على ‪ ℝ‬كما يلي ‪:‬‬
‫ّ‬ ‫لتكن الدالة 𝑓‬
‫‪𝑥+1‬‬
‫𝑒𝑥‬ ‫‪+ 1 ; 𝑥 ≤ −1‬‬
‫احسب مساحة الحيّز المظلل في الشكل‪.‬‬

‫‪−1‬‬ ‫‪3‬‬
‫‪1‬‬ ‫‪3‬‬
‫∫=𝐴‬ ‫‪(𝑥𝑒 𝑥+1‬‬ ‫𝑥𝑑 ) ‪+ 1)𝑑𝑥 − ∫ ( 𝑥 2 − 𝑥 −‬‬
‫‪⏟−4‬‬ ‫‪⏟−1 2‬‬ ‫‪2‬‬
‫𝐼‬ ‫𝐽‬
‫‪−1‬‬ ‫‪−1‬‬ ‫‪−1‬‬
‫‪𝐼 = ∫ (𝑥𝑒 𝑥+1 + 1)𝑑𝑥 = ∫ 𝑥𝑒 𝑥+1 𝑑𝑥 + ∫ 𝑑𝑥 ; 𝑢 = 𝑥 ⇒ 𝑢′ = 1 ; 𝑣 ′ = 𝑒 𝑥+1 ⇒ 𝑣 = 𝑒 𝑥+1‬‬
‫‪−4‬‬ ‫‪−4‬‬ ‫‪−4‬‬
‫‪−1‬‬
‫‪𝐼 = [𝑥𝑒 𝑥+1 ]−1‬‬
‫𝑒 ∫ ‪−4 −‬‬
‫‪𝑥+1‬‬
‫‪𝑑𝑥 + [𝑥]−1‬‬
‫𝑒𝑥[ = ‪−4‬‬
‫‪𝑥+1 ]−1‬‬
‫𝑒[ ‪−4 −‬‬
‫‪𝑥+1 ]−1‬‬ ‫‪−1‬‬
‫𝑒)‪−4 + [𝑥]−4 = [(𝑥 − 1‬‬
‫‪𝑥+1‬‬
‫‪+ 𝑥]−1‬‬
‫‪−4‬‬
‫‪−4‬‬
‫‪= 5𝑒 −3 + 1‬‬
‫‪3‬‬
‫‪1 2‬‬ ‫‪3‬‬ ‫‪1 3 1 2 3 3‬‬ ‫‪16‬‬
‫‪𝐽 = ∫ ( 𝑥 − 𝑥 − ) 𝑑𝑥 = [ 𝑥 + 𝑥 − 𝑥] = −‬‬
‫‪−1 2‬‬ ‫‪2‬‬ ‫‪6‬‬ ‫‪2‬‬ ‫‪2 −1‬‬ ‫‪3‬‬
‫‪16‬‬ ‫‪19‬‬
‫‪𝐴 = 𝐼 − 𝐽 = 5𝑒 −3 + 1 +‬‬ ‫‪= 5𝑒 −3 +‬‬ ‫𝑎 ‪𝑢.‬‬
‫‪3‬‬ ‫‪3‬‬

‫‪‬‬
‫‪‬‬

‫‪12‬‬
‫‪ -1‬إيجاد القيم الممكنة للقاسم المشترك األكبر ‪:‬‬
‫إليجاد القيم الممكنة لـ )𝑏 ;𝑎(𝐷𝐶𝐺𝑃 = 𝑑 ‪ ،‬نبحث عن عالقة بين 𝑎 و 𝑏 مستقلة عن 𝑛‬
‫مثال ‪𝑏 = 5𝑛 − 2 ; 𝑎 = 2𝑛 + 3 :1‬‬
‫)𝑏 ;𝑎(𝐷𝐶𝐺𝑃 = 𝑑 يعني 𝑑 يقسم 𝑎 و 𝑑 يقسم 𝑏 ‪ ،‬منه 𝑑 يقسم 𝑏‪ 5𝑎 − 2‬أي 𝑑 يقسم ‪19‬‬
‫فالقيم الممكنة لـ 𝑑 هي قواسم ‪ 19‬أي ‪ 1‬و ‪.19‬‬
‫مثال ‪𝑏 = 𝑛2 + 2 ; 𝑎 = 5𝑛2 + 7 :2‬‬
‫)𝑏 ;𝑎(𝐷𝐶𝐺𝑃 = 𝑑 يعني 𝑑 يقسم 𝑎 و 𝑑 يقسم 𝑏 ‪ ،‬منه 𝑑 يقسم 𝑏‪ −𝑎 + 5‬أي 𝑑 يقسم ‪3‬‬
‫فالقيم الممكنة لـ 𝑑 هي قواسم ‪ 3‬أي ‪ 1‬و ‪.3‬‬
‫‪ -2‬إيجاد قيم 𝒏 التي من أجلها يأخذ 𝒅 قيمة مع ّينة‪:‬‬
‫إليجاد قيم 𝑛 التي من أجلها يأخذ 𝑑 قيمة معيّنة (غالبا القيمة المختلفة عن ‪ )1‬نستعمل الموافقات على النحو التالي‪:‬‬
‫مثال ‪( 1‬السابق)‪ 𝑑 = 1 ، 𝑏 = 5𝑛 − 2 ; 𝑎 = 2𝑛 + 3 :‬أو ‪𝑑 = 19‬‬
‫نبحث عن قيم 𝑛 التي من أجلها ‪: 𝑑 = 19‬‬
‫أن 𝑎 و 𝑏 من مضاعفات ‪ 19‬أي ‪:‬‬ ‫‪ 𝑑 = 19‬يعني ّ‬
‫]‪𝑎 ≡ 0[19‬‬ ‫]‪2𝑛 + 3 ≡ 0[19‬‬
‫{ ⇒ ‪𝑑 = 19‬‬ ‫{⇒‬ ‫⇒‬ ‫]‪3𝑛 − 5 ≡ 0[19] ⇒ 3𝑛 ≡ 5[19‬‬
‫]‪𝑏 ≡ 0[19‬‬ ‫نطرح الموافقتين ]‪5𝑛 − 2 ≡ 0[19‬‬
‫إليجاد قيم 𝑛 التي تحقق الموافقة السابقة نستعمل إحدى الطريقتين ‪:‬‬
‫الطريقة األولى ‪ :‬الجدول‬
‫ّ‬
‫نعطي لـ 𝑛 القيم من ‪ 1‬إلى ‪( 18‬الترديد ‪ ، )1-‬ث ّم نضرب هذه القيم في ‪ 3‬وكلما زاد العدد عن الترديد (أو أحد مضاعفاته) نطرح منه‬
‫الترديد (أو أحد مضاعفاته)‬
‫‪𝑛≡ 0 1 2 3 4 5 6 7 8‬‬ ‫‪9‬‬
‫‪3𝑛 ≡ 0 3 6 9 12 15 18 2 5‬‬ ‫‪8‬‬
‫‪𝑛 ≡ 10 11 12 13 14 15 16 17 18‬‬
‫]‪[19‬‬
‫‪3𝑛 ≡ 11 14 17 1 4 7 10 13 16‬‬
‫‪3𝑛 ≡ 5[19] ⇒ 𝑛 ≡ 8[19] ⇒ 𝑛 = 19𝑘 + 8 ; 𝑘 ∈ ℕ‬‬

‫الطريقة الثانية ‪ :‬إضافة الترديد إلى الطرف الثاني حتى يقبل القسمة على الطرف األول‬
‫‪3𝑛 ≡ 5[19] ⇒ 3𝑛 ≡ 5 + 19[19] ⇒ 3𝑛 ≡ 24[19] ⇒ 𝑛 ≡ 8[19] ⇒ 𝑛 = 19𝑘 + 8 ; 𝑘 ∈ ℕ‬‬
‫مالحظة هامة ‪ :‬تستعمل هذه الطريقة فقط إذا كان الطرف األول أولي مع الترديد (‪ 3‬مع ‪)11‬‬
‫مثال ‪( 2‬السابق)‪ 𝑑 = 1 ، 𝑏 = 𝑛2 + 2 ; 𝑎 = 5𝑛2 + 7 :‬أو ‪𝑑 = 3‬‬
‫نبحث عن قيم 𝑛 التي من أجلها ‪: 𝑑 = 3‬‬
‫أن 𝑎 و 𝑏 من مضاعفات ‪ 3‬أي ‪:‬‬‫‪ 𝑑 = 3‬يعني ّ‬
‫‪2‬‬
‫]‪𝑎 ≡ 0[3‬‬ ‫]‪5𝑛 + 7 ≡ 0[3‬‬
‫{‬ ‫‪⇒{ 2‬‬ ‫⇒‬ ‫⏟ ‪4 𝑛2 +‬‬
‫⏟‬ ‫]‪5 ≡ 0[3‬‬
‫]‪𝑏 ≡ 0[3‬‬ ‫]‪ ≡1[3‬نطرح الموافقتين ]‪𝑛 + 2 ≡ 0[3‬‬ ‫]‪≡2[3‬‬

‫⇒ ]‪⇒ 𝑛2 + 2 ≡ 0[3] ⇒ 𝑛2 ≡ −2[3‬‬ ‫]‪𝑛2 ≡ 1[3‬‬


‫نضيف الترديد‬
‫إليجاد قيم 𝑛 التي تحقق الموافقة السابقة نستعمل إحدى الطريقتين السابقتين باإلضافة إلى طريقة ثالثة عندما يكون 𝑛 من الدرجة‬
‫الثانية والترديد أوليا ‪:‬‬
‫ط‪ :1‬الجدول‬
‫]‪ 𝑛 ≡ 2[3‬أو ]‪𝑛 ≡ 0 1 2 [3] 𝑛2 ≡ 1[3] ⇒ 𝑛 ≡ 1[3‬‬
‫‪ 𝑛 = 3𝑘 + 2 ; 𝑘 ∈ ℕ‬أو ‪𝑛2 ≡ 0 1 1 [3] ⇒ 𝑛 = 3𝑘 + 1‬‬

‫‪11‬‬
‫ط‪ :2‬إضافة الترديد حتى نحصل على مربع تام‬
‫]‪ 𝑛 ≡ 1[3‬أو ]‪ 𝑛 ≡ −2[3] ⇒ 𝑛 ≡ 2[3‬أو ]‪𝑛2 ≡ 1[3] ⇒ 𝑛2 ≡ 4[3] ⇒ 𝑛 ≡ 2[3‬‬
‫ط‪ :3‬استعمال المتطابقات الشهيرة‬
‫‪2‬‬ ‫‪2‬‬
‫]‪𝑛 ≡ 1[3] ⇒ 𝑛 − 1 ≡ 0[3] ⇒ (𝑛 − 1)(𝑛 + 1) ≡ 0[3‬‬
‫]‪ 𝑛 + 1 ≡ 0[3‬أو ]‪⇒ 𝑛 − 1 ≡ 0[3‬‬
‫]‪ 𝑛 ≡ 2[3‬أو ]‪ 𝑛 ≡ −1[3] ⇒ 𝑛 ≡ 1[3‬أو ]‪⇒ 𝑛 ≡ 1[3‬‬
‫‪ -3‬إيجاد قيم 𝒏 ل ّما يكون الترديد مجهوال ‪:‬‬
‫إليجاد قيم 𝑛 ل ّما يكون الترديد مجهوال نكتب الموافقة عل الشكل ‪] :‬الترديد[‪ ≡ 0‬عدد‬
‫مثال‪𝑛 + 9 ≡ 0[𝑛 + 1] :1‬‬
‫]‪𝑛 + 9 ≡ 0[𝑛 + 1] ⇒ 𝑛 + 9 ≡ 𝑛 + 1[𝑛 + 1] ⇒ (𝑛 + 9) − (𝑛 + 1) ≡ 0[𝑛 + 1‬‬
‫)‪ (𝑛 + 1‬قاسم لـ ‪ 8 ⇒ 8‬مضاعف لـ )‪⇒ 8 ≡ 0[𝑛 + 1] ⇒ (𝑛 + 1‬‬
‫}‪(𝑛 + 1) ∈ 𝐷8 ⇒ (𝑛 + 1) ∈ {1; 2; 4; 8} ⇒ 𝑛 ∈ {0; 1; 3; 7‬‬
‫مالحظة هامة ‪ :‬لتحديد القواسم ينبغي دائما التأ ّكد من السؤال إن كان المجهول طبيعيا أم صحيحا‪ ،‬فإن كان المجهول طبيعيا نكتفي‬
‫بالقواسم الطبيعية )}‪ ، (𝐷8 = {1; 2; 4; 8‬أ ّما إن كان المجهول صحيحا نأخذ القواسم الصحيحة‬
‫)}‪(𝐷8 = {−8; −4; −2; −1; 1; 2; 4; 8‬‬
‫مثال‪𝑛2 + 3𝑛 + 8 ≡ 0[𝑛 + 1] :2‬‬
‫‪2‬‬ ‫‪2‬‬ ‫‪2‬‬
‫]‪𝑛 + 3𝑛 + 8 ≡ 0[𝑛 + 1] ⇒ 𝑛 + 3𝑛 + 8 ≡ (𝑛 + 1) + (𝑛 + 1)[𝑛 + 1‬‬
‫}‪⇒ 𝑛2 + 3𝑛 + 8 ≡ 𝑛2 + 3𝑛 + 2[𝑛 + 1] ⇒ 6 ≡ 0[𝑛 + 1] ⇒ (𝑛 + 1) ∣ 6 ⇒ (𝑛 + 1) ∈ {1; 2; 3; 6‬‬
‫}‪⇒ 𝑛 ∈ {0; 1; 2; 5‬‬
‫; ‪𝑛2 + 3𝑛 + 8 = (𝑛 + 1)(𝑛 + 2) + 6‬‬ ‫طريقة ثانية‪:‬‬
‫‪𝑛2 + 3𝑛 + 8 ≡ 0[𝑛 + 1] ⇒ 6 ≡ 0[𝑛 + 1] ⇒ (𝑛 + 1) ∣ 6‬‬
‫}‪⇒ (𝑛 + 1) ∈ {1; 2; 3; 6} ⇒ 𝑛 ∈ {0; 1; 2; 5‬‬
‫‪ -4‬إيجاد باقي قسمة عدد طبيعي على آخر‪:‬‬
‫‪0‬‬
‫أن باقي 𝑎 على 𝑏‬‫بواقي القسمة اإلقليدية للعدد الطبيعي 𝑛𝑎 على 𝑏 تكون دورية‪ ،‬أي أنها تتكرر من أجل قيم معينّة للعدد 𝑛‪ ،‬وبما ّ‬
‫يكون دائما ‪ ، 1‬نحسب بواقي قسمة 𝑛𝑎 على 𝑏 حتى نحصل على قيمة للعدد 𝑛 حيث باقي 𝑛𝑎 على 𝑏 يساوي ‪ ،1‬ويكون الدور‬
‫حينئذ هو 𝑛‪.‬‬
‫𝑛‬
‫مثال‪ :1‬دراسة حسب قيم العدد الطبيعي ‪ n‬بواقي قسمة العدد ‪ 4‬على ‪7‬‬
‫‪0‬‬ ‫‪1‬‬ ‫‪2‬‬ ‫‪3‬‬
‫]‪4 ≡ 1[7] ; 4 ≡ 4[7] ; 4 ≡ 2[7] ; 4 ≡ 1[7‬‬
‫]‪43𝑘 ≡ 1[7] ; 43𝑘+1 ≡ 4[7] ; 43𝑘+2 ≡ 2[7‬‬
‫مثال‪ :2‬دراسة حسب قيم العدد الطبيعي ‪ n‬بواقي قسمة العدد 𝑛‪ 5‬على ‪7‬‬
‫]‪50 ≡ 1[7] ; 51 ≡ 5[7] ; 52 ≡ 4[7] ; 53 ≡ 6[7] ; 54 ≡ 2[7] ; 55 ≡ 3[7] ; 56 ≡ 1[7‬‬
‫]‪56𝑘 ≡ 1[7] ; 56𝑘+1 ≡ 5[7]; 56𝑘+2 ≡ 4[7]; 56𝑘+3 ≡ 6[7]; 56𝑘+4 ≡ 2[7]; 56𝑘+5 ≡ 3[7‬‬
‫حالة خاصة‪ :‬دراسة بواقي القسمة اإلقليدية للعدد الطبيعي 𝑛‪ 2‬على ‪10‬‬
‫; ]‪20 ≡ 1[10] ; 21 ≡ 2[10] ; 22 ≡ 4[10] ; 23 ≡ 8[10] ; 24 ≡ 6[10‬‬
‫; ]‪25 ≡ 2[10] ; 26 ≡ 4[10] ; 27 ≡ 8[10] ; 28 ≡ 6[10‬‬
‫(ألن العدد 𝑛‪ 2‬زوجي من أجل كل ∗‪ ، )𝑛 ∈ ℕ‬لهذا السبب نتوقف عند تكرار أول باقي يختلف عن ‪1‬‬ ‫ّ‬ ‫أن الباقي ‪ 1‬لن يتكرر‬‫نالحظ ّ‬
‫(أي ‪ )2‬ويكون الدور ‪.4‬‬
‫𝑘‪4‬‬ ‫‪4𝑘+1‬‬ ‫‪4𝑘+2‬‬ ‫‪4𝑘+3‬‬ ‫∗‬
‫‪2 ≡ 6[10] ; 2‬‬ ‫‪≡ 2[10] ; 2‬‬ ‫‪≡ 4[10] ; 2‬‬ ‫‪≡ 8[10] ; 𝑘 ∈ ℕ‬‬
‫𝑛‬
‫بعد دراسة حسب قيم العدد الطبيعي ‪ n‬بواقي قسمة العدد 𝑎 على 𝑏 يُطلب منكم إيجاد باقي قسمة عدد 𝑐 مرفوع بقوة على 𝑏 وهنا‬
‫نميز الحاالت التالية ‪:‬‬
‫‪𝒄 = 𝒂 .1‬‬
‫]‪42014 = 43(671)+1 ⇒ 42014 ≡ 4[7] ; 51435 = 56(239)+1 ⇒ 51435 ≡ 5[7‬‬

‫‪11‬‬
‫‪𝒄 ≡ 𝒂[𝒃] .2‬‬
‫]‪2013 ≡ 4[7] ⇒ 20131434 ≡ 43(478) [7] ⇒ 20131434 ≡ 1[7‬‬
‫]‪2014 ≡ 5[7] ⇒ 20141995 ≡ 56(332)+3 [7] ⇒ 20141995 ≡ 6[7‬‬
‫‪ 𝒄 ≡ 𝟏[𝒃] .3‬أو ]𝒃[𝟏‪𝒄 ≡ −‬‬
‫]‪ 62𝑛+1 ≡ −1[7‬و ]‪8 ≡ 1[7] ⇒ 8𝑛 ≡ 1[7] ; 6 ≡ −1[7] ⇒ 62𝑛 ≡ 1[7‬‬
‫‪ .4‬قوة العدد 𝒄 مختلفة عن الدور‬
‫]‪46𝑘+5 = 43(2𝑘+1)+2 ⇒ 46𝑘+5 ≡ 2[7]; 518𝑘+10 = 56(3𝑘+1)+4 ⇒ 518𝑘+9 ≡ 2[7‬‬
‫أخيرا يُطلب منكم تعيين قيم العدد الطبيعي 𝑛 حتى يقبل عدد ‪ 𝑐′‬القسمة على 𝑏‪ .‬في هذه الحالة نبسط العدد ‪ 𝑐′‬ونح ّل الموافقة باستعمال‬
‫الطرق المذكورة سابقا‪.‬‬
‫‪6𝑛+3‬‬ ‫‪6𝑛+4‬‬ ‫‪2‬‬
‫‪19‬‬ ‫‪+ 26‬‬ ‫مثال‪ :‬تعيين قيم العدد الطبيعي ‪ n‬حيث ‪+ 4𝑛 + 4 ≡ 0[7] :‬‬
‫‪6𝑛+3‬‬ ‫‪6𝑛+3‬‬ ‫‪6𝑛+3‬‬
‫‪19 ≡ 5[7] ⇒ 19‬‬ ‫‪≡5‬‬ ‫‪[7] ⇒ 19‬‬ ‫]‪≡ 6[7‬‬
‫‪6𝑛+4‬‬ ‫‪6𝑛+4‬‬ ‫‪6𝑛+4‬‬
‫‪26 ≡ 5[7] ⇒ 26‬‬ ‫‪≡5‬‬ ‫‪[7] ⇒ 26‬‬ ‫]‪≡ 2[7‬‬
‫‪6𝑛+3‬‬ ‫‪6𝑛+4‬‬
‫‪19‬‬ ‫‪+ 26‬‬ ‫]‪+ 4𝑛 + 4 ≡ 0[7] ⇒ 8 + 4𝑛2 + 4 ≡ 0[7] ⇒ 4𝑛2 + 12 ≡ 0[7‬‬
‫‪2‬‬

‫)ألن ‪ 7‬أولي مع ‪⇒ 4(𝑛2 + 3) ≡ 0[7] ⇒ 𝑛2 + 3 ≡ 0[7](4‬‬ ‫ّ‬ ‫]‪⇒ 𝑛2 ≡ 4[7] ⇒ 𝑛2 − 4 ≡ 0[7‬‬


‫‪ 𝑛 = 7𝑘 + 5‬أو ‪ 𝑛 ≡ 5[7] ⇒ 𝑛 = 7𝑘 + 2‬أو ]‪⇒ (𝑛 − 2)(𝑛 + 2) ≡ 0[7] ⇒ 𝑛 ≡ 2[7‬‬
‫‪ .5‬العبارة تشتمل على عددين دور باقي قسمتهما على الترديد مختلف‬
‫مثال‪:‬‬
‫دراسة حسب قيم العدد الطبيعي ‪ n‬بواقي قسمة العددين 𝑛‪ 4‬و 𝑛‪ 8‬على ‪9‬‬
‫‪43𝑘 ≡ 1[9]; 43𝑘+1 ≡ 4[9]; 43𝑘+2‬‬ ‫‪≡ 7[9]; 𝑘 ∈ ℕ‬‬
‫‪82𝑘 ≡ 1[9]; 82𝑘+1 ≡ 8[9] ; 𝑘 ∈ ℕ‬‬
‫تعيين قيم العدد الطبيعي ‪ n‬حتى يكون ‪ 6‬باقي قسمة ) 𝑛‪ (2006𝑛 − 2002‬على ‪9‬‬
‫]‪2006𝑛 ≡ 8𝑛 [9]; 2002𝑛 ≡ 4𝑛 [9]; 2006𝑛 − 2002𝑛 ≡ 6[9] ⇒ 8𝑛 − 4𝑛 ≡ 6[9‬‬
‫بما أن بواقي قسمة 𝑛‪ 4‬و 𝑛‪ 8‬على ‪ 9‬لها دورين مختلفين (‪ 2‬و ‪ ، )1‬نأخذ المضاعف المشترك األصغر لهذين الدورين ‪:‬‬
‫=𝑛‬ ‫𝑘‪6‬‬ ‫‪6𝑘 + 1 6𝑘 + 2 6𝑘 + 3 𝟔𝒌 + 𝟒 6𝑘 + 5‬‬
‫𝑛‬ ‫]‪[9‬‬
‫≡ ‪8‬‬ ‫‪1‬‬ ‫‪8‬‬ ‫‪1‬‬ ‫‪8‬‬ ‫𝟏‬ ‫‪8‬‬
‫𝑛‬ ‫]‪[9‬‬
‫≡ ‪4‬‬ ‫‪1‬‬ ‫‪4‬‬ ‫‪7‬‬ ‫‪1‬‬ ‫𝟒‬ ‫‪7‬‬
‫𝑛‬ ‫𝑛‬ ‫]‪[9‬‬
‫‪8 −4 ≡ 0‬‬ ‫‪4‬‬ ‫‪3‬‬ ‫‪7‬‬ ‫𝟔‬ ‫‪1‬‬
‫‪8𝑛 − 4𝑛 ≡ 6[9] ⇒ 𝑛 = 6𝑘 + 4 ; 𝑘 ∈ ℕ‬‬
‫‪ .6‬تعيين الثنائيات )𝒚 ;𝒙( التي تحقّق‪] :‬الترديد[𝒄 ≡ 𝒚𝒃 ‪𝒂𝒙 +‬‬
‫إليجاد الثنائيات )𝑦 ;𝑥( نستعمل الجدول المتقاطع )‪(𝑇𝑎𝑏𝑙𝑒𝑎𝑢 𝑐𝑟𝑜𝑖𝑠é‬‬
‫مثال‪:‬‬
‫دراسة حسب قيم العدد الطبيعي ‪ n‬بواقي قسمة كل من 𝑛‪ 3‬و 𝑛‪ 5‬على ‪16‬‬
‫‪34𝑘 ≡ 1[16] ; 34𝑘+1‬‬ ‫]‪≡ 3[16] ; 34𝑘+2 ≡ 9[16] ; 34𝑘+3 ≡ 11[16‬‬
‫‪54𝑘 ≡ 1[16] ; 54𝑘+1‬‬ ‫]‪≡ 5[16] ; 54𝑘+2 ≡ 9[16] ; 54𝑘+3 ≡ 13[16‬‬
‫تعيين جميع الثنائيات )𝑦 ‪ (𝑥,‬من األعداد الطبيعية حيث ‪3𝑥 + 5𝑦 ≡ 0[16] :‬‬
‫=𝑥‬ ‫𝑘‪4‬‬ ‫‪4𝑘 + 1‬‬ ‫‪4𝑘 + 2‬‬ ‫‪4𝑘 + 3‬‬
‫≡ 𝑥‪3‬‬
‫=𝑦‬ ‫‪1‬‬ ‫‪3‬‬ ‫‪9‬‬ ‫‪11‬‬
‫≡ 𝑦‪5‬‬
‫‪4𝑘′‬‬ ‫‪1‬‬ ‫‪2‬‬ ‫‪4‬‬ ‫‪10‬‬ ‫‪12‬‬
‫‪4𝑘′ + 1‬‬ ‫‪5‬‬ ‫‪6‬‬ ‫‪8‬‬ ‫‪14‬‬ ‫‪0‬‬
‫‪4𝑘′ + 2‬‬ ‫‪9‬‬ ‫‪10‬‬ ‫‪12‬‬ ‫‪2‬‬ ‫‪4‬‬
‫‪4𝑘′ + 3‬‬ ‫‪13‬‬ ‫‪14‬‬ ‫‪0‬‬ ‫‪6‬‬ ‫‪8‬‬
‫‪(𝑥, 𝑦) ∈ {(4𝑘 + 1; 4𝑘 ′ + 3), (4𝑘 + 3; 4𝑘 ′ + 1)} ; (𝑘, 𝑘′) ∈ ℕ2‬‬

‫‪11‬‬
‫‪ -5‬حل في ‪ ℤ‬المعادلة 𝒄 = 𝒚𝒃 ‪:𝒂𝒙 +‬‬
‫‪ .1‬تقبل هذه المعادلة حلوال في ‪ ℤ‬ل ّما يقسم )𝑏 ;𝑎(𝐷𝐶𝐺𝑃 العدد 𝑐‬
‫مثال ‪ :1‬المعادلة ‪ 7𝑥 + 21𝑦 = 3‬ال تقبل حلوال في ‪ّ ℤ‬‬
‫ألن ‪ 𝑃𝐺𝐶𝐷(7; 21) = 7‬ال يقسم ‪3‬‬
‫مثال ‪ :2‬المعادلة ‪ 6𝑥 − 5𝑦 = 2‬تقبل حلوال في ‪ّ ℤ‬‬
‫ألن ‪ 𝑃𝐺𝐶𝐷(6; 5) = 1‬يقسم ‪2‬‬
‫‪ .2‬إليجاد الحل الخاص نستعمل خوارزمية إقليدس‬
‫مثال‪ :‬لنبحث عن حل خاص للمعادلة ‪27𝑥 + 22𝑦 = 1‬‬
‫‪27 = 22 + 5‬‬ ‫‪⇒ 5 = 27 − 22‬‬
‫)‪22 = 4(5) + 2 ⇒ 2 = 22 − 4(5‬‬
‫‪5 = 2(2) + 1‬‬ ‫)‪⇒ 1 = 5 − 2(2‬‬
‫)‪1 = 5 − 2(2) = 5 − 2[22 − 4(5)] = 9(5) − 2(22‬‬
‫)‪1 = 9(27 − 22) − 2(22) = 27(9) + 22(−11) ⇒ (𝑥0 ; 𝑦0 ) = (9; −11‬‬
‫مالحظة هامة ‪ :‬إذا كانت الثنائية ) ‪ (𝑥0 ; 𝑦0‬حال خاصا للمعادلة 𝑐 = 𝑦𝑏 ‪ّ 𝑎𝑥 +‬‬
‫فإن الثنائية ) ‪ (𝑛𝑥0 ; 𝑛𝑦0‬حال خاصا للمعادلة‬
‫𝑐𝑛 = 𝑦𝑏 ‪𝑎𝑥 +‬‬
‫مثال‪ :‬نالحظ ّ‬
‫أن )‪ (1; 1‬حل خاص للمعادلة ‪ ، 6𝑥 − 5𝑦 = 1‬منه الثنائية )‪ (2; 2‬حل خاص للمعادلة ‪.6𝑥 − 5𝑦 = 2‬‬
‫‪ .3‬لحل المعادلة نستعمل مبرهنة غوص‬
‫مثال‪ :‬حل في ‪ ℤ‬للمعادلة ‪6𝑥 − 5𝑦 = 2‬‬
‫‪6𝑥 − 5𝑦 = 2‬‬
‫{‬ ‫)‪⇒ 6(𝑥 − 2) − 5(𝑦 − 2) = 0 ⇒ 6(𝑥 − 2) = 5(𝑦 − 2‬‬
‫‪6(2) − 5(2) = 2‬‬
‫بما ّ‬
‫أن ‪ 5‬يقسم )‪ 6(𝑥 − 2‬و ‪ 5‬أولي مع ‪ ، 6‬إذن ‪ 5‬يقسم )‪ (𝑥 − 2‬منه ‪𝑥 = 5𝑘 + 2‬‬
‫بالتعويض في العبارة السابقة نجد‪ 6(5𝑘) = 5(𝑦 − 2) :‬منه ‪𝑦 = 6𝑘 + 2‬‬
‫حلول المعادلة هي ‪𝑆 = {(5𝑘 + 2; 6𝑘 + 2)} ; 𝑘 ∈ ℤ :‬‬
‫مالحظة‪ :‬حلول المعادلة 𝑐 = 𝑦𝑏 ‪ 𝑎𝑥 −‬تكون من الشكل ) ‪ ،(𝑏𝑘 + 𝑥0 ; 𝑎𝑘 + 𝑦0‬أ ّما حلول المعادلة 𝑐 = 𝑦𝑏 ‪ 𝑎𝑥 +‬تكون من الشكل‬
‫) ‪ (𝑏𝑘 + 𝑥0 ; −𝑎𝑘 + 𝑦0‬أو من الشكل ) ‪(−𝑏𝑘 + 𝑥0 ; 𝑎𝑘 + 𝑦0‬‬
‫مثال‪ :1‬حلول المعادلة ‪ 27𝑥 − 22𝑦 = 1‬ذات الحل الخاص )‪ (9; 11‬هي ‪:‬‬
‫‪𝑆 = {(22𝑘 + 9 ; 27𝑘 + 11)} ; 𝑘 ∈ ℤ‬‬
‫مثال‪ :2‬حلول المعادلة ‪ 11𝑥 + 7𝑦 = 2‬ذات الحل الخاص )‪ (−3; 5‬هي ‪:‬‬
‫‪𝑆 = {(7𝑘 − 3 ; −11𝑘 + 5)} = {(−7𝑘 − 3 ; 11𝑘 + 5)} ; 𝑘 ∈ ℤ‬‬

‫‪ .4‬يمكن حل المعادلة باستعمال الموافقات‬


‫مثال‪ :1‬حل المعادلة ‪4𝑥 − 9𝑦 = 19‬‬
‫‪4𝑥 − 9𝑦 = 19 ⇒ 4𝑥 = 9𝑦 + 19 ⇒ 9𝑦 + 19 ≡ 0[4] ⇒ 𝑦 + 3 ≡ 0[4] ⇒ 𝑦 ≡ 1[4] ⇒ 𝑦 = 4𝑘 + 1‬‬
‫‪4𝑥 = 9(4𝑘 + 1) + 19 = 36𝑘 + 28 = 4(9𝑘 + 7) ⇒ 𝑥 = 9𝑘 + 7‬‬
‫‪𝑆 = {(9𝑘 + 7; 4𝑘 + 1)} ; 𝑘 ∈ ℤ‬‬

‫‪ -6‬حل المعادالت المشتملة على 𝒎 و 𝒅 ‪:‬‬


‫لحل المعادالت المشتملة على )𝑏 ;𝑎(𝑀𝐶𝑃𝑃 = 𝑚 و)𝑏 ;𝑎(𝐷𝐶𝐺𝑃 = 𝑑 نتبع الخطوات التالية‬
‫‪ .1‬كتابة 𝑎 و 𝑏 بداللة ‪ 𝑎′‬و ‪𝑏′‬‬
‫‪′‬‬ ‫‪′‬‬ ‫)‪′ ′‬‬
‫‪𝑑 = 𝑃𝐺𝐶𝐷(𝑎; 𝑏) ⇒ 𝑎 = 𝑑𝑎 ; 𝑏 = 𝑑𝑏 ; 𝑃𝐺𝐶𝐷(𝑎 ; 𝑏 = 1‬‬
‫‪ .2‬إيجاد عالقة بين كتابة 𝑚 و 𝑑 ‪ 𝑎′ ،‬و ‪𝑏′‬‬
‫‪𝑚 × 𝑑 = 𝑎 × 𝑏 ⇒ 𝑚 × 𝑑 = 𝑑𝑎′ × 𝑑𝑏′ ⇒ 𝑚 = 𝑑𝑎′𝑏′‬‬
‫‪ .3‬تعيين القيم الممكنة لـ ‪ 𝑎′‬و‪ 𝑏′‬مع مراعاة الشرط ‪ ، 𝑃𝐺𝐶𝐷(𝑎′ ; 𝑏 ′ ) = 1‬ث ّم استنتاج القيم الممكنة لـ 𝑎 و𝑏‬

‫‪11‬‬
‫مثال‪:‬‬
‫عيّن كل الثنائيات )𝑏 ‪ (𝑎,‬من األعداد الطبيعية حيث ‪:‬‬
‫‪𝑎 + 𝑏 = 420‬‬
‫{ )‪1‬‬ ‫‪…  ; 𝑎 = 84𝑎′ ; 𝑏 = 84𝑏 ′ ; 𝑃𝐺𝐶𝐷(𝑎′ , 𝑏 ′ ) = 1‬‬
‫‪𝑃𝐺𝐶𝐷(𝑎, 𝑏) = 84‬‬
‫‪ ⇒ 84𝑎′ + 84𝑏′ = 420 ⇒ 84(𝑎′ + 𝑏 ′ ) = 420 ⇒ 𝑎′ + 𝑏 ′ = 5‬‬
‫})‪⇒ (𝑎′ , 𝑏 ′ ) ∈ {(1,4); (4,1); (2,3); (3,2)} ⇒ (𝑎, 𝑏) ∈ {(84,336); (336,84); (168,252); (252,168‬‬
‫‪𝑎 × 𝑏 = 360‬‬
‫{ )‪2‬‬ ‫‪…  ; 𝑎 = 6𝑎′ ; 𝑏 = 6𝑏 ′ ; 𝑃𝐺𝐶𝐷(𝑎′ , 𝑏 ′ ) = 1‬‬
‫‪𝑃𝐺𝐶𝐷(𝑎, 𝑏) = 6‬‬
‫‪ ⇒ 6𝑎′ × 6𝑏 ′ = 360 ⇒ 36(𝑎′ × 𝑏 ′ ) = 360 ⇒ 𝑎′ × 𝑏 ′ = 10‬‬
‫})‪⇒ (𝑎′ , 𝑏 ′ ) ∈ {(1,10); (10,1); (2,5); (5,2)} ⇒ (𝑎, 𝑏) ∈ {(6,60); (60,6); (12,30); (30,12‬‬
‫‪𝑎2 − 𝑏 2 = 825‬‬
‫{ )‪3‬‬ ‫‪…  ; 𝑎 = 5𝑎′ ; 𝑏 = 5𝑏 ′ ; 𝑃𝐺𝐶𝐷(𝑎′ , 𝑏 ′ ) = 1‬‬
‫‪𝑃𝐺𝐶𝐷(𝑎, 𝑏) = 5‬‬
‫‪2‬‬ ‫‪2‬‬ ‫‪2‬‬ ‫‪2‬‬
‫‪ ⇒ (5𝑎′ )2 − (5𝑏 ′ )2 = 825 ⇒ 25(𝑎′ − 𝑏 ′ ) = 825 ⇒ 𝑎′ − 𝑏 ′ = 33 ⇒ (𝑎′ − 𝑏 ′ )(𝑎′ + 𝑏 ′ ) = 33‬‬
‫‪𝑎′ − 𝑏 ′ = 1‬‬
‫‪{ ′‬‬ ‫)‪⇒ 2𝑎′ = 34 ⇒ 𝑎′ = 17 ⇒ 𝑏 ′ = 16 ⇒ (𝑎, 𝑏) = (85,80‬‬
‫‪𝑎 + 𝑏 ′ = 33‬‬
‫‪𝑎′ − 𝑏 ′ = 3‬‬
‫‪{ ′‬‬ ‫)‪⇒ 2𝑎′ = 14 ⇒ 𝑎′ = 7 ⇒ 𝑏 ′ = 4 ⇒ (𝑎, 𝑏) = (35,20‬‬
‫‪𝑎 + 𝑏 ′ = 11‬‬
‫‪′‬‬ ‫‪′‬‬ ‫‪′‬‬ ‫‪′‬‬
‫مالحظة‪ :‬الحالتان ‪ {𝑎 ′− 𝑏 ′ = 33‬و ‪ {𝑎 ′− 𝑏 ′ = 11‬مرفوضتان ّ‬
‫ألن ‪ 𝑎′ ( 𝑎′ + 𝑏 ′ > 𝑎′ − 𝑏 ′‬و ‪ 𝑏 ′‬طبيعيان)‬
‫‪𝑎 +𝑏 =3‬‬ ‫‪𝑎 +𝑏 =1‬‬
‫‪𝑃𝑃𝐶𝑀(𝑎, 𝑏) = 90‬‬
‫{ )‪4‬‬ ‫‪…  ; 𝑎 = 18𝑎′ ; 𝑏 = 18𝑏 ′ ; 𝑃𝐺𝐶𝐷(𝑎′ , 𝑏 ′ ) = 1‬‬
‫‪𝑃𝐺𝐶𝐷(𝑎, 𝑏) = 18‬‬
‫‪𝑚 90‬‬
‫= = ‪𝑚. 𝑑 = 𝑎. 𝑏 = 𝑑𝑎′ . 𝑑𝑏 ′ = 𝑑2 𝑎′ 𝑏 ′ ⇒ 𝑚 = 𝑑𝑎′ 𝑏 ′ ⇒ 𝑎′ 𝑏 ′‬‬ ‫‪=5‬‬
‫‪𝑑 18‬‬
‫})‪(𝑎′ , 𝑏 ′ ) ∈ {(1,5); (5,1)} ⇒ (𝑎, 𝑏) ∈ {(18,90); (90,18‬‬

‫‪𝑃𝑃𝐶𝑀(𝑎, 𝑏) − 9𝑃𝐺𝐶𝐷(𝑎, 𝑏) = 13‬‬


‫{ )‪5‬‬ ‫‪…  ; 𝑎 = 𝑑𝑎′ ; 𝑏 = 𝑑𝑏 ′ ; 𝑃𝐺𝐶𝐷(𝑎′ , 𝑏 ′ ) = 1‬‬
‫𝑏<𝑎‬
‫}‪ ⇒ 𝑚 − 9𝑑 = 13 ⇒ 𝑑𝑎′ 𝑏 ′ − 9𝑑 = 13 ⇒ 𝑑(𝑎′ 𝑏 ′ − 9) = 13 ⇒ 𝑑 ∣ 13 ⇒ 𝑑 ∈ {1,13‬‬
‫})‪𝑑 = 1: 𝑎′ 𝑏 ′ − 9 = 13 ⇒ 𝑎′ 𝑏 ′ = 22 ⇒ (𝑎′ , 𝑏 ′ ) ∈ {(1,22); (2,11)} ⇒ (𝑎, 𝑏) ∈ {(1,22); (2,11‬‬
‫})‪𝑑 = 13: 𝑎′ 𝑏 ′ − 9 = 1 ⇒ 𝑎′ 𝑏 ′ = 10 ⇒ (𝑎′ , 𝑏 ′ ) ∈ {(1,10); (2,5)} ⇒ (𝑎, 𝑏) ∈ {(13,130); (26,65‬‬
‫‪𝑑 + 𝑚 = 156‬‬
‫{ )‪6‬‬ ‫‪…  ; 𝑎 = 𝑑𝑎′ ; 𝑏 = 𝑑𝑏 ′ ; 𝑃𝐺𝐶𝐷(𝑎′ , 𝑏 ′ ) = 1‬‬
‫‪𝑚 = 𝑑2‬‬
‫‪ ⇒ 𝑑2 + 𝑑 − 156 = 0 ⇒ 𝑑 = 12 ⇒ 𝑚 = 144‬‬
‫𝑚‬
‫})‪𝑚 = 𝑑𝑎′ 𝑏 ′ ⇒ 𝑎′ 𝑏 ′ = = 12 ⇒ (𝑎′ ; 𝑏 ′ ) ∈ {(1; 12); (12; 1); (3; 4); (4; 3‬‬
‫𝑑‬
‫})‪⇒ (𝑎; 𝑏) ∈ {(12; 144); (144; 12); (36; 48); (48; 36‬‬
‫‪ -7‬التعداد ‪:‬‬
‫‪ .1‬لتحويل عدد من النظام العشري إلى نظام غير عشري نجري قسمات متتالية لهذا العدد على األساس ونكتب البواقي‬
‫المتحصل عليها من النهاية إلى البداية‬
‫‪2014 7‬‬ ‫‪1435 8‬‬
‫‪5‬‬ ‫‪287 7‬‬ ‫‪3‬‬ ‫‪179 8‬‬
‫‪0‬‬ ‫‪41‬‬ ‫‪7‬‬ ‫‪3‬‬ ‫‪22‬‬ ‫‪8‬‬
‫‪6‬‬ ‫‪5‬‬ ‫‪7‬‬ ‫‪6‬‬ ‫‪2‬‬ ‫‪8‬‬
‫‪5‬‬ ‫‪0‬‬ ‫‪2‬‬ ‫‪0‬‬

‫‪̅̅̅̅̅̅̅7‬‬
‫‪2014 = 5605‬‬ ‫‪̅̅̅̅̅̅̅8‬‬
‫‪1435 = 2633‬‬

‫‪11‬‬
‫‪ .2‬لتحويل عدد من نظام غير عشري إلى النظام العشري نضرب أرقام العدد في األساس مرفوع بقوة تناسب رتبة الرقم‬
‫̅̅̅̅̅̅̅‬
‫‪56057 = 5 + 0(7) + 6(7)2 + 5(7)3 = 2014‬‬
‫̅̅̅̅̅̅̅‬
‫‪26338 = 3 + 3(8) + 6(8)2 + 2(8)3 = 1435‬‬
‫أن كل أرقام العدد هي أصغر تماما‬ ‫‪ .3‬لحل المعادالت المشتملة على أعداد مكتوبة في أسس مختلفة ‪ ،‬ال بدّ من االنتباه ّ‬
‫من األساس ‪ ،‬وبعد كتابة األعداد في النظام العشري (غالبا) ما تحصلون على المعادلة المطلوب حلها في بداية السؤال‪.‬‬

‫مثال ‪:1‬‬
‫‪ )1‬حل في مجموعة األعداد الصحيحة المعادلة ‪7𝑥 − 9𝑦 = −19‬‬
‫̅̅̅̅̅ في نظام العد ذي األساس ‪ ،7‬و يُكتب ‪ 1𝛽3‬في نظام العد ذي األساس ‪.9‬‬
‫̅̅̅̅̅‬ ‫‪ )2‬نعتبر العدد الطبيعي 𝑛 الذي يُكتب ‪2𝛼5‬‬
‫عيّن ‪ α‬و ‪ ، β‬ث ّم اكتب العدد 𝑛 في النظام العشري‪.‬‬
‫‪7𝑥 − 9𝑦 = −19 ⇒ 𝑆 = {(9𝑘 + 5 ; 7𝑘 + 6)} ; 𝑘 ∈ ℤ‬‬
‫‪̅̅̅̅̅7 = 1𝛽3‬‬
‫‪𝑛 = 2𝛼5‬‬ ‫‪̅̅̅̅̅9 ⇒ 2(7)2 + 7𝛼 + 5 = 92 + 9𝛽 + 3‬‬
‫)‪⇒ 7𝛼 − 9𝛽 = −19 ⇒ (𝛼; 𝛽) = (9𝑘 + 5 ; 7𝑘 + 6‬‬
‫‪0≤𝛼<7‬‬ ‫‪0 ≤ 9𝑘 + 5 < 7‬‬
‫{‬ ‫{⇒‬ ‫‪⇒ 𝑘 = 0 ⇒ 𝛼 = 5 ; 𝛽 = 6 ; 𝑛 = 138‬‬
‫‪0≤𝛽<9‬‬ ‫‪0 ≤ 7𝑘 + 6 < 9‬‬
‫مثال ‪:2‬‬
‫‪ -1‬حل في ‪ ℤ2‬المعادلة ‪23𝑥 − 17𝑦 = 6 :‬‬
‫‪ -2‬استنتج األعداد الطبيعية ‪ A‬األصغر من ‪ 1000‬حيث ]‪ 𝐴 ≡ 2[23‬و ]‪𝐴 ≡ 8[17‬‬
‫‪ -3‬اكتب هذه األعداد في النظام ذي األساس ‪7‬‬
‫‪23𝑥 − 17𝑦 = 6 ⇒ 𝑆 = {(17𝑘 + 1 ; 23𝑘 + 1)} ; 𝑘 ∈ ℤ‬‬
‫)‪𝐴 = 23𝑥 + 2 = 17𝑦 + 8 ⇒ 23𝑥 − 17𝑦 = 6 ⇒ (𝑥; 𝑦) = (17𝑘 + 1 ; 23𝑘 + 1‬‬
‫‪𝑘 = 0 ∶ (𝑥; 𝑦) = (1 ; 1) ; 𝐴 = 25 = ̅34‬‬
‫‪̅̅̅7‬‬
‫̅̅̅̅̅̅̅ = ‪𝑘 = 1 ∶ (𝑥; 𝑦) = (18 ; 24) ; 𝐴 = 416‬‬
‫‪11337‬‬
‫‪̅̅̅̅̅̅̅7‬‬
‫‪𝑘 = 2 ∶ (𝑥; 𝑦) = (35 ; 47) ; 𝐴 = 807 = 2232‬‬

‫‪ -8‬تذكّر ‪:‬‬
‫‪2‬‬
‫فإن 𝑑 يقسم 𝑏𝛽 ‪(𝛼; 𝛽) ∈ ℤ ، 𝛼𝑎 +‬‬ ‫‪ .1‬إذا كان 𝑑 يقسم 𝑎 و 𝑑 يقسم 𝑏 ّ‬
‫فإن 𝑎 يقسم 𝑐 )𝑠𝑠𝑢𝑎𝐺(‬‫‪ .2‬إذا كان 𝑎 يقسم 𝑐𝑏 و 𝑎 أولي مع 𝑏 ّ‬
‫فإن ‪ 𝑃𝐺𝐶𝐷(𝑎; 𝑏; 𝑐) = 1‬مهما يكن العدد 𝑐‬ ‫‪ .3‬إذا كان ‪ّ 𝑃𝐺𝐶𝐷(𝑎; 𝑏) = 1‬‬
‫)𝑛 𝑛‬
‫فإن ‪ 𝑃𝐺𝐶𝐷(𝑎 ; 𝑏 = 1‬مهما يكن العدد 𝑛‬ ‫‪ .4‬إذا كان ‪ّ 𝑃𝐺𝐶𝐷(𝑎; 𝑏) = 1‬‬
‫‪ .5‬إذا ُو ِجدت ثنائية ‪ (𝛼; 𝛽) ∈ ℤ2‬حيث ‪ّ ، 𝛼𝑎 + 𝛽𝑏 = 1 :‬‬
‫فإن 𝑎 و 𝑏 أوليان فيما بينهما )𝑡𝑢𝑜𝑧‪(𝐵é‬‬
‫‪ .6‬إذا كان 𝑎 و 𝑏 أوليين فيما بينهما ّ‬
‫فإن ‪ 𝑎 + 𝑏 :‬و 𝑏𝑎 أوليان فيما بينهما ‪ ،‬كذلك ‪ (𝑎 + 𝑏)2‬و 𝑏𝑎 أوليان فيما بينهما‪.‬‬

‫‪‬‬
‫‪‬‬

‫‪11‬‬
‫حل معادلة من الدرجة الثانية‬ ‫‪.1‬‬
‫أ‪𝑥 2 + 3𝑥 − 4 = 0 .‬‬
‫)‪w5(EQN)3(𝑎𝑥 2 + 𝑏𝑥 + 𝑐 = 0) 1=(𝑎)3=(𝑏) p4=(𝑐)= (1)= (−4‬‬
‫ب‪𝑥 2 + 2𝑥 + 17 = 0 .‬‬
‫)𝑖‪C1=(𝑎)2=(𝑏)17=(𝑐)= (−1 + 4𝑖)= (−1 − 4‬‬
‫حل معادلة من الدرجة الثالثة 𝟎 = 𝟏 ‪𝒙𝟑 + 𝒙𝟐 + 𝒙 +‬‬ ‫‪.2‬‬
‫)𝑑(=‪w5(EQN)4(𝑎𝑥 3 + 𝑏𝑥 2 + 𝑐𝑥 + 𝑑 = 0) 1=(𝑎) 1=(𝑏) 1=(𝑐) 1‬‬
‫)𝑖‪=(−1)= (𝑖)= (−‬‬
‫𝟖 = 𝒚𝟑 ‪𝟐𝒙 +‬‬
‫{‬ ‫حل جملة معادلتين‬ ‫‪.3‬‬
‫𝟏‪𝟕𝒙 − 𝟒𝒚 = −‬‬
‫=‪w5(EQN)1(𝑎𝑛 𝑥 + 𝑏𝑛 𝑦 = 𝑐𝑛 ) 2=3=8=7=p4=p1‬‬
‫)‪=(1)= (2‬‬
‫𝟎 = 𝟑 ‪𝒙 − 𝟐𝒚 + 𝒛 −‬‬
‫حل جملة ‪ 3‬معادالت (تقاطع ‪ 3‬مستويات في الفضاء) 𝟎 = 𝒛 ‪{ −𝟐𝒙 + 𝒚 +‬‬ ‫‪.4‬‬
‫𝟎= 𝟔‪𝒙+𝒚+𝒛−‬‬
‫=‪w5(EQN)2(𝑎𝑛 𝑥 + 𝑏𝑛 𝑦 + 𝑐𝑛 𝑧 = 𝑑𝑛 ) 1=p2=1=3=p2‬‬
‫)‪1=1=0=1=1=1=6==(2)= (1)= (3‬‬
‫أن قيم 𝑑 تنقل إلى الطرف الثاني( )‪(𝑃) ∩ (𝑄) ∩ (𝐴𝐵𝐶) = 𝐼(2; 1; 3‬‬ ‫)الحظ ّ‬
‫𝒙𝟑‬
‫تعيين نقاط مساعدة لرسم منحنى بياني ) 𝒆 ‪ 𝒇(𝒙) = 𝒙 − 𝟒 + 𝐥𝐧(𝟏 +‬على المجال ]𝟓 ;𝟓‪[−‬‬ ‫‪.5‬‬
‫=)‪w7(TABLE)Q) (X)p4+h1+qh(𝑒 𝑥 )3 Q)$‬‬
‫)الخطوة ? ‪)1=( Step‬نهاية المجال? ‪)5=( End‬بداية المجال? ‪p5=( Start‬‬
‫حصر العدد ‪ α‬الذي يعدم الدالة 𝒙𝒏𝒍𝒙 ‪𝒈(𝒙) = 𝟏 + 𝒙 −‬‬ ‫‪.6‬‬
‫=‪w71+Q)pQ)hQ)=1=5=1‬‬
‫أن ‪ .3 < 𝛼 < 4‬إليجاد حصر للعدد ‪ α‬سعته ‪ 10−1‬نقوم بالعملية التالية ‪:‬‬ ‫نالحظ ّ‬
‫)? ‪C=3=(Start ?)4=(End ?)0.1=(Step‬‬
‫أن ‪ .3,5 < 𝛼 < 3,6‬بنفس الطريقة يمكننا إيجاد حصر للعدد ‪ α‬سعته ‪10−2‬‬ ‫نستنتج ّ‬
‫حيث نحدّد بداية المجال عند ‪ 3,5‬ونهايته عند ‪ 3,6‬والخطوة ‪0,01‬‬

‫‪ .7‬كتابة عدد مركب على شكله الجبري‬


‫)𝑖‪1) 𝑧1 = (1 + 2𝑖)2 : w2(CMPLX)(1+2b(i))d=(−3 + 4‬‬
‫𝑖‪5+11√3‬‬
‫= ‪2) 𝑧2‬‬ ‫=‪: Ca5+11s3$bR7p4s3$b‬‬
‫𝑖‪7−4√3‬‬
‫‪ .8‬تعيين طويلة وعمدة عدد مركب‬
‫)‪1) 𝑧1 = 1 + 𝑖 : 1+bq2(CMPLX)3(𝑟∠𝜃)= (√2∠45‬‬
‫إليجاد قيمة ‪ θ‬بالراديان نقوم بالعملية التالية ‪qw4(Rad) :‬‬
‫‪2) 𝑧2 = (3 + √3) + (−3 + √3)𝑖 :‬‬
‫‪1‬‬
‫)𝜋 ‪3+s3$+(p3+s3$)bq23=(2√6∠ − 12‬‬
‫𝑖‪1+‬‬ ‫‪√2‬‬ ‫‪1‬‬
‫= ‪3) 𝑧3‬‬ ‫)𝜋 ‪: a1+bRs3$+b$q23=( 2 ∠ 12‬‬
‫𝑖‪√3+‬‬

‫‪19‬‬
‫سي إلى الشكل الجبري‬
‫‪ .1‬لالنتقال من الشكل المثلثي أو األ ّ‬
‫𝜋‬
‫𝜋‬ ‫𝜋‬
‫‪1) 𝑧1 = 4 (cos + 𝑖 sin ) = 4𝑒 𝑖 :‬‬ ‫‪4‬‬
‫‪4‬‬ ‫‪4‬‬
‫)𝑖‪4qz(∠)aqK(𝜋)R4$q2(CMPLX)4(𝑎 + 𝑏𝑖)=(2√2 + 2√2‬‬
‫𝜋‬
‫𝜋‬ ‫𝜋‬
‫‪2) 𝑧2 = √5 (cos 6 + 𝑖 sin 6 ) = √5𝑒 𝑖 6 :‬‬
‫‪√15‬‬ ‫‪√5‬‬
‫(=‪s5$qzaqKR6$q24‬‬ ‫‪+‬‬ ‫)𝑖‬
‫‪2‬‬ ‫‪2‬‬
‫‪ .11‬حساب العدد المشتق‬
‫‪2‬‬
‫‪𝑓(𝑥) = 𝑥 − 1 + 2 ln 𝑥 ; 𝑥0 = 2 :‬‬
‫𝑑‬
‫)‪qy(𝑑𝑥)Q)dp1+2hQ))$2(𝑥0 )=(5‬‬
‫‪ .11‬حساب التكامل‬
‫‪2‬‬ ‫) 𝑥‪2‬‬
‫=𝐼‬ ‫𝑥( ‪∫1‬‬ ‫𝑒‪+ 2 − 3‬‬ ‫‪𝑑𝑥 :‬‬
‫=)‪yE2R1!!(Q)+2p3qh2Q)$‬‬

‫‪ .12‬حساب حدود متتالية عددية‬


‫𝑛𝑢‬ ‫‪√2‬‬
‫= ‪𝑢𝑛+1‬‬ ‫= ‪; 𝑢0 = 1 : aQ)RsQ)d+1r1=(𝑢1‬‬ ‫)‬
‫‪2 +1‬‬ ‫‪2‬‬
‫𝑛𝑢√‬

‫‪√3‬‬ ‫‪1‬‬
‫= ‪rM=(𝑢2‬‬ ‫‪)rM=(𝑢3‬‬ ‫… )‪= 2‬‬
‫‪3‬‬
‫المقربة لكسر أو جذر‬
‫ّ‬ ‫‪ .13‬إليجاد القيم‬
‫‪2‬‬ ‫‪3‬‬ ‫‪23‬‬ ‫‪23‬‬
‫)‪+ 4 = 20 = 1,15 : a2R5$+a3R4=(20)n(1,15‬‬
‫‪5‬‬
‫=‪√2 + √8 = 3√2 : s2$+s8‬‬
‫=‪√2 + √8 ≈ 4,24 : s2$+s8q‬‬

‫‪ .14‬إليجاد معادلة مستقيم االنحدار (خاص بشعبة التسيير واالقتصاد)‬


‫مثال ‪ :‬التمرين الثاني للموضوع األول (بكالوريا ‪)2111‬‬
‫األقدمية )بالسنوات( 𝑖𝑥‬ ‫‪2‬‬ ‫‪8‬‬ ‫‪15‬‬ ‫‪19‬‬ ‫‪24‬‬
‫األجرة )بالدنانير( 𝑖𝑦‬ ‫‪32400‬‬ ‫‪35400‬‬ ‫‪39600‬‬ ‫‪41400‬‬ ‫‪44700‬‬

‫‪ (2=8=15=19=24=R$‬إدخال قيم 𝑖𝑥)‪w32‬‬


‫=‪ (32400=35400=39600‬إدخال قيم 𝑖𝑦)‬
‫=‪41400=44700=Cq152‬‬
‫)‪(𝑎 = 556,356)q151=(𝑏 = 31133,55‬‬

‫مالحظة ‪ :‬يمكننا أيضا الحصول على إحداثيي النقطة المتوسطة بإجراء العمليات التالية ‪:‬‬
‫)‪q142= (𝑥̅ = 13; 6)q145= (𝑦̅ = 38700‬‬

‫‪‬‬
‫‪‬‬

‫‪11‬‬

You might also like